ITE 2022

Lakukan tugas rumah & ujian kamu dengan baik sekarang menggunakan Quizwiz!

Current guidelines recommend moderate-intensity exercise (approximately 3.0-5.9 METs) for 150 minutes per week. Which one of the following activities is equivalent to this level of energy expenditure? A) Brisk walking B) High-intensity interval training C) Light housework D) Jogging E) Sitting at a desk

ANSWER: A A MET is the amount of energy used by the body per minute of activity. Light intensity is <3 METs and includes activities such as sitting at a desk, light housework, casual walking, and stretching. Moderate intensity is 3.0-5.9 METs and includes brisk walking, water aerobics, and ballroom dancing. Vigorous intensity is >6 METs and is represented by activities such as high-intensity interval training, jogging, and heavy gardening

A 45-year-old male presents for follow-up of migraine headaches, which had previously responded well to occasional as-needed use of ibuprofen but have recently worsened in severity and frequency. He is interested in trying an abortive therapy. Triptan use would be CONTRAINDICATED with a history of which one of the following in this patient? A) Coronary artery stent placement B) Depression with psychotic features C) Diabetes mellitus with a hemoglobin A1c >7.5% D) Hypertension requiring two medications to achieve control E) Stage 4 chronic kidney diseas

ANSWER: A Because of their vasoconstricting properties, triptan medications are contraindicated in patients with established coronary artery disease, cerebrovascular disease, or peripheral vascular disease; patients with uncontrolled or multiple cardiovascular risk factors; and patients with certain high-risk migraine syndromes, including basilar and hemiplegic migraines. Triptan use for the treatment of migraine headaches would not be contraindicated with a history of depression with psychotic features, poorly controlled diabetes mellitus, hypertension requiring two medications, or stage 4 chronic kidney disease.

A 55-year-old male comes to your clinic for follow-up of his recent diagnosis of New York Heart Association class II heart failure with an ejection fraction of 40%. His past medical history is notable only for coronary artery disease. His current medications include the following: Aspirin, 81 mg daily Atorvastatin (Lipitor), 80 mg daily Furosemide (Lasix), 40 mg daily Lisinopril (Zestril), 40 mg daily Metoprolol succinate (Toprol-XL), 100 mg daily Spironolactone (Aldactone), 25 mg daily Today he is asymptomatic. His vital signs include a temperature of 37.0°C (98.6°F), a blood pressure of 118/75 mm Hg, and a heart rate of 60 beats/min. A physical examination is unremarkable. Which one of the following additional medications would be most appropriate to reduce his risk for worsening heart failure? A) Dapagliflozin (Farxiga) B) Digoxin C) Isosorbide dinitrate/hydralazine (BiDil) D) Ivabradine (Corlanor) E) Liraglutide (Victoza)

ANSWER: A The prevalence of heart failure has continued to increase due to the aging population in the United States. Dapagliflozin is approved by the FDA for the treatment of New York Heart Association class II-IV heart failure with reduced ejection fraction regardless of the presence of diabetes mellitus. Notably, recent studies showed a reduction in the worsening of heart failure and death from cardiovascular causes. Digoxin may be initiated in patients who remain symptomatic despite optimal therapy with other agents, but it does not affect morbidity or mortality. Isosorbide dinitrate/hydralazine provides a mortality benefit in patients who are unable to tolerate an ACE inhibitor or angiotensin receptor blocker. Ivabradine is a sinus node modulator and may reduce hospitalization or cardiovascular death in patients with a resting heart rate >70 beats/min who are taking a beta-blocker at maximal dosage. Liraglutide reduces cardiovascular events in patients with diabetes but has no role in the treatment of heart failure.

An 80-year-old female is brought to the emergency department after her family finds her on the floor at her home. They state that she has been confused and not acting like herself for the past 5-7 days. She lives independently and cares for herself, and she is active in a senior church group. The patient is unable to explain why she was on the floor, so the history is obtained from the family. There are no other associated symptoms and no known inciting incident. There have been no recent medication changes and the family is unaware of any fever or chills. The patient has not had episodes like this in the past. Her past medical history is notable for controlled blood pressure and a TIA several years ago with no residual neurologic deficit or impairment. A physical examination is negative except for her confusion, and there are no focal neurologic findings. Imaging shows no acute process. Which one of the following is the most likely explanation for these findings? A) Alzheimer's disease B) Delirium C) Ischemic stroke D) Mild neurocognitive disorder E) Vascular dementia

ANSWER: B This patient presents with acute altered mental status. Delirium should always be considered in this setting because it is both common and frequently overlooked (SOR C). There are multiple potential causes of this patient's acute altered mental status, including but not limited to systemic infections, metabolic disturbances, medications, systemic conditions, and central nervous system insults such as ischemic stroke. The clinical examination does not indicate the focal neurologic changes of a stroke, and this patient's TIA was several years ago with no subsequent chronic cognitive changes. The history and examination in this case do not suggest the presence of chronic cognitive changes indicative of conditions such as Alzheimer's disease, mild neurocognitive disorder, or vascular dementia. Alzheimer's disease has an insidious and gradual onset of cognitive symptoms. In vascular dementia, the symptoms begin after cerebrovascular events. Importantly, in the setting of acute mental status changes, the Choosing Wisely campaign recommends not presuming a diagnosis of dementia in an older adult with acute symptoms of confusion without first assessing for delirium.

A 62-year-old male presents with bright red hemoptysis. While not severe, it is recurrent and has persisted for several weeks. He is otherwise asymptomatic. A thorough history and physical examination does not provide any additional clues. Gastrointestinal and ear, nose, and throat etiologies are considered and are not thought to be the cause. Posteroanterior and lateral chest radiographs are normal. Which one of the following would be the most appropriate next step in diagnosis? A) Antineutrophil cytoplasmic antibody (ANCA) testing B) A sputum smear for acid-fast bacillus C) Sputum cytology D) CT of the chest E) Bronchoscopy

ANSWER: D A chest radiograph is appropriate in the initial evaluation of hemoptysis (SOR C). If the chest radiograph does not indicate a cause, then CT or CT angiography with intravenous contrast should be performed (SOR C). CT has become the preferred modality over bronchoscopy because it is more effective in determining the etiology. If CT does not identify the cause, bronchoscopy would be the next step. In addition, other tests including a sputum Gram stain, acid-fast bacillus smear, or sputum cytology can be useful depending upon the clinical situation. If there are concerns about the possibility of immunologic, rheumatologic, or vasculitic disease, testing for immunologic antibodies such as antineutrophil cytoplasmic antibody (ANCA) can be ordered.

Once hemolysis is excluded, the most common cause of unconjugated hyperbilirubinemia is A) alcoholic liver disease B) biliary tract disease C) fatty liver disease D) Gilbert syndrome E) Wilson disease

ANSWER: D Unconjugated hyperbilirubinemia can be defined as an elevated indirect bilirubin level. While unconjugated hyperbilirubinemia is most commonly seen in hemolysis, another common cause is Gilbert syndrome, which stems from a genetic defect that affects how the liver processes bilirubin. Alcoholic liver disease, biliary tract disease, fatty liver disease, and Wilson disease do not lead to unconjugated hyperbilirubinemia.

A 30-year-old gravida 2 para 1 at 20 weeks gestation presents with a 1-day history of a fever, cough, headache, and myalgias. A nasal swab confirms influenza B. Which one of the following is the preferred antiviral treatment for this patient? A) No treatment because of her pregnancy B) No treatment because antiviral medication is indicated for influenza A but not influenza B C) Baloxavir marboxil (Xofluza) D) Oseltamivir (Tamiflu) E) Peramivir (Rapivab)

ANSWER: D Antiviral medications are recommended for the treatment of influenza only within 48 hours of symptom onset (SOR A). However, in high-risk patient populations and in severe cases of disease, antiviral medications should be provided regardless of the duration of symptoms (SOR B). According to the CDC, oseltamivir remains the drug of choice for the treatment of influenza A and B during pregnancy because it has good safety data. Baloxavir marboxil is indicated for patients >12 years of age but should be avoided during pregnancy. There is less safety data for peramivir and zanamivir.

In adolescents and adults, what proportion of cases of uncomplicated, acute bronchitis are caused by atypical organisms such as Mycoplasma pneumoniae and Chlamydia pneumoniae?

Atypical organisms such as Mycoplasma pneumoniae and Chlamydia pneumoniae are rare causes and have been found in less than 1% of cases of acute bronchitis. Acute bronchitis is caused by a viral infection in 90%-99% of cases.

A 22-year-old female presents to your clinic after awakening to find a rash on her hands (shown below). She does not have any other medical issues today. Which one of the following is the most likely cause of this rash?

A) Addison's disease B) An allergic reaction C) Cellulitis D) A chemical burn E) Phytophotodermatitis . . . . . . . . . . . . . . . . . . . . . . . . . . . ANSWER: E Phytophotodermatitis is an inflammation and/or discoloration of the skin caused by contact with specific plants followed by exposure to sunlight. Limes are commonly associated with this phenomenon. Addison's disease causes generalized hyperpigmentation and has an insidious onset along with other constitutional symptoms such as anorexia, nausea, and weakness. An allergic reaction would likely be pruritic and a chemical burn would be expected to be painful. Cellulitis would also be uncomfortable and would likely be associated with erythema.

A 72-year-old female presents with progressive hand pain and stiffness. She is a seamstress and is concerned because sewing has been more difficult over the past 6 months. She recalls that her mother's hands were misshapen, but her mother never received a diagnosis. You examine her hands, which are shown below. Which one of the following would be the most appropriate pharmacotherapy?

A) Colchicine (Colcrys) B) Diclofenac (Zorvolex) C) Hydroxychloroquine (Plaquenil) D) Infliximab (Remicade) injections E) Methotrexate (Trexall) . . . . . . . . . . . . . . . . . . . . . . . . . . . . . . . . ANSWER: B This patient presents with erosive osteoarthritis that involves the distal interphalangeal (DIP) and proximal interphalangeal (PIP) joints with sparing of the metacarpophalangeal (MCP) joints. The primary goals for treating osteoarthritis are to control symptoms such as pain and stiffness and optimize function in order to preserve quality of life. Topical or oral NSAIDs are the most appropriate pharmacotherapy for osteoarthritis of the hand. Colchicine and methotrexate have not been studied for the treatment of osteoarthritis and their use for this condition is not recommended. Colchicine is indicated for the treatment of gout, which is usually pauciarticular and asymmetrical, and methotrexate is effective for rheumatoid arthritis. Conventional synthetic and biologic disease-modifying medications such as hydroxychloroquine and infliximab have not been shown to be effective in the treatment of osteoarthritis. These medications are appropriate for the treatment of systemic lupus erythematosus and rheumatoid arthritis, which have examination findings that involve the MCP and PIP joints but spare the DIP joints.

A 23-year-old male presents with a skin rash on his chest and back. An examination reveals widespread, slightly pink macules on his chest and back. A KOH preparation is shown below. Which one of the following would be the most appropriate treatment?

A) Topical nystatin cream B) Topical selenium sulfide C) Topical triamcinolone cream D) Oral fluconazole (Diflucan) E) Oral nystatin . . . . . . . . . . . . . . . . . . . . . . . . . . . . . . . ANSWER: B This image shows typical hyphae of pityriasis versicolor, a superficial infection caused by yeasts in the genus Malassezia. Of the listed therapies, topical selenium sulfide would be the most appropriate first-line treatment. Topical antifungals such as terbinafine and miconazole are other first-line options. Oral fluconazole can be used, but oral therapy is usually reserved for when topical treatment is impractical or unsuccessful. Topical nystatin cream and oral nystatin are ineffective, and topical corticosteroids such as triamcinolone may temporarily suppress symptoms while exacerbating the infection.

A patient sees you because of eye pain and swelling that have been present for a few days. A physical examination reveals a small, pink, tender area of the upper eyelid with an adjacent, slightly inflamed gland opening at the eyelid margin (see image). Which one of the following would be the most appropriate initial treatment?

A) Warm compresses and gentle massage of the eyelid B) Topical moxifloxacin ophthalmic solution 0.5% (Vigamox) C) Topical tobramycin/dexamethasone ophthalmic solution 0.3%/0.1% (Tobradex) D) Oral cephalexin (Keflex) E) Incision and drainage . . . . . . . . . . . . . . . . . . . . . . . . . . ANSWER: A This patient has a hordeolum (stye). Typical first-line treatment is to apply warm compresses and perform gentle massage of the area to promote drainage of the occluded gland. Antibiotics and incision and drainage are not necessary unless surrounding cellulitis is present or there is failure to improve with initial therapy.

Which one of the following cardiovascular medications may lead to hyperthyroidism? A) Amiodarone B) Digoxin C) Flecainide D) Metoprolol E) Valsartan

ANSWER: A Amiodarone-induced thyrotoxicosis (AIT) is a less common cause of hyperthyroidism and can be particularly difficult to accurately diagnose and treat. AIT type 1 is a form of iodine-induced thyrotoxicosis caused by the high iodine content in amiodarone. AIT type 2 is a form of amiodarone-induced thyroiditis. Digoxin, flecainide, metoprolol, and valsartan do not cause hyperthyroidism.

Based on the current CDC treatment guidelines, which one of the following is recommended as first-line treatment of urethritis in a 24-year-old male who weighs 152 kg (335 lb), when nucleic acid amplification testing (NAAT) for gonorrhea is positive and Chlamydia testing is negative? A) One dose of ceftriaxone, 1 g intramuscularly B) One dose of ceftriaxone, 500 mg intramuscularly, plus one dose of azithromycin (Zithromax), 1 g orally C) One dose of ceftriaxone, 500 mg intramuscularly, plus doxycycline, 100 mg orally twice daily for 7 days D) One dose of gentamicin, 240 mg intramuscularly, plus one dose of azithromycin, 2 g orally

ANSWER: A In 2020, the CDC updated its treatment guidelines for gonococcal infections. The recommended first-line therapy for patients weighing >150 kg (330 lb) with gonococcal urethritis is one dose of ceftriaxone, 1 g intramuscularly. One dose of ceftriaxone, 500 mg intramuscularly, is recommended for those weighing <150 kg. Patients presenting with an unknown cause of urethritis, such as before urine or urethral nucleic acid amplification test results are known, should be prescribed a combination of one dose of ceftriaxone, 500 mg intramuscularly (1 g if >150 kg), and doxycycline, 100 mg orally for 7 days. Azithromycin, 1 g orally as a single dose, may be used as an alternative to doxycycline for treatment of chlamydial infection, but it is no longer the preferred agent in nonpregnant adults and adolescents. Intramuscular gentamicin is inferior to intramuscular ceftriaxone for the treatment of gonorrhea, even when used in combination with oral azithromycin (SOR B).

Cardiac stress testing would be most appropriate for which one of the following patients? A) A 57-year-old female who is scheduled for a knee replacement and has dyspnea when walking up a few stairs B) A 60-year-old male with diabetes mellitus who was admitted to the hospital for chest pain and acute stroke and has a normal EKG and troponin levels C) A 66-year-old male with diabetes and hypertension without cardiac symptoms who would like to stratify his risk for heart disease D) A 68-year-old female with coronary artery disease who is scheduled for a knee replacement and does not have cardiac symptoms when walking up a flight of stairs E) A 79-year-old male who is scheduled for a transcatheter aortic valve replacement for severe aortic stenosis and has dyspnea when walking up a few stairs?

ANSWER: A In the setting of acute symptoms, cardiac stress testing is indicated when there is an intermediate probability of acute coronary syndrome. Cardiac stress testing is also indicated in a preoperative assessment when surgery is at least a moderate risk and the patient cannot reach 4 METs of exertion (climbing a single flight of stairs) without cardiac symptoms. Cardiac stress testing is contraindicated after a recent stroke or TIA and in patients with severe symptomatic aortic stenosis. It is not indicated in asymptomatic patients with no history of revascularization.

A 12-year-old female presents with a sore throat and tonsillar exudate, and a rapid antigen test is positive for streptococcal pharyngitis. She returns to your office after completing a 10-day course of penicillin this morning. She says that although she saw some initial improvement, she now has a sore throat again, accompanied by a runny nose and cough. Her mother asks if another antibiotic would be appropriate. A physical examination reveals nonexudative pharyngitis, but a rapid antigen test for group A Streptococcus is again positive. Which one of the following would be the most appropriate treatment at this point? A) No further antibiotic therapy B) Oral azithromycin (Zithromax) for 5 days C) Oral ciprofloxacin (Cipro) for 10 days D) A single dose of intramuscular benzathine penicillin E) A single dose of intramuscular ceftriaxone

ANSWER: A Most bacteriologic treatment failures for group A Streptococcus (GAS) represent a GAS carrier state. This patient had clinical improvement followed by a second illness with typical features of a viral infection. Oral azithromycin, oral ciprofloxacin, intramuscular benzathine penicillin, and intramuscular ceftriaxone are not appropriate for the treatment of viral infections in a patient who is a pharyngeal GAS carrier.

A 45-year-old male sees you for a routine visit. His medical history includes hypertension treated with hydrochlorothiazide, amlodipine (Norvasc), and losartan (Cozaar). He also has type 2 diabetes treated with metformin and empagliflozin (Jardiance). Laboratory findings are significant for an LDL-cholesterol level of 167 mg/dL and you prescribe simvastatin (Zocor), 80 mg daily. At a follow-up visit 3 months later he tells you that he stopped taking the simvastatin after a week due to muscle pain and weakness. Which one of the following medications in this patient's current regimen most likely contributed to his risk for developing statin-induced myopathy? A) Amlodipine B) Empagliflozin C) Hydrochlorothiazide D) Losartan E) Metformin

ANSWER: A Most statins are metabolized in the liver by cytochrome P450 3A4 (CYP3A4) enzymes. In patients on statin therapy, concurrent use of other medications that are also metabolized by this system, including amiodarone, calcium channel blockers such as amlodipine, certain anti-HIV medications, and certain antifungal medications, can increase the risk of complications such as statin-induced myopathy. In this patient, only simvastatin and amlodipine are metabolized by CYP3A4. Losartan is metabolized by cytochrome P450 enzymes other than 3A4 (2C9), and this patient's other medications are metabolized by different mechanisms (empagliflozin) or not significantly metabolized (hydrochlorothiazide and metformin).

A 62-year-old female underwent a total knee replacement 3 months ago. She has no other surgical history and is in good health. Her dental office calls you to discuss antibiotic prophylaxis prior to a dental cleaning. Based on current guidelines, which one of the following would be most appropriate regarding antibiotic prophylaxis prior to routine dental procedures in this patient? A) No prophylaxis B) Prophylaxis for 6 months post knee replacement C) Prophylaxis for 1 year post knee replacement D) Prophylaxis for 5 years post knee replacement E) Prophylaxis for the patient's lifetime

ANSWER: A One of the most potentially devastating late complications of joint replacement surgery is infection of the prosthetic joint. Because dental procedures are known to induce transient bacteremia, the use of prophylactic antibiotics prior to dental procedures for patients with prosthetic joints was considered orthopedic dogma for many years. However, current evidence to support this practice is limited and antibiotic use is known to increase cost, bacterial resistance, and the risk of adverse drug reactions. In most cases the risks of antibiotic prophylaxis outweigh the likelihood of benefit. Recent guidelines from the American Dental Association and the American Academy of Orthopaedic Surgeons recommend against the routine use of prophylactic antibiotics for dental procedures in patients with a history of joint replacement, except for situations in which infectious risk is increased, such as immunocompromise or a history of a previous joint infection.

A 62-year-old male is found to have an alkaline phosphatase (ALP) level of 152 U/L (N 32-91). Laboratory studies performed last year showed an ALP level of 134 U/L. The review of systems today is negative, including for pain, nausea, and dyspnea. You note that his AST and ALT levels are in the normal range, and a gamma-glutamyl transaminase level is also normal. Which one of the following would be the most appropriate next step in the evaluation? A) Plain radiography of the skull, pelvis, and tibia B) Right upper quadrant ultrasonography C) A full-body CT scan D) A HIDA scan E) A radionuclide bone scan

ANSWER: A Paget disease of bone is the second most common metabolic bone disorder after osteoporosis and has a lifetime prevalence of 1%-2% in the United States. Only 30%-40% of patients have symptoms such as bone pain at diagnosis. Most patients are diagnosed after an incidental finding of elevated alkaline phosphatase (ALP) on routine laboratory studies or by plain films performed for another reason. When an elevated ALP level is found in an asymptomatic patient, other liver function tests such as a gamma-glutamyl transaminase level should be performed to evaluate for hepatobiliary pathology. If negative, this should be followed by plain radiography of the skull and tibia, and an enlarged view of the pelvis to assess for lytic lesions and cortical thickening. If plain radiography is consistent with Paget disease of bone, a radionuclide bone scan is performed to assess the full extent of the disease. Bisphosphonates are the first-line treatment in active disease, which is signified by bone pain, hearing loss, and lytic lesions. Right upper quadrant ultrasonography, a full-body CT scan, and a HIDA scan would not be the most appropriate next step in the evaluation.

Which one of the following is the most common radiologic finding in early pulmonary sarcoidosis? A) Bilateral hilar adenopathy B) Caseating granulomas C) Pleural granulomas D) Peribronchiolar thickening

ANSWER: A Sarcoidosis is an inflammatory disease that can affect many organ systems, but 90% of patients have pulmonary involvement. While many patients diagnosed with sarcoidosis are asymptomatic, pulmonary symptoms including dry cough, the gradual onset of dyspnea, and fatigue are nonspecific, and the condition is often not suspected until chest radiography is performed. The most common finding is bilateral hilar adenopathy alone (stage 1). Other findings, which usually develop over time, include infiltrates and, in some patients, ultimately fibrosis. The classic pathologic findings from biopsies are noncaseating granulomas. Caseating granulomas are indicative of tuberculosis. Pleural involvement is not typical in sarcoidosis. Peribronchiolar or peritracheal thickening and interstitial infiltrates may be seen on CT scans, but bilateral hilar adenopathy is the most characteristic finding in earlier pulmonary sarcoidosis and is readily seen on plain chest radiographs.

You see a 45-year-old male with fatigue, arthralgias, and mildly elevated liver function tests. You are considering hereditary hemochromatosis as a possible diagnosis. Which one of the following should you order first? A) A serum ferritin level and transferrin saturation B) Genetic testing for HFE mutations C) T2-weighted MRI for hepatic iron concentration D) A liver biopsy

ANSWER: A The initial tests used in the workup for suspected hemochromatosis are a serum ferritin level and transferrin saturation. A transferrin saturation >45% and a serum ferritin level >300 ng/mL in men or >200 ng/mL in women are indicative of iron overload and highly suggestive of hereditary hemochromatosis. A serum iron level is ordered as part of transferrin saturation testing, but an elevated iron level by itself is not as sensitive or specific as the other tests. Other etiologies of iron overload should be ruled out, including liver disease, alcohol abuse, and metabolic syndrome. If no secondary etiologies are found, genetic testing would be appropriate to identify HFE mutations indicating hereditary hemochromatosis. Genetic testing should not be performed in a patient without iron overload or a family history of hereditary hemochromatosis. MRI may help determine the risk of developing cirrhosis, and a liver biopsy is used to determine the amount of liver damage.

A 12-year-old female is brought to your office by her mother for evaluation of a knee abrasion sustained earlier that day when she fell off her bike onto a gravel driveway. Her immunizations are up to date, including Tdap vaccination last year. A physical examination reveals an irregular, superficial abrasion measuring roughly 1×2 cm, containing dirt and stony debris. Which one of the following infection prevention measures would be most appropriate in this situation? A) Irrigation with tap water B) Irrigation with 3% hydrogen peroxide C) A topical antibiotic D) An oral antibiotic E) A tetanus booster

ANSWER: A The most important aspect of infection prevention in treating a superficial wound is cleaning and irrigation. Studies have shown that irrigation with tap water provides similar outcomes compared to sterile saline (SOR B). Antiseptic solutions such as hydrogen peroxide are no more effective than tap water, can be caustic to wound tissue, and may delay healing (SOR C). Antibiotics should be used for treatment of wound infections; however, non-infected wounds do not routinely require antibiotic prophylaxis unless there is an increased risk of infection. Risk factors for wound infection include bite wounds, delayed presentation, retained foreign material, insufficient cleaning, puncture or crush wounds, open fractures, significant immunocompromise, and joint, cartilage, or tendon involvement. Patients with three or more doses of tetanus toxoid with the most recent vaccination within the past 5 years do not require a tetanus booster or tetanus immune globulin for prophylaxis, regardless of the type of wound.

A 37-year-old female presents to your clinic with a long-standing history of abnormal menstrual cycles, often occurring irregularly more than 40 days apart. She has ongoing struggles with weight gain, acne, and facial hair growth. She states that she is not currently sexually active. Her last Papanicolaou smear 2 years ago was normal. Her vital signs and a physical examination are unremarkable other than a BMI of 36 kg/m2. An office urine pregnancy test is negative. Laboratory evaluation reveals a hemoglobin A1c of 6.2%, and normal TSH, prolactin, and 17-hydroxyprogesterone levels. Which one of the following is required to confirm the most likely diagnosis? A) No additional evaluation B) A serum C-peptide test C) A dexamethasone suppression test D) Ultrasonography of the pelvis E) CT of the abdomen and pelvis

ANSWER: A This patient has signs and symptoms consistent with polycystic ovary syndrome (PCOS). The Rotterdam 2003 criteria are the most widely used diagnostic criteria for PCOS, endorsed by multiple national and international professional societies. These criteria require the presence of two out of the following three features: oligomenorrhea, hyperandrogenism, and the presence of polycystic ovaries on ultrasonography. When the first two of these criteria are clearly met, ultrasonography to establish the presence of polycystic ovaries is not required. Therefore, a diagnosis is already warranted for this patient and additional evaluation is not needed. When patients require imaging, pelvic ultrasonography is the preferred modality rather than CT. While this patient has evidence of insulin resistance, as is common for patients with PCOS, a C-peptide test is not indicated. Dexamethasone suppression testing is not indicated because this patient does not have any other clinical signs and symptoms that would be consistent with Cushing syndrome.

An 83-year-old female with a history of Alzheimer's dementia presents with concerns about worsening agitation in the evenings. She is accompanied by her daughter who has power of attorney. The patient is dependent on her daughter for all instrumental activities of daily living and requires assistance with certain core activities of daily living such as dressing and bathing. She has no other chronic medical problems. Her daughter states that starting around 4:00 p.m., the patient becomes increasingly disoriented and agitated. There has been no physical aggression, but the daughter asks for medical intervention to "help calm her down." Which one of the following should you recommend initiating first for the management of this patient's symptoms? A) Sensory stimulation including touch and music B) Cognitive training C) Haloperidol D) Quetiapine (Seroquel) E) Valproic acid.

ANSWER: A This patient is experiencing behavioral and psychological symptoms of dementia (BPSD) as her cognitive and functional status decline. Evening agitation is a common form of BPSD, often referred to as sundowning. As with most BPSD, a nonpharmacologic approach to improve agitation has much stronger evidence for efficacy compared to a pharmacologic approach. A recent network meta-analysis showed that sensory stimulation, including massage, touch, and music therapy, significantly outperforms pharmacologic intervention (level of evidence 1a). Cognitive training, especially for BPSD in the context of advanced dementia, does not have strong evidence to support its use. Pharmacologic intervention should be initiated with caution given the potential for side effects, and should be a last resort in situations where there is a risk of self-harm or harm to others.

A 35-year-old female presents to your office with a feeling of vague fullness in her neck for the last month. She has noticed a gradual onset of fatigue, constipation, and cold intolerance over that time. A few weeks ago the patient took a selfie and was surprised by how puffy her face appeared in the photo. On examination her thyroid is diffusely enlarged and nontender and feels pebbly on palpation. An HEENT examination, including an eye examination, is otherwise normal. Which one of the following is the most likely diagnosis? A) Chronic autoimmune (Hashimoto) thyroiditis B) Graves disease C) Lymphadenitis D) Lymphoma E) Thyroid cáncer

ANSWER: A This patient's clinical picture is most consistent with chronic autoimmune thyroiditis, traditionally known as Hashimoto thyroiditis. This diagnosis is suggested by her neck fullness and symptoms of hypothyroidism. Additionally, a nontender goiter that feels like pebbles on examination is classically reported with chronic autoimmune thyroiditis. Graves disease typically presents with symptoms of hyperthyroidism and, in many patients, orbitopathy (eye bulging). A patient with lymphadenitis typically shows symptoms of a causative infection. Lymphadenitis tends to rapidly enlarge the lymph nodes, which are also typically painful and tender. Lymphoma more commonly presents with fevers, night sweats, unintentional weight loss, itchy skin, and dyspnea. This patient lacks a discrete thyroid nodule, which makes thyroid cancer less likely. Thyroid nodules are more frequently painful, while the neck fullness in chronic autoimmune thyroiditis is usually painless and nontender.

Three days after a camping trip in New Hampshire a patient develops influenza-like symptoms of a fever, mild myalgias, and malaise followed by an expanding, erythematous, annular rash with central clearing on his thigh. Which one of the following is the most likely diagnosis for the rash? A) Erythema migrans B) Erythema multiforme C) Nummular eczema D) Pityriasis rosea E) Tinea corporis

ANSWER: A Annular lesions can be a presentation of several different conditions. This patient's history of possible tick exposure and current prodromal constitutional symptoms suggest acute Lyme disease. Erythema migrans is the characteristic rash of acute Lyme disease. Erythema multiforme can be spontaneous, related to a viral or Mycoplasma infection, or associated with a medication reaction. Prodromal symptoms are uncommon in limited erythema multiforme and the clinical context of this case suggests a different etiology. Nummular eczema is an intensely pruritic, annular lesion that is not associated with constitutional symptoms. Pityriasis rosea is thought to be viral in etiology and is usually otherwise asymptomatic. Tinea corporis is a fungal infection and is not associated with systemic symptoms.

Patients with autosomal dominant polycystic kidney disease are most likely to develop extrarenal cysts in which one of the following locations? A) Liver B) Pancreas C) Spleen D) Central nervous system E) Reproductive system

ANSWER: A Autosomal dominant polycystic kidney disease (ADPKD) is the most common hereditary cause of kidney disease and is a frequent cause of end-stage renal disease. The most common extrarenal cystic complication is the formation of liver cysts, which are found in >90% of patients with ADPKD who are older than 35 years of age. Other locations for ADPKD-related cyst formation include the pancreas, spleen, and reproductive system, although these are not as common as hepatic cysts. The most severe complication of ADPKD is intracranial aneurysms, which are 2-4 times more prevalent in patients with ADPKD than in the general population but are not as common as liver cysts.

You admit a 50-year-old female to the hospital with pneumonia secondary to COVID-19. Her medical history is significant for controlled major depressive disorder. Upon admission, she requires respiratory support with oxygen via a high-flow nasal cannula. You ask her wishes regarding emergency measures and she states that she does not wish to be resuscitated or intubated. She reports that she watched her mother die a "horrible death" from COVID-19 about 6 months earlier and does not want to put herself through that. She does not have an advance directive and her only living relative is her father who is cognitively impaired. You are hesitant to write the DNR/DNI order given her good health and high likelihood of survival if she were to clinically decline. Which one of the following would be most appropriate prior to writing a DNR/DNI order for this patient? A) No further actions prior to writing the order B) Administering a Patient Health Questionnaire-9 (PHQ-9) and writing the order if her score is <10 C) Asking a second attending physician to verify her wishes D) Consulting a psychiatrist for a formal assessment of mental capacity E) Consulting the hospital ethics board

ANSWER: A Individuals are assumed to have the mental capacity to make their own medical decisions unless there are reasons to question a patient's capacity, such as risk factors for impaired decision-making or abrupt changes in mental status. It is the responsibility of the care team to honor the reasonable wishes of each patient, including decisions regarding end-of-life care. DNR/DNI orders can be fluid and changed throughout a hospital stay as more information becomes available. Patients should be provided with the tools needed to make medical decisions and ensure that they have insight into the consequences of the decisions. A DNR/DNI order should be written at this patient's request without administering a depression screen or consulting other professionals.

A 57-year-old female presents for follow-up 6 weeks after falling and breaking her wrist. The fracture has healed but she has developed burning pain in the area of the fracture, which she rates as 9 on a scale of 10. On examination you note moderate hyperalgesia in the area with some minimal localized swelling. You suspect complex regional pain syndrome (CRPS). CRPS is best diagnosed with which one of the following? A) The patient history and physical examination B) Ultrasonography C) MRI D) Nerve conduction testing E) A technetium 99m bone scan

ANSWER: A Complex regional pain syndrome (CRPS) usually develops after an injury, often a fracture, to a distal extremity, although it can present without prior injury. The diagnosis is made clinically using the history and physical examination. Its pathophysiology is poorly understood. Ultrasonography or MRI may be used to rule out other diagnoses but are not necessary for the diagnosis of CRPS. Nerve injury can be seen on nerve conduction testing with type 2 CRPS, also known as causalgia, but nerve injury is not always identified with type 1 CRPS, also known as reflex sympathetic dystrophy. Nerve conduction testing is not necessary for making the diagnosis, and both types of CRPS are treated with the same approach. A technetium 99m bone scan may reveal increased bone resorption at the site, but it is neither sensitive nor specific for CRPS.

A 15-year-old female presents for a well adolescent examination and reports painful, heavy periods. Menarche occurred at age 11, and by age 12 her menses were regular but quite painful. She misses at least 1 day of school each month due to the discomfort. She has tried acetaminophen/caffeine/pyrilamine (Midol Complete) and ibuprofen, 200 mg, without much relief. She is not sexually active. Her past medical history and surgical history are unremarkable. A urine pregnancy test is negative. Which one of the following would be the most appropriate next step for this patient? A) Empiric treatment with maximum-dose naproxen B) Screening for sexually transmitted infections C) A pelvic examination D) Pelvic ultrasonography E) Referral to a gynecologist

ANSWER: A Dysmenorrhea affects 50%-90% of females and the great majority of cases are primary dysmenorrhea, or pain that occurs in the absence of pelvic pathology. After a complete history confirming cyclic cramping pelvic pain beginning around the start of menses and a negative urine pregnancy test, empiric treatment should be offered (SOR C). First-line treatment is an NSAID at moderate to maximum dosing, such as naproxen, 500 mg every 12 hours. Any NSAID can be used and should be started 1-2 days before the onset of menses and continued through the first several days of bleeding. A secondary benefit to NSAID use is a reduction in heavy menstrual bleeding. Combined estrogen/progestin oral contraceptives may also be used as first-line therapy or in conjunction with NSAIDs. While screening for sexually transmitted infections is important for sexually active adolescents, it is not indicated in the evaluation of dysmenorrhea. Neither pelvic examination nor imaging is indicated when the history is consistent with primary dysmenorrhea. If there is evidence of secondary dysmenorrhea (due to pelvic pathology or a recognized medical condition), then an examination and imaging are indicated. Family physicians are able to manage the majority of cases of primary dysmenorrhea. If there is no improvement in treatment after 3 months, referral to a gynecologist may be indicated.

You receive a call from a hospitalist reporting that a 77-year-old male who is a patient of yours has been admitted to the hospital. You note that this is his third hospital admission over the last 4 months, due to a variety of acute medical concerns. His medical history includes heart failure, mild dementia, chronic kidney disease, and type 2 diabetes. You consider transitions of care planning for this patient upon discharge. Which one of the following interventions would offer the best efficacy for this patient? A) Rapid medication reconciliation, a home visit, and follow-up with you within 7 days of discharge B) Rapid medication reconciliation, a home visit, and follow-up with his endocrinologist within 7 days of discharge C) Rapid post-discharge laboratory studies, and follow-up with his cardiologist within 7 days of discharge D) Rapid post-discharge laboratory studies, and follow-up with both you and his cardiologist within 7 days of discharge E) Rapid medication reconciliation, rapid post-discharge laboratory studies, and follow-up with his endocrinologist within 7 days of discharge

ANSWER: A Given this patient's substantial burden of multimorbidity as well as ongoing patterns of high acute care utilization, he is at very high risk for readmission upon discharge. Although there has been significant heterogeneity across interventions over the last decade in optimizing transitions of care for high-risk patients, many analyses, including systematic reviews, have identified medication reconciliation and close follow-up with a primary care physician to be among the components critical to success, along with a home visit from either a nursing team member or a licensed independent practitioner. There is no clear evidence that rapid post-discharge laboratory studies improve outcomes. While there may be a tendency to focus on disease-specific interventions via specialty care, the preponderance of evidence supports a holistic focus through comprehensive primary care follow-up. Primary care continuity and accountability for care after discharge are also key components valued by patients and caregivers.

A 45-year-old male with no known medical history presents as a new patient for a physical examination. A review of symptoms is negative. He notes that his father died in his fifties of heart disease, but the patient does not know any further details. An examination is notable for a systolic murmur at the lower left sternal border. Which one of the following additional findings in this patient would be most consistent with hypertrophic cardiomyopathy? A) Decreased intensity of the murmur when supine B) Decreased intensity of the murmur with the Valsalva maneuver C) Elevated jugular venous distention D) Elevated pulse pressure E) A differential in blood pressure between the arms

ANSWER: A Hypertrophic cardiomyopathy (HCM), formerly known as idiopathic hypertrophic subaortic stenosis, is a common and underdiagnosed form of inherited heart disease with a prevalence of 1:500 in the United States and worldwide. HCM is associated with a systolic murmur at the lower left sternal border with an intensity that changes along with changes to preload of the heart. Lying down increases preload, which decreases the murmur. The Valsalva maneuver decreases preload and increases the murmur. Advanced HCM may cause heart failure and jugular venous distention, but at that stage symptoms would be expected. Elevated pulse pressures are classically seen with aortic insufficiency rather than HCM. Differential blood pressures in the arms would not be expected with HCM.

Which one of the following is necessary to make the diagnosis of a functional gastrointestinal disorder? A) Symptom-based clinical criteria B) Noninvasive testing for Helicobacter pylori infection C) Celiac serology D) Gastric emptying studies E) Esophagogastroduodenoscopy

ANSWER: A In the absence of red-flag symptoms such as nocturnal defecation, weight loss, or gastrointestinal bleeding, functional gastrointestinal disorders can be diagnosed using symptom-based clinical criteria. Symptoms such as recurrent abdominal pain related to defecation, pain related to a change in the frequency of defecation, abdominal bloating and distension, and loose and watery or lumpy and hard stools are used to diagnose functional bowel disorders. Noninvasive testing for Helicobacter pylori, celiac serology, gastric emptying studies, and esophagogastroduodenoscopy are not required in order to make a diagnosis.

A 24-year-old gravida 2 para 2 presents to your office for evaluation of breast pain. The patient reports that the pain affects both breasts, particularly during the week prior to her menses. She does not report any redness, nipple discharge, or fever. This is the first time the patient has presented for evaluation of this problem. In addition to conservative measures, which one of the following would be most appropriate to manage the patient's pain without interfering with fertility? A) Diclofenac gel (Voltaren Arthritis Pain) B) Danazol C) Goserelin (Zoladex) D) Tamoxifen (Soltamox)

ANSWER: A Mastalgia is a common symptom requiring evaluation in the primary care setting. Cyclic mastalgia accounts for about two-thirds of all breast pain and is thought to be caused by increased sensitivity of the breast tissues to hormonal stimulation during the luteal phase of the menstrual cycle. Topical NSAIDs such as diclofenac are the first-line pharmacologic treatment for mastalgia (SOR B). Danazol is the only drug that is approved by the FDA for treatment of mastalgia, but it is poorly tolerated due to menorrhagia, muscle cramps, weight gain, and other androgenic effects. Goserelin is only indicated for severe, refractory mastalgia. Tamoxifen is more effective and better tolerated than danazol, but is associated with hot flashes, vaginal discharge, venous thromboembolism, endometrial cancer, and teratogenicity.

A 20-year-old pregnant female presents to your clinic to discuss the possibility of delivering her baby at her home. A few of her friends have had their babies at home and she asks for your advice about her delivery. Which one of the following presents the greatest risk with an out-of-hospital birth? A) Nulliparity B) Maternal age <25 C) Prior vaginal delivery D) Single gestation E) Cephalic presentation

ANSWER: A Of the options listed, any primigravida should be encouraged to consider a hospital delivery rather than an out-of-hospital delivery. Pregnant patients with a previous cesarean delivery, multiple gestation (twins or higher), or fetal malpresentation (breech or other) should also be strongly encouraged to deliver in a hospital setting (SOR B). Similarly, pregnant patients at >41 weeks gestation should be encouraged to pursue a hospital delivery (SOR B). The age of the mother is not relevant in this decision unless there are other related increased risks and comorbidities that need to be considered. Patients who do plan a community birth should ensure that their maternity and neonatal health professionals are licensed and meet international Confederation of Midwives Global Standards for Midwifery Education, are practicing within an integrated and regulated health system, and have access to timely and safe transport to a hospital if necessary (SOR C). Unassisted childbirth should also be strongly discouraged (SOR C).

Which one of the following disorders is caused by an underlying mechanism of osteochondrosis rather than apophysitis? A) Legg-Calvé-Perthes disease B) Osgood-Schlatter disease C) Sever's disease D) Sinding-Larsen-Johansson syndrome

ANSWER: A Osteochondrosis refers to degenerative changes in the epiphyseal ossification areas of growing bones. Legg-Calvé-Perthes disease is a type of osteochondrosis that affects the femoral head. Patients with Legg-Calvé-Perthes disease should be referred to an orthopedist and instructed to avoid all weight-bearing activities until reossification occurs. Osteochondrosis should be differentiated from apophysitis because the etiologies and management strategies differ. Apophysitis is a traction injury to the cartilage and bony attachments of tendons in growing children. Osgood-Schlatter disease, Sever's disease, and Sinding-Larsen-Johansson syndrome are apophysitis disorders that affect the anterior tibial tubercle, posterior heel, and inferior patellar pole, respectively. Treatment of apophysitis involves stretching, activity modification, icing, and limited use of NSAIDs

Which one of the following would be most appropriate regarding screening for nonalcoholic fatty liver disease? A) No routine screening for any patients B) Routine screening for patients with obesity C) Routine screening for patients with hypothyroidism D) Routine screening for patients with polycystic ovary syndrome E) Routine screening for patients with type 2 diabetes

ANSWER: A Routine screening for nonalcoholic fatty liver disease (NAFLD) is not recommended for any patients, even those considered to be at high risk. The highest risk factor for NAFLD is obesity, and other risk factors include type 2 diabetes and metabolic syndrome. There is emerging evidence that HIV, hypothyroidism, polycystic ovary syndrome, obstructive sleep apnea, and genetic variation of the PNPLA3 gene are likely risk factors as well. NAFLD is usually discovered with the incidental finding of elevated liver enzymes. An AST/ALT ratio >1.5 is suspicious for excessive alcohol use, while a ratio of 0.8 is more likely due to NAFLD. If NAFLD is suspected, the first step is to obtain a detailed history, particularly for hepatotoxic medications and alcohol use. Drugs that can be problematic include chemotherapy medications, amiodarone, aspirin, corticosteroids, cocaine, NSAIDs, tetracyclines, and valproic acid. The next step is to evaluate for hepatitis B and C infection and measure ferritin and iron levels, lipids, and a fasting glucose level or hemoglobin A1c. If these are negative, then ultrasonography of the liver should be ordered.

Which one of the following medications for the treatment of type 2 diabetes has been associated with ketoacidosis? A) Dapagliflozin (Farxiga) B) Liraglutide (Victoza) C) Metformin D) Pioglitazone (Actos) E) Sitagliptin (Januvia)

ANSWER: A SGLT2 inhibitors such as dapagliflozin have increasingly been shown to be associated with diabetic ketoacidosis under certain circumstances. Liraglutide, metformin, pioglitazone, and sitagliptin are not associated with diabetic ketoacidosis.

An otherwise healthy 37-year-old female presents to your office with elbow pain 1 day after falling while skiing. She has pain and tenderness along the lateral aspect of the affected elbow and limited range of motion of the elbow and forearm. Radiographs confirm a nondisplaced fracture of the radial head. Which one of the following would be most likely to produce satisfactory outcomes? A) Immobilization for 3 days followed by range-of-motion exercises B) A long arm posterior splint for 6 weeks C) A long arm cast for 6 weeks D) Referral to an orthopedist for cast placement E) Referral to an orthopedist for surgical repair

ANSWER: A This patient has a nondisplaced radial head fracture. Current evidence supports a brief period of immobilization followed by early range-of-motion exercises to avoid decreased range of motion. This results in good outcomes in 85%-95% of patients. Immobilization for 6 weeks using either a long arm posterior splint or a long arm cast is not necessary. More advanced or displaced fractures may require a referral to an orthopedist for cast placement or operative repair, but nondisplaced radial head fractures can be managed by primary care physicians.

A 46-year-old female with known chronic kidney disease presents to discuss treatment of her recently diagnosed type 2 diabetes. She also has long-standing hypertension and her current medications include telmisartan/hydrochlorothiazide (Micardis HCT), amlodipine (Norvasc), and metformin. Her most recent estimated glomerular filtration rate is 52 mL/min/1.73 m2 and her hemoglobin A1c is 7.6%. Her serum electrolytes are all within the normal range. You refer her to a diabetes educator for counseling on self-management and lifestyle modification. Which one of the following is the recommended additional pharmacologic treatment for this patient? A) Empagliflozin (Jardiance) B) Glipizide (Glucotrol) C) Insulin glargine (Lantus) D) Pioglitazone (Actos) E) Sitagliptin (Januvia)

ANSWER: A The American Diabetes Association and the Kidney Disease: Improving Global Outcomes group recommend combination treatment with metformin and an SGLT2 inhibitor for patients with type 2 diabetes and chronic kidney disease (CKD) with an estimated glomerular filtration rate >30 mL/min/1.73 m2 . Metformin is first-line medical therapy for the majority of patients with type 2 diabetes. The addition of an SGLT2 inhibitor limits the progression of kidney disease and improves cardiac outcomes (SOR A). They should be considered first-line treatment along with metformin regardless of the baseline or target hemoglobin A1c. Sulfonylureas and thiazolidinediones would not be used as first-line therapy in this scenario. Their use is now recommended secondarily only when cost is a major issue. Insulin is not indicated in this patient as oral hypoglycemics are first-line therapy unless the hemoglobin A1c is >10% or the patient is experiencing persistent symptoms of hyperglycemia. DPP-4 inhibitors such as sitagliptin are second-line therapy to help patients reach glycemic targets. They are not used as first-line treatment with metformin or as monotherapy. The significant weight loss benefits of GLP-1 receptor agonists and SGLT2 inhibitors make them preferred choices.

A 50-year-old male comes to your office for a health maintenance visit. He does not have any symptoms today. You last saw him 4 years ago and at that time his BMI was 24 kg/m2 , his blood pressure was 124/70 mm Hg, and a lipid panel was normal. Today he tells you that he gained some weight and started smoking due to a stressful job. He is concerned about his risk for cardiovascular disease. Which one of the following would be the most appropriate screening test for this patient at this time? A) Hypertension screening B) An ankle-brachial index C) A coronary artery calcium score D) An EKG E) Abdominal duplex ultrasonography

ANSWER: A The U.S. Preventive Services Task Force (USPSTF) recommends screening for hypertension with office blood pressure measurement in adults >18 years of age (A recommendation). Adults >40 years of age and those >18 years of age with risk factors should receive annual hypertension screening. Less frequent screening is recommended for adults 18-39 years of age without risk factors. The USPSTF found insufficient evidence to recommend an ankle-brachial index (I recommendation) or a coronary artery calcium score (I recommendation) for cardiovascular screening, even in patients with traditional risk factors such as male sex, older age, current smoking, and overweight/obesity status. Cardiovascular screening with an EKG is not recommended (D recommendation). The USPSTF did not find adequate evidence to suggest that adding a resting or exercise EKG helped guide treatment decisions to reduce cardiovascular events in asymptomatic adults. It identified some evidence that small to moderate harms may exist, such as unnecessary invasive procedures. The USPSTF recommends abdominal duplex ultrasonography (B recommendation) for men 65-75 years of age who have ever smoked, which is usually defined as 100 cigarettes or more in a lifetime. The only appropriate screening test for the patient in this scenario is blood pressure measurement.

A 62-year-old male presents with a 3-day history of left lower quadrant pain and a low-grade fever. Findings on CT are consistent with acute diverticulitis. The patient has a history of intolerance to metronidazole (Flagyl). If antibiotics are given, the preferred agent for this patient would be A) amoxicillin/clavulanate (Augmentin) B) azithromycin (Zithromax) C) cephalexin (Keflex) D) ciprofloxacin (Cipro) E) doxycycline

ANSWER: A The traditional approach to outpatient management of acute diverticulitis consists of clinical diagnosis (with or without imaging), antibiotics, and bowel rest. Two cohort studies found no difference in the effectiveness of outpatient treatment of diverticulitis with amoxicillin/clavulanate or with metronidazole + fluoroquinolone. Azithromycin is more appropriate for Campylobacter or Escherichia coli infections that cause lower gastrointestinal bleeding. Cephalexin is not an appropriate treatment, and ciprofloxacin monotherapy will not provide adequate coverage. Doxycycline is a treatment for watery diarrhea caused by Vibrio cholerae and Yersinia infections

A 75-year-old male sees you for evaluation of a unilateral resting tremor of his right hand. The tremor resolves if he is touched on the hand by someone. His wife notes that he seems to drag his feet now, but he has no history of falls. Which one of the following has been shown to delay progression of his disease? A) No currently available pharmacologic agents B) Amantadine C) Carbidopa/levodopa (Sinemet) D) Rasagiline (Azilect) E) Ropinirole

ANSWER: A There are no currently available medications that have been shown to delay progression of Parkinson's disease. However, guidelines recommend initiating the treatment of motor symptoms when they begin to affect the functions of daily life or decrease the quality of life. The first-line treatment for motor symptoms is carbidopa/levodopa due to its effectiveness for tremors, rigidity, and bradykinesia. It is a myth that delaying the use of levodopa will prevent a lack of efficacy later in the course of the illness, as what appears to be a lack of efficacy actually represents progression of the disease. Amantadine can be used for patients under 65 years of age who are only experiencing tremors. Monoamine oxidase inhibitors such as rasagiline and non-ergot dopamine agonists such as ropinirole are not as effective as carbidopa/levodopa for motor symptoms, but they do not cause the dyskinesias and motor fluctuations seen with levodopa. Monoamine oxidase inhibitors are considered first-line therapy for patients under age 65 with mild motor symptoms.

A 6-year-old male presents with a cough and a fever. His vital signs include a temperature of 38.0°C (100.4°F), a heart rate of 120 beats/min, a respiratory rate of 22/min, a blood pressure of 90/52 mm Hg, and an oxygen saturation of 94% on room air. An examination reveals a child who is occasionally coughing but is in no acute distress. The mucous membranes are moist and capillary refill time is 2 seconds. There are no retractions, nasal flaring, use of accessory muscles of respiration, or any other signs of respiratory distress. The heart is mildly tachycardic without murmur and the lungs have rhonchi in the left lung base. Rapid tests for influenza and COVID-19 are negative. You suspect typical community-acquired pneumonia. Which one of the following would be the most appropriate next step in the management of this patient's condition? A) Oral amoxicillin B) Oral doxycycline C) Intramuscular ceftriaxone D) Withholding antibiotics until the diagnosis is confirmed with radiography E) Hospitalization for intravenous antibiotics

ANSWER: A This child presents with symptoms and signs consistent with community-acquired pneumonia. Since the child is well hydrated and in no distress, outpatient therapy with oral antibiotics is most appropriate. The preferred first-line antibiotic is amoxicillin. In patients older than 7 years of age, doxycycline is an alternative option when an atypical bacterial cause is presumed likely. Intramuscular ceftriaxone, withholding antibiotics until the diagnosis is confirmed with radiography, and hospitalization for intravenous antibiotics would not be appropriate in this case.

40-year-old female presents with a low back strain that occurred when moving furniture over the weekend. She rates her pain as mild to moderate. She initially tried acetaminophen, 1 g every 8 hours, and when this was ineffective, she switched to ibuprofen, 600 mg every 6-8 hours without relief. She is in good health otherwise and does not take any other medications. Which one of the following would be the most appropriate pharmacologic therapy to recommend next? A) A combination of acetaminophen, 500 mg, and ibuprofen, 600 mg, every 8 hours B) CBD oil applied to the low back up to four times daily C) Diclofenac topical (Voltaren Arthritis Pain) applied to the low back every 6 hours D) Hydrocodone/acetaminophen (Norco), 5/325 mg every 6 hours E) Oxycodone (Roxicodone), 5 mg, every 4-6 hours

ANSWER: A This patient has acute low back pain of moderate severity. If acetaminophen and NSAIDs are ineffective when used alone, the most appropriate next step is a combination of both medications. Acetaminophen/NSAID combinations have been shown to be more effective for acute pain than either agent alone. CBD oil does not have a specific indication for acute pain, and low-quality studies show mixed results. Diclofenac topical gel is an appropriate treatment option for acute, non-low back musculoskeletal pain. This patient describes her pain as mild to moderate in severity, so other options should be tried before prescribing opioids such as hydrocodone/acetaminophen or oxycodone.

A 13-year-old male presents with a 3-week history of pain in the anterior right knee that is worse when descending stairs and jumping. He is active in sports but has no recent history of injury. On examination you note tenderness and swelling over the tibial tuberosity, but no redness or warmth. Which one of the following would be most appropriate before initiating treatment? A) No imaging B) Plain film radiographs C) Ultrasonography D) MRI E) A bone scan

ANSWER: A This patient has apophysitis at the insertion of the patellar tendon at the tibial tubercle. The clinical diagnosis of Osgood-Schlatter disease is based on history and examination, and imaging is not needed initially. If this patient's symptoms persist despite treatment, or if there are atypical features or a history of trauma, imaging would be appropriate. Plain film radiography or ultrasonography would be a good first step, with ultrasonography offering the advantage of no radiation exposure. MRI and a bone scan would not be indicated.

A 75-year-old male smoker presents with intermittent, throbbing pain in both lower extremities that is relieved with rest. An ankle-brachial index is 0.7. All of the following would be appropriate for this patient EXCEPT A) apixaban (Eliquis), 5 mg twice daily B) enteric-coated aspirin, 81 mg daily C) rosuvastatin (Crestor), 10 mg daily D) a structured exercise program E) tobacco cessation

ANSWER: A This patient has confirmed peripheral artery disease (PAD) with an abnormal ankle-brachial index. Guideline-directed therapy for PAD includes low-dose aspirin, moderate- to high-intensity statin therapy, an ACE inhibitor or angiotensin receptor blocker, a structured exercise program, and smoking cessation. Apixaban is a novel oral anticoagulant that is used for stroke prevention in nonvalvular atrial fibrillation as well as treatment of deep vein thrombosis and pulmonary embolism. Apixaban is not used for the treatment of PAD.

A 67-year-old male presents to your office with a 1-month history of fever with edema and erythema of his right foot. His medical history is significant for peripheral artery disease and poorly controlled type 2 diabetes with diabetic neuropathy. Significant vital signs include a temperature of 38.6°C (101.5°F), a blood pressure of 155/90 mm Hg, and a pulse rate of 85 beats/min. A physical examination is most notable for a draining ulcer on the ball of his right foot. The edema and erythema are limited to his right foot and he has no calf tenderness. Dorsalis pedis and posterior tibial artery pulses are present but diminished at the right ankle. Sensation to monofilament testing of his right foot is diffusely diminished, which is consistent with his baseline. The best initial imaging test of the foot ulcer would be: A) radiography B) ultrasonography C) CT D) MRI E) technetium 99m bone scintigraphy

ANSWER: A This patient has osteomyelitis until proven otherwise. A radiograph of his right foot is the best initial test to look for evidence of this diagnosis. In most treatment settings, radiography is much easier to obtain than ultrasonography, CT, MRI, or technetium 99m bone scintigraphy. In addition, it is generally considerably less expensive than the other options listed. A radiograph also allows the physician to rule out other bony pathologies. MRI is useful if the radiograph is inconclusive, and is more helpful than radiography for determining bony versus soft-tissue infection. CT may be used in cases where MRI is contraindicated. Ultrasonography is not useful for evaluating bony lesions. Bone scintigraphy has low sensitivity, particularly in the setting of recent trauma or surgery.

40-year-old female presents with several pruritic, thickened, scaly areas on her lower back, knees, and elbows. She says that when she tries to remove the scales they often bleed. Which one of the following would be the most appropriate pharmacologic therapy for this patient? A) Clobetasol propionate 0.05% lotion (Clobex) B) Selenium sulfide 2.5% lotion C) Permethrin cream (Nix) D) Terbinafine cream 1% E) Loratadine (Claritin), 10 mg daily

ANSWER: A This patient has psoriasis that is characterized by plaques on her extensor extremities and limited bleeding with removal of the scales (Auspitz sign). First-line treatment for localized plaques is topical corticosteroid therapy, such as clobetasol propionate lotion. Antifungals such as selenium sulfide lotion and terbinafine cream are used to treat dermatophytosis infections including tinea pedis and tinea versicolor. Permethrin cream is indicated for treatment of scabies and lice. Loratadine, an oral antihistamine, is used to treat urticaria

46-year-old female presents with a 2-week history of polyuria. She has not had any discomfort with urination or visible change in her urine. Her past medical history includes hysterectomy for fibroids. Further history reveals no concerning environmental exposures other than a 5-pack-year history of smoking in her twenties. A physical examination, including a pelvic examination, is unremarkable. A urine dipstick reveals only 1+ RBCs. A microscopic urinalysis is negative with the exception of 7 RBCs/hpf, and a urine culture is negative. Which one of the following would you recommend at this point? A) Repeat urinalysis in 6 months B) Renal ultrasonography only C) Urine cytology and renal ultrasonography D) Urine cytology and CT urography E) Cystoscopy and CT urography

ANSWER: A This patient presents with microscopic hematuria without a clear cause. Past guidelines from the American Urologic Association recommended cystoscopy and CT urography for all patients over the age of 35 with microscopic hematuria. However, current guidelines recommend risk stratification and emphasize the use of imaging that has less radiation exposure. Because she is female and younger than 50 years of age, this patient is at low risk of urologic malignancy. Her smoking history and RBC count of <10 RBCs/hpf are also considered low risk. The guidelines recommend the option of repeating urinalysis in 6 months before proceeding with imaging or procedures, given her low risk (SOR C). Renal ultrasonography may be used in intermediate-risk patients, while CT urography is reserved for high-risk patients. Urine cytology is not a recommended test in this setting.

A healthy 80-year-old female sees you for a routine visit. She is active and follows a healthy diet. She is enthusiastic about vitamin supplements and asks you regularly about their benefits. Her laboratory chemistry profile demonstrates a persistent calcium level elevation at 10.9 mg/dL (N 8.5-10.2). You review her prescription medications and do not find any associated with hypercalcemia. You also review her calcium and vitamin D intake. Because you know about her tendency to take supplements, you consider other vitamins that may contribute to the hypercalcemia. Excessive intake of which one of the following would be the most likely explanation for these findings? A) Vitamin A B) Vitamin B1 C) Vitamin C D) Vitamin E E) Vitamin K

ANSWER: A Vitamin A intoxication can cause hypercalcemia. This includes analogs of vitamin A such as those used to treat acne. The excessive intake of vitamin A is associated with multisystem effects that can include bone resorption and hypercalcemia. Sources of preformed vitamin A include supplements as well as animal sources such as liver, fish liver oil, dairy, and eggs. Vitamin A toxicity should be considered in unexplained cases of parathyroid hormone-independent hypercalcemia. Vitamins B1, C, E, and K are not associated with hypercalcemia.

An 18-month-old male is brought to your office by his parents for a well child check. The child was born at 28 weeks gestation and had a month-long NICU stay but has remained healthy and out of the hospital since that time. He is up to date on vaccines and his growth and development are appropriate. He was on omeprazole (Prilosec) for GERD but the parents have recently stopped the medication and he is doing well. He received palivizumab (Synagis) monthly last year during respiratory syncytial virus (RSV) season and never developed a respiratory infection. His parents are hoping that he can receive palivizumab again this year to prevent complications if he develops RSV. He recently started attending day care and they are worried about his exposure risk. Which one of the following would you recommend this year for chemoprophylaxis against RSV in this patient? A) No chemoprophylaxis B) A single dose of palivizumab only if RSV exposure is confirmed C) A single dose of palivizumab prior to RSV season D) Monthly administration of palivizumab during RSV season

ANSWER: A With the increasing shortage of pediatric providers, especially in rural areas, family physicians need to be comfortable managing the care of premature infants. Palivizumab is recommended for all infants born before 29 weeks gestational age who are less than 1 year of age at the beginning of respiratory syncytial virus season, or for those born at less than 32 weeks gestational age who develop chronic lung disease of prematurity. After 1 year of age, palivizumab is only recommended for infants with chronic lung disease of prematurity who continue to require medical intervention for their lung disease. Therefore, this child should not receive palivizumab.

A 62-year-old male was recently diagnosed with adhesive capsulitis. Which one of the following is associated with a higher risk of developing adhesive capsulitis compared with the general population? A) Addison's disease B) Diabetes mellitus C) Hyperparathyroidism D) Hypertension E) Rheumatoid arthritis

ANSWER: B According to a 2016 meta-analysis, patients with diabetes mellitus were five times more likely than the control group to have adhesive capsulitis. The same study also found the prevalence of diabetes in patients with adhesive capsulitis to be about 30%. Because of this high prevalence, screening for diabetes with a fasting glucose level or hemoglobin A1c is recommended in patients with adhesive capsulitis who have not previously been diagnosed with diabetes. There is no evidence to support screening for Addison's disease or rheumatoid arthritis in patients with adhesive capsulitis (SOR C). Hyperparathyroidism and hypertension are not associated with adhesive capsulitis.

Which one of the following statements regarding hormone therapy for transgender patients is true? A) Hormone therapy to facilitate development of secondary sex characteristics is generally reversible B) Patients who receive hormone therapy generally report improved quality of life, higher self-esteem, and decreased anxiety C) Masculinizing hormone therapy is associated with reduced muscle mass and fat redistribution D) Patients receiving feminizing hormone therapy are at increased risk for erythrocytosis

ANSWER: B Hormone therapy is not required for all transgender patients, but those who receive treatment generally report improved quality of life, higher self-esteem, and decreased anxiety. Feminizing and masculinizing hormone therapies, including the use of estrogen and/or androgen therapies such as testosterone, are partially irreversible. Thus, it is important to make a reasonable, educated decision and use informed consent prior to treatment. Patients who receive masculinizing therapy are at increased risk for erythrocytosis and those who receive feminizing hormone therapy often experience reduced muscle mass and fat redistribution.

Which one of the following is the recommended duration of anticoagulation for a first episode of a provoked proximal deep vein thrombosis of the leg? A) 6 weeks B) 3 months C) 9 months D) 1 year E) Lifelong

ANSWER: B The 2016 CHEST guideline and expert panel report gives a strong recommendation (Grade 1B) to support 3 months of anticoagulation after a first episode of a provoked proximal deep vein thrombosis (DVT) of the leg. Treatment for 3 months appears to be superior to shorter treatment (6 weeks) with regard to recurrence of DVT. Treatment regimens >6 months for an unprovoked DVT may slightly reduce the rate of recurrence, but this is offset by an increased rate of bleeding and mortality

A 36-year-old female presents for evaluation of elevated blood pressure. She is asymptomatic and does not take any medications. On examination her blood pressure is 160/96 mm Hg and her BMI is 26 kg/m2. Fasting laboratory studies include the following: Sodium 142. Potassium 3.0. Creatinine 0.76. Glucose 97. Which one of the following additional laboratory evaluations should be performed to assess her blood pressure? A) A 24-hour urine collection for 5-hydroxyindoleacetic acid (5-HIAA) B) A serum aldosterone/renin ratio C) A serum cortisol level D) A serum cystatin C level

ANSWER: B Primary hyperaldosteronism should be suspected as a cause for hypertension if a patient has a spontaneously low potassium level or persistent hypertension despite the use of three or more antihypertensive medications, including a diuretic. This can be evaluated by checking a serum renin activity level and a serum aldosterone concentration and determining the aldosterone/renin ratio. Primary hyperaldosteronism typically presents with a very low serum renin activity level and an elevated serum aldosterone concentration. A 24-hour urine collection for 5-hydroxyindoleacetic acid (5-HIAA) would be used to evaluate for a neuroendocrine tumor, which can present as chronic flushing and diarrhea. Cortisol levels can be checked if Cushing syndrome is suspected. Hypertension can be present in Cushing syndrome, but it is typically associated with other signs such as obesity and an elevated blood glucose level due to insulin resistance. Cystatin C is a marker of renal function and measurement would not be indicated given this patient's normal creatinine level.

A 12-year-old female is brought to your office for a routine well child examination. The U.S. Preventive Services Task Force recommends screening this patient for which one of the following? A) Anemia B) Depression C) Diabetes mellitus D) Dyslipidemia E) HIV

ANSWER: B The U.S. Preventive Services Task Force (USPSTF) recommends screening for depression in adolescents and adults starting at age 12. The USPSTF states that the current evidence is insufficient to assess the balance of benefits and harms of screening for iron deficiency anemia in children 6-24 months of age (I recommendation) and does not offer recommendations regarding other age groups. There are no USPSTF recommendations regarding universal screening for diabetes mellitus in children or adolescents. The American Academy of Pediatrics now recommends screening for dyslipidemia in children once between 9 and 11 years of age, but the USPSTF concludes that the current evidence is insufficient to assess the balance of benefits and harms of screening for lipid disorders in children and adolescents <20 years of age (I recommendation). HIV screening is recommended in adolescents and adults 15-65 years of age (A recommendation).

A 67-year-old male with a history of hypertension comes to your clinic for a follow-up visit. He has had two myocardial infarctions in the past 5 years and has undergone stent placement. He is currently asymptomatic. His vital signs are stable and his blood pressure is well controlled. Laboratory studies reveal a normal hemoglobin A1c and lipid profile. In addition to high-dose statin therapy, his current medication regimen includes the following: Aspirin Carvedilol (Coreg) Chlorthalidone Clopidogrel (Plavix) Lisinopril (Zestril) Adding which one of the following would help to provide secondary prevention of cardiovascular events in this patient? A) Azithromycin (Zithromax) B) Colchicine (Colcrys) C) DHA D) Niacin E) Omega-3 supplements

ANSWER: B The central role of inflammation in the progression of coronary disease is well recognized and the use of an anti-inflammatory medication may improve outcomes in these patients. The low-dose colchicine (LoDoCo2) trial evaluated colchicine, 0.5 mg daily, versus placebo in patients with chronic coronary artery disease and found a 30% risk reduction in cardiovascular deaths, spontaneous myocardial infarctions, ischemic stroke, and ischemia-driven revascularization. It did not find any observable difference with regard to new-onset atrial fibrillation, deep vein thrombosis, diabetes mellitus, or pulmonary embolism. Of note, the trial excluded individuals with heart failure or renal impairment. At one time, azithromycin had shown some evidence in the secondary prevention of cardiovascular disease, but subsequent trials did not show the same benefit. Studies of fish oil capsules that contain marine omega-3 fatty acid supplements mixed with EPA/DHA formulations have failed to show cardiovascular benefit in patients with known cardiovascular disease. Similarly, niacin does not reduce overall mortality, cardiovascular mortality, or noncardiovascular mortality. The benefits of niacin therapy in the prevention of cardiovascular disease events are not well proven.

A 56-year-old male presents to your office with a new onset of nonvalvular atrial fibrillation. His CHA2DS2-VASc score is 3 based on his previous history of hypertension, diabetes mellitus, and heart failure. He has no major risk factors for bleeding. Which one of the following would be recommended for the prevention of ischemic stroke secondary to atrial fibrillation in this patient? A) No antithrombotic therapy B) Apixaban (Eliquis), 5 mg twice daily C) Aspirin only, 81 mg daily D) Aspirin only, 325 mg daily E) Aspirin, 81 mg daily, plus full-dose warfarin, with a target INR of 2..0-3.0

ANSWER: B The decision regarding antithrombotic therapy in atrial fibrillation is a careful risk assessment balancing the reduction in the risk of ischemic stroke against the risk of major bleeding associated with anticoagulants and antiplatelets. The CHA2DS2-VASc tool is widely used to help weigh these benefits versus potential harms. Due to the overall benefit of stroke reduction, anticoagulation with either a direct oral anticoagulant (DOAC) such as apixaban or a vitamin K antagonist such as warfarin is recommended in patients with atrial fibrillation who have a CHA2DS2-VASc score >2. Aspirin monotherapy is considered an ineffective antithrombotic strategy and inferior to a DOAC or warfarin for preventing thromboembolic events in patients with atrial fibrillation. Adding aspirin therapy to warfarin does not confer extra benefit and increases the risk of major bleeding.

An obese 40-year-old female with diabetes mellitus sees you for evaluation of painful, deep-seated nodules in both axillae. On examination you note nodules in the axillae with purulent drainage and associated scarring. This condition is associated with which one of the following? A) Amyotrophic lateral sclerosis B) Crohn's disease C) Dermatitis herpetiformis D) Systemic lupus erythematosus E) Trauma

ANSWER: B The patient has hidradenitis suppurativa, a chronic folliculitis affecting intertriginous areas in the axillae and the groin that may also occur around the anus and nipples. Treatment depends on severity and ranges from topical to systemic antibiotics. Hidradenitis suppurativa is associated with obesity, diabetes mellitus, Crohn's disease, arthritis and spondyloarthropathy, metabolic syndrome, polycystic ovary syndrome, pyoderma gangrenosum, and trisomy 21. There are three stages: - stage I is single or multiple abscesses without sinus tracts or scarring - stage II is abscess recurrence with sinus tracts and scarring and widely separated lesions - stage III is diffuse abscesses with interconnecting sinus tracts. Amyotrophic lateral sclerosis has no typical skin manifestation. Dermatitis herpetiformis is associated with celiac disease and has clusters of pruritic lesions. Systemic lupus erythematosus has cutaneous manifestations of a malar rash and may involve subcutaneous lesions without scarring. Hidradenitis suppurativa is not associated with trauma.

A 65-year-old male is discharged following placement of a drug-eluting stent in the left anterior descending artery. Which one of the following is NOT appropriate first-line therapy in this patient? A) Aspirin plus clopidogrel (Plavix) B) Diltiazem (Cardizem) C) Metoprolol D) Rosuvastatin (Crestor)

ANSWER: B Beta-Blockers are first-line therapy for antihypertensive therapy and antianginal therapy, whereas calcium channel blockers are second-line agents in patients who are unable to tolerate beta-blockers. Calcium channel blockers may also be added as additional therapy when hypertension and angina symptoms are not controlled with beta-blockers alone. Patients who have been treated with a drug-eluting stent require dual antiplatelet therapy for 6-12 months. All patients with coronary artery disease should be on high-dose statin therapy.

An ill-appearing 50-year-old male presents with malaise, nausea, anorexia, and lethargy. He has a recent diagnosis of a high-grade lymphoma and is undergoing aggressive chemotherapy. His last chemotherapy session was 2 days ago. An examination is nonspecific. Initial laboratory studies reveal a creatinine level of 2.1 mg/dL (N 0.6-1.2). His baseline creatinine level is 1.0 mg/dL. Which one of the following laboratory findings would be expected in this patient? A) Hypercalcemia B) Hyperuricemia C) Hypokalemia D) Hypophosphatemia E) Low LDH

ANSWER: B Tumor lysis syndrome is considered the most common oncologic emergency. It is caused by the rapid release of intracellular material from lysis of the malignant cells. The breakdown of nucleic acids releases large amounts of uric acid and leads to acute kidney failure, which limits clearance of potassium, phosphorus, and uric acid. This leads to hyperuricemia, secondary hypocalcemia, hyperkalemia, and hyperphosphatemia. It can result in acute renal failure, arrhythmia, seizure, and sudden death. While tumor lysis syndrome has been reported with many cancer types, it is more common with acute leukemia and high-grade lymphomas. Patients with this condition generally present within 7 days of cancer treatment, including chemotherapy, radiation, or biologic therapies. It can also occur spontaneously. An LDH elevation related to a high cell turnover rate prior to cancer treatment may indicate an increased risk of tumor lysis syndrome. Hypercalcemia, hypokalemia, hypophosphatemia, and low LDH would not be expected laboratory findings in patients with tumor lysis syndrome.

You are evaluating a couple for infertility. The semen analysis demonstrates oligozoospermia on two separate samples. The history and physical examination of the male partner are otherwise unremarkable. Which one of the following would be the most appropriate next step in the evaluation of his oligozoospermia? A) A CBC with differential and a basic metabolic panel B) FSH and early morning total testosterone levels C) Antisperm antibody testing D) Scrotal ultrasonography E) Referral for a testicular biopsy

ANSWER: B A semen analysis is the first step in the evaluation of male infertility. In males with oligozoospermia (especially if the sperm count is <10 million/mL), the American Urological Association recommends an endocrine evaluation with an FSH level and early morning total testosterone levels. The results of that testing can dictate next steps. A CBC and a basic metabolic panel have no role in the evaluation of male infertility. Antisperm antibody testing is rarely recommended and should only be considered in consultation with a fertility specialist. Scrotal ultrasonography is not recommended in individuals with a normal physical examination and should only be performed in individuals with palpable varicoceles on physical examination. A testicular biopsy is not usually required to help differentiate between obstructive and nonobstructive azoospermia.

A 17-year-old female has a positive urine drug screen for cannabis. She does not use marijuana but recently attended a party where others were smoking it. How long after passive exposure to cannabis smoke can an otherwise cannabis-naïve person test positive for it on a drug screen? A) Never B) Up to 24 hours C) 3 days D) 1 week E) 1 month

ANSWER: B A urine drug screen may be positive for cannabis for up to 24 hours after exposure to secondhand cannabis smoke in an enclosed space. The urine drug screen can be positive for 4-5 days after a single use of cannabis and for a month after cessation in someone who uses it daily.

A 61-year-old male is found to have a 2-cm right adrenal incidentaloma on CT. He has no history of hypertension, electrolyte abnormalities, headaches, flushing, or sweating. Which one of the following studies should be performed in patients found to have an adrenal incidentaloma? A) An ACTH stimulation test B) A dexamethasone suppression test C) Paired serum aldosterone and plasma renin activity D) Serum or urine metanephrines E) A PET scan

ANSWER: B Adrenal incidentalomas usually do not produce overt hormone excess, but mild autonomous cortisol secretion (MACS) is present in up to 30%-50% of cases. This mild secretion of cortisol may predispose patients to metabolic syndrome, osteoporosis, and cardiovascular events. MACS can be ruled out with an overnight 1-mg dexamethasone suppression test. The remainder of the evaluation can be based on CT findings and clinical symptoms. An ACTH stimulation test is used to evaluate for adrenal insufficiency, which is not caused by an adenoma. If no hypertension or hypokalemia are present, the serum aldosterone and plasma renin activity paired values may not be necessary. Testing for metanephrines is indicated only if pheochromocytoma is suspected. If no clinical symptoms of pheochromocytoma (such as hypertension, sweating, or headaches) are present and the lesions are <10 Hounsfield units on CT, pheochromocytoma and malignancy are very unlikely. A PET scan would not be indicated in this scenario.

A 44-year-old female presents to your office reporting that she hurts all over. After performing a thorough history and physical examination and appropriate laboratory studies you diagnose fibromyalgia. You explain to the patient that the initial treatment recommendation with the most proven efficacy is: A) acupuncture B) aerobic exercise C) amitriptyline D) duloxetine (Cymbalta) E) tramadol

ANSWER: B Aerobic exercise, a balanced diet, good sleep hygiene, and weight reduction are appropriate strategies for the management of fibromyalgia, and treatment goals should be focused on improving function and quality of life, along with managing symptoms. According to the 2017 European League Against Rheumatism, exercise is the strongest and most critical treatment for fibromyalgia. Not only does it lessen fibromyalgia symptoms, but it can also help with coexisting conditions including sleep disorders, depression, and anxiety. While some studies show improvement in symptoms with acupuncture, most evidence is low to moderate in quality. A Cochrane review found that acupuncture was superior to no treatment at all, but not superior to sham acupuncture. Pharmacologic treatments have shown only modest benefits and are often accompanied by adverse effects, so they are best used in conjunction with nonpharmacologic therapies.

A 40-year-old female presents to your office with jaundice and dark urine. Testing for acute hepatitis A is positive. Her 12-year-old twin boys, who are in good health, have not been vaccinated against hepatitis A or B and are currently asymptomatic. Which one of the following would be the most appropriate postexposure prophylaxis for these children? A) No prophylaxis B) Hepatitis A vaccine only C) Immune globulin only D) Hepatitis A vaccine and immune globulin

ANSWER: B All unvaccinated household contacts and sexual contacts should receive postexposure prophylaxis following significant exposure to hepatitis A within the previous 2 weeks. Healthy individuals 12 months to 40 years of age should receive the hepatitis A vaccine as prophylaxis. Infants younger than 12 months of age should receive immune globulin as postexposure prophylaxis. Individuals >40 years of age, as well as immunocompromised patients, should receive both hepatitis A vaccine and immune globulin.

You evaluate a patient with fatigue, anorexia, and nausea for adrenal insufficiency. A morning cortisol level is low. Which one of the following would be the most appropriate confirmatory test? A) A 21-hydroxylase antibody level B) An ACTH stimulation test C) A low-dose dexamethasone suppression test D) A high-dose dexamethasone suppression test E) CT of the adrenal glands

ANSWER: B An ACTH stimulation test is the most appropriate confirmatory test for suspected adrenal insufficiency. If a cortisol level drawn 1 or 2 hours after administration of ACTH is inappropriately low, adrenal insufficiency is confirmed, and further evaluation is indicated to determine the etiology. A 21-hydroxylase antibody level may indicate that adrenal insufficiency has an autoimmune cause but does not confirm the presence of adrenal insufficiency. Dexamethasone suppression tests, in which morning cortisol is tested after administration of dexamethasone the previous evening, are used in the evaluation of suspected Cushing syndrome. Inappropriately elevated cortisol levels are indicative of Cushing syndrome. Imaging might be useful in determining the cause of adrenal gland dysfunction once adrenal insufficiency is confirmed.

A 25-year-old primigravida presents at 28 weeks gestation for a routine prenatal visit. She is undecided about breastfeeding versus bottle feeding and asks if breastfeeding provides any benefits for her own health. You advise her that breastfeeding would decrease her risk of later developing A) colon cancer B) diabetes mellitus C) lung cancer D) osteoarthritis E) recurrent respiratory infections

ANSWER: B Breastfeeding provides many health benefits to both the mother and the infant. Maternal benefits include a decreased risk of developing cardiometabolic disease, including diabetes mellitus, hypertension, and cardiovascular disease; a decreased risk of breast cancer and ovarian cancer; and a decreased risk of postpartum depression. A link has not been established between breastfeeding and a reduced risk of developing colon cancer or lung cancer or osteoarthritis later in life. While breastfeeding may reduce the infant's risk of respiratory disease, this is not an expected benefit for the mother.

A 72-year-old male presents for follow-up of a recent emergency department (ED) visit for chest pain that was diagnosed as costochondritis. He has a history of hypertension and chronic atrial fibrillation. A CBC in the ED was significant for a hemoglobin level of 11.1 g/dL (N 13.5-17.5) and a mean corpuscular volume of 104 μm 3 (N 80-100). Follow-up laboratory studies showed the following: Peripheral smear: normal other than macrocytosis Reticulocyte index: 1.7% (N 0.5-2.5) Vitamin B12: 512 pg/mL (N 190-950) Folate: 12 pg/mL (N 2.7-17) Which one of the following is the most likely cause of this patient's anemia? A) Anemia of chronic disease B) Chronic alcohol use C) Gastrointestinal bleeding D) Hemolysis E) Myelodysplastic syndrome

ANSWER: B Chronic alcohol use is one of the most common causes of macrocytic anemia. This patient's normal peripheral smear and reticulocyte index <2% in the setting of anemia suggest decreased red blood cell production, which may be related to alcohol use or nutritional deficiency. Vitamin B12 and folate levels in the normal range rule out deficiencies in these vitamins, though patients with low-normal vitamin B12 and folate levels may warrant testing of homocysteine and methylmalonic acid levels. Anemia of chronic disease is typically characterized by a normocytic or microcytic anemia and a low reticulocyte count. Blood loss and hemolytic anemia are typically associated with a reticulocyte index >2%. Myelodysplastic syndrome is not suggested by the peripheral smear and is less common than anemia related to alcohol use disorder.

Increasing greenhouse gas concentrations are resulting in multiple climate changes that can adversely affect the health of patients. Which one of the following examples describes a direct effect of climate change? A) Weather-related disasters have lessened, leading to an improvement in overall mental health B) Shorter and warmer winters allow insect vectors to spread into new areas, increasing the rate of multiple infectious diseases C) Increasing global temperatures have decreased the production of highly allergenic pollens such as ragweed D) Increasing consumption of plant-based foods has resulted in increased air pollution, further exacerbating many cardiopulmonary conditions

ANSWER: B Climate change is responsible for multiple negative impacts on patient health, including furthering the spread of multiple infectious diseases, a decline in mental health due to weather-related natural disasters, an increased risk of allergies, and increased exacerbation of cardiopulmonary conditions. Following a plant-based diet, transitioning to more active modes of transportation, and working within the health care system to help decrease greenhouse emissions can positively impact climate change.

The front office staff reports that a patient has been difficult to deal with. Which one of the following is an effective strategy for managing a difficult patient encounter? A) Avoiding any mention of the patient's anger B) Listening with empathy and a nonjudgmental attitude C) Ignoring one's own internal emotional responses D) Limiting the encounter to discussion of a single problem E) Using a directive approach to the patient's care with a predetermined agenda

ANSWER: B Difficult patient encounters may arise from a wide variety of patient, situational, and even physician factors. The triggering of an emotional response from the staff or the physician is the common factor that defines a difficult encounter. Empathetic listening skills and a nonjudgmental attitude are helpful to facilitate effective communication. Acknowledging anger and ascertaining the patient's concerns can help to validate the patient's feelings and defuse the situation. Physicians should be aware of their own emotional response in order to navigate the situation successfully. While boundaries can be helpful, arbitrarily limiting the visit to a single problem is unlikely to meet the patient's needs effectively. A patient-centered approach to interviewing, rather than a directive approach, is also more likely to be successful.

You see a 15-year-old male for a well child check. He does not have any health concerns and his developmental and social histories are unremarkable. His vital signs include a height of 152 cm (60 in), a weight of 79 kg (174 lb), a blood pressure of 133/80 mm Hg, a heart rate of 85 beats/min, and a respiratory rate of 14/min. Which one of the following would be the most appropriate next step in management? A) Discussing healthy lifestyle changes, with follow-up at the next annual visit B) Repeating blood pressure measurements two more times during this visit C) Checking blood pressure in the upper and lower extremities and starting an angiotensin receptor blocker D) Screening for hyperlipidemia, diabetes mellitus, fatty liver, and kidney disease only E) Screening for hyperlipidemia, diabetes, fatty liver, and kidney disease, and referral for intensive weight management and dietary therapy

ANSWER: B Hypertension in children up to age 12 is defined by a blood pressure at the 95th percentile or higher based on age, height, and sex. Starting at age 13, hypertension can be defined in absolute numbers of 130/80 mm Hg or higher. Diagnosing hypertension in children and adolescents requires careful evaluation, and it is recommended to recheck the blood pressure twice during the same visit using auscultation and average the values to determine the final blood pressure. If this results in a persistent hypertensive blood pressure level, initial lifestyle modifications should be recommended, and blood pressure should be rechecked in 1-2 weeks. Ambulatory blood pressure monitoring should be considered at that time, particularly if the blood pressure is borderline. If blood pressure remains high, a targeted evaluation for secondary hypertension and an evaluation for hyperlipidemia, diabetes mellitus, and renal disease should be performed, as well as checking upper and lower extremity blood pressures to evaluate for possible coarctation of the aorta. Referral to intensive programming for weight management and diet therapy would be appropriate at that time, particularly in children with obesity. Medication should not be started for asymptomatic stage 1 hypertension unless lifestyle modifications are unsuccessful.

A 45-year-old male presents to your office accompanied by his sister. He tells you that 6 months ago he was laid off from a job where he had been employed for more than 20 years. He says that he declined a promotion 2 years ago because he was worried about working with a new team. He has been unable to secure a new job, stating that the interview process has been embarrassing and that he feels unfairly judged when his resume is critiqued. He reports feeling inadequate and says that he is more comfortable staying at home alone. His sister adds that he has been shy since he was a teenager, and now seems unwilling to change despite his dire financial situation. Depression and anxiety screenings are negative. He does not use alcohol or other substances. Which one of the following personality disorders is most consistent with this patient's presentation? A) Antisocial B) Avoidant C) Borderline D) Dependent E) Histrionic

ANSWER: B In order to make a diagnosis, personality disorders must meet specific criteria as outlined in the DSM-5. Other mental disorders, substance use or exposure, and medical conditions must also be excluded. This patient has avoidant personality disorder, which is characterized by social inhibition, fears of inadequacy, and hypersensitivity to criticism or rejection. It often presents in early adulthood. Persons with avoidant personality disorder may avoid new or unfamiliar situations, such as this patient who is unwilling to seek a new job. Persons with antisocial personality disorder exhibit a lack of respect for the rights of others, as well as deceitfulness, aggressiveness, and recklessness. Psychopathy and sociopathy are alternate terms. Borderline personality disorder is marked by instability in interpersonal relationships and self-image, impulsivity, reactivity of mood, and self-destructive behavior. Dependent personality disorder is described as an excessive need to be taken care of, intense fear of being alone, and extreme reliance on others for motivation and direction. Persons with histrionic personality disorder demonstrate excessive emotionality and attention-seeking behavior, often overestimating the closeness of interpersonal relationships and alienating others with hypersexual or hyperemotional reactions.

You suspect somatic symptom disorder in one of your patients. Supporting this diagnosis, the patient scores very high on the Somatic Symptom Scale-8 with "quite a bit" or "very much" back pain, chest pain, dizziness, low energy, headaches, stomach problems, and trouble sleeping. You have completed a thorough history and physical examination. In addition to referral for cognitive behavioral therapy, which one of the following would be the most appropriate next step in patient care? A) Frequent, as-needed office visits B) Frequent, regularly scheduled office visits C) Infrequent, as-needed office visits D) Infrequent, regularly scheduled office visits E) Regularly scheduled annual examinations and as-needed office visits

ANSWER: B In somatic symptom disorder, the primary feature is the patient's concern with physical symptoms that have manifested through psychological or emotional distress and have no other medical explanation. The family physician should offer frequent, regularly scheduled visits for these patients. The acronym CARE MD describes an approach to management of somatic symptom disorder: • Consultation with mental health professionals for cognitive behavioral therapy or mindfulness-based therapy (SOR B) • Assessment for other medical and psychiatric diseases • Regular short-interval follow-up visits to stress coping rather than cure and to reduce the overuse of health care visits such as the emergency department • Empathy demonstrated by listening to the patient while acknowledging and legitimizing the patient's symptoms • Medical-psychiatric interface emphasizing the mind-body connection • Doing no harm by reassuring the patient while limiting diagnostic testing and referrals to subspecialists

A 68-year-old female presents for an annual health maintenance visit. She says that she does not receive the influenza vaccine because she developed hives many years ago after eating eggs. Which one of the following would you recommend? A) Avoiding influenza vaccine B) Proceeding with in-office vaccination without premedication C) Diphenhydramine (Benadryl Allergy), 50 mg 4 hours prior to vaccination D) Prednisone, 20 mg 4 hours prior to vaccination E) Referral to an allergist for desensitization

ANSWER: B It is uncommon for people to experience severe reactions, including anaphylaxis, after influenza vaccination. This is true even for those with egg allergies, despite the fact that embryonic chicken eggs are used to grow most influenza vaccine viruses. Influenza vaccine is safe to administer regularly to those who have only had hives after exposure to eggs. If more serious allergic symptoms occur with egg exposure, such as respiratory distress or anaphylaxis, influenza vaccine should be administered in an inpatient or supervised outpatient setting. Premedication with diphenhydramine or prednisone is not recommended. Referral to an allergist for desensitization would not be recommended for this patient.

A 51-year-old patient asks about recommended lung cancer screenings. The U.S. Preventive Services Task Force recommends annual lung cancer screening with low-dose CT for individuals starting at what age and how many pack year smoking history? A) 45 with a 15-pack-year smoking history B) 50 with a 20-pack-year smoking history C) 55 with a 30-pack-year smoking history D) 60 with a 35-pack-year smoking history E) 65 with a 40-pack-year smoking history

ANSWER: B Lung cancer is the second most common cancer in both women and men, after breast cancer for women and prostate cancer for men. It is the leading cause of cancer deaths in the United States, making it important for primary care providers to screen for this disease process. The primary risk factor for lung cancer is tobacco smoking, which accounts for 90% of all lung cancer cases. Lung cancer has a relatively poor prognosis, but early-stage lung cancer is more amenable to treatment and has a better prognosis. Low-dose CT has a reasonable specificity and high sensitivity for lung cancer in patients at high risk. The eligibility criteria were recently updated by the U.S. Preventive Services Task Force (March 2021) due to evidence of mortality benefit, with a recommendation for screening to begin at age 50 for patients with a 20-pack-year smoking history who are current smokers or have quit within the past 15 years.

You see a 26-year-old male who was diagnosed with maturity-onset diabetes of the young at age 22. He has a BMI of 24 kg/m2 and his hemoglobin A1c is now 8.5%. Which one of the following would be most appropriate for this patient? A) A ketogenic diet B) Glipizide (Glucotrol) C) Metformin D) Short-acting sliding scale insulin with meals E) Basal insulin at bedtime

ANSWER: B Maturity-onset diabetes of the young (MODY) is a form of diabetes mellitus in nonobese young adults (under age 30) who have preserved pancreatic beta-cell function. Nearly 80% of patients with MODY are misdiagnosed as having type 1 or type 2 diabetes. These patients exhibit no signs of insulin resistance (metabolic syndrome, acanthosis nigricans, skin tags, androgenic alopecia), are not obese, have positive C-peptide levels, and have a strong family history of diabetes. MODY does not respond to metformin, but because beta-cell function is preserved, the hyperglycemia does respond to sulfonylureas. While exercise and a balanced diet of appropriate portions and low carbohydrates are also necessary in patients with MODY, a ketogenic diet is not specifically indicated. Insulin is required only during pregnancy.

A 70-year-old female tells you she is confused about recommendations regarding aspirin. She has heard through friends and news articles that new guidelines seem to be discouraging people from taking a daily aspirin due to the risk of bleeding, especially severe gastrointestinal bleeding. She has no history of bleeding but has decided to stop taking her aspirin, 81 mg daily. Her blood pressure is well controlled on her current antihypertensive regimen, and she also takes a daily statin. Her medical history includes a stroke a few years ago. Which one of the following would you recommend? A) No antithrombotic therapy B) Resuming aspirin, 81 mg daily C) Starting aspirin, 500 mg daily D) Starting apixaban (Eliquis), 2.5 mg daily E) Starting warfarin, with a target INR >3.0

ANSWER: B Most studies of aspirin for secondary prevention of cardiovascular disease involved prevention of recurrent strokes, and showed a reduction in recurrent strokes in patients taking 75-325 mg of aspirin daily. For this patient with a history of stroke, resuming aspirin at 81 mg daily would be clearly indicated. Multiple organizations have advised on the role of aspirin in the primary prevention of cardiovascular disease. In 2022 the U.S. Preventive Services Task Force released an update recommending against the initiation of low-dose aspirin for primary prevention of cardiovascular disease in adults >60 years of age as there is no net benefit. The American College of Cardiology/American Heart Association (ACC/AHA) came to a similar conclusion regarding primary prevention. Specifically, the ACC states that low-dose aspirin, 75-100 mg daily, should not be administered on a routine basis for the primary prevention of atherosclerotic cardiovascular disease in adults >70 years of age. There is no benefit in taking an aspirin dosage >325 mg daily. The apixaban dosage for stroke prophylaxis is 5 mg twice daily. When taking warfarin for stroke prophylaxis, the INR target is 2-3, not >3.

A registered nurse at the local hospital calls to notify you of a newborn admission. She reports that the neonate was born at 40 weeks gestation to a G2P2002 via vaginal delivery. The mother was noted to be group B Streptococcus positive, and delivery was complicated by a maternal fever of 38.3°C (101.0°F) and suspected intra-amniotic infection for which the mother received 6 hours of broad-spectrum antibiotics. Her membranes ruptured 6 hours before delivery. The nurse notes that the infant has normal vital signs and appears healthy. Which one of the following would be recommended for the newborn to reduce antibiotic administration in this case? A) Categorical risk factor assessment B) Use of the neonatal early-onset sepsis calculator C) A C-reactive protein level D) A CBC and differential E) Blood cultures

ANSWER: B Neonatal early-onset sepsis (EOS) has an incidence of 0.5 per 1000 live births according to the CDC, and group B Streptococcus (GBS) remains the most common cause. Risk factors for neonatal EOS include maternal GBS, prolonged rupture of membranes, intrauterine inflammation or infection, and the combination of inflammation and infection, commonly known as maternal chorioamnionitis, or triple I. Updated guidelines from the American Academy of Pediatrics (AAP) and the American College of Obstetricians and Gynecologists replaced traditional guidelines on prevention of neonatal EOS from the CDC. Current guidelines recommend either categorical risk factor assessment, use of the neonatal EOS calculator, or enhanced observation. However, categorical risk factor assessment, similar to 2002 and 2010 CDC guidelines, would result in blood cultures and administration of antibiotics to any newborn where there was a maternal intrapartum fever. The other two approaches, particularly use of the neonatal EOS calculator, have been demonstrated to decrease antibiotic administration. This calculator uses a multivariate approach to determining neonatal EOS risk, combining information from both the delivery and postpartum assessment of the newborn. Enhanced observation utilizes frequent clinical assessment and is thought to similarly reduce antibiotic administration. The AAP guidelines suggest C-reactive protein levels and CBCs have poor predictive value in identifying neonatal EOS and should not be used to guide management. Blood cultures are frequently obtained with co-administration of antibiotics and there is no data to suggest that blood cultures alone would reduce antibiotic administration.

A 67-year-old male with a history of diabetes mellitus, hypertension, and heart failure with reduced ejection fraction has developed stage 5 chronic kidney disease. Which one of the following would be the best option for treatment of his diabetes? A) Glimepiride (Amaryl) B) Insulin glargine (Lantus) C) Metformin D) Pioglitazone (Actos)

ANSWER: B Patients with end-stage renal disease and diabetes mellitus need careful monitoring of glucose because insulin requirements are difficult to predict and there is an increased risk of hypoglycemia in this setting. The optimal hemoglobin A1c has not been established but maintaining a value between 6% and 9% does decrease mortality. With close monitoring, insulin is preferred for most individuals. Sulfonylureas such as glimepiride and glyburide are associated with a high risk of hypoglycemia and should be avoided in these patients. Metformin should be avoided in those with a glomerular filtration rate <30 mL/min/1.73 m 2 . Pioglitazone should also be avoided in chronic kidney disease due to the risk of fluid retention and precipitating heart failure.

A 30-year-old gravida 3 para 2 sees you for prenatal care at 13 weeks gestation. During her previous pregnancies she became hypertensive and had bilateral leg edema and proteinuria. These conditions resolved after delivery. Her only current medication is a prenatal vitamin. In order to prevent this condition, which one of the following should be started today? A) No new medications B) Aspirin C) Fish oil D) Magnesium E) Vitamin C

ANSWER: B Preeclampsia is diagnosed when the blood pressure is >140/90 mm Hg on two separate occasions after 20 weeks gestation, accompanied by proteinuria (>300 mg protein in a 24-hour urine collection or 2+ protein on a dipstick). If there is no protein in the urine, new-onset hypertension and the presence of any of the following would meet the criteria for preeclampsia: thrombocytopenia, renal insufficiency, impaired liver function, pulmonary edema, or cerebral or visual symptoms. This patient had preeclampsia during her previous pregnancies, which puts her at high risk for preeclampsia during her current pregnancy. Aspirin, 81 mg daily, is recommended for high-risk pregnant patients to prevent preeclampsia. Prophylaxis should begin after 12 weeks gestation and continue until delivery. Fish oil, magnesium, and vitamin C are not beneficial in the prevention of preeclampsia.

A 27-year-old female with hypothyroidism presents to your clinic at 5 weeks gestation. Her current medications include levothyroxine (Synthroid), 100 μg daily, and a prenatal vitamin. She had a normal TSH level 3 months ago. She is very concerned about the negative effects of medication during pregnancy and asks if she should continue taking levothyroxine. Which one of the following would be most appropriate for this patient at this time? A) Continuing the current levothyroxine dosage and referring her to an endocrinologist B) Increasing the levothyroxine dosage by 30% by taking an extra dose twice weekly C) Checking her TSH level today and increasing the levothyroxine dosage by 12.5 μg daily if it is >2.5 μU/mL D) Discontinuing levothyroxine and checking her TSH and free T4 levels in 6 weeks

ANSWER: B Pregnant patients with hypothyroidism require increased dosages of levothyroxine as early as the first 4 weeks of pregnancy. General recommendations advise taking an extra dose 2 days per week for a total of 9 weekly doses, which is roughly a 30% increase. The TSH level should be monitored every 4 weeks during pregnancy, and the levothyroxine dosage should be titrated to the pregnancy-specific reference range, which is generally lower than the normal reference range. Referral to an endocrinologist to manage fluctuating levels should be considered. Untreated hypothyroidism can lead to adverse pregnancy outcomes including spontaneous abortion, preterm birth, preeclampsia, and placental abruption.

An 8-year-old male is brought to your office because of acute lower abdominal pain. He does not have constipation and has never had abdominal surgery. You suspect acute appendicitis. Which one of the following imaging modalities would be most appropriate to consider first? A) Plain radiography B) Ultrasonography C) CT without contrast D) CT with contrast E) MRI

ANSWER: B Ultrasonography is recommended as the initial imaging modality to evaluate acute abdominal pain in children. It avoids radiation exposure and is useful for detecting many causes of abdominal pain, including appendicitis. After ultrasonography, CT or MRI can be used if necessary to diagnose appendicitis. Abdominal radiography is helpful in patients with constipation, possible bowel obstruction, or a history of previous abdominal surgery. The American Academy of Pediatrics Choosing Wisely recommendation on the evaluation of abdominal pain states that CT is not always necessary. Similarly, the American College of Surgeons Choosing Wisely recommendation on the evaluation of suspected appendicitis in children says that CT should be avoided until after ultrasonography has been considered as an option

You are providing medical care during a national emergency crisis and are tasked with triaging and allocating limited critical care resources when demand is greater than supply. Which one of the following processes is consistent with ethically sound crisis standards of care? A) Making triage decisions based on social and community value B) Explaining triage decision practices and providing a process for appealing decisions C) Allocating limited resources on a first-come, first-served basis when unable to distinguish medical need among patients D) Continuing a patient's treatment without reassessment once resources have been allocated

ANSWER: B The American Medical Association Code of Medical Ethics offers helpful guidance for making difficult decisions, including triage and reassessment decisions, when health care resources are limited during crisis situations. Explaining triage decision policies and procedures and providing patients who are denied initial resources a process for appealing decisions is a recommended process and the most ethically sound option of those listed. It is also recommended to make triage decisions based on medical need rather than social worth, and to allocate limited resources first to prevent premature death and then to those with the greatest duration of benefit after recovery. When unable to distinguish need based on medical factors, a random process or lottery is recommended rather than a first-come, first-served process since patients with obstacles to care who cannot present first would be unfairly disadvantaged. Reassessment of whether continued treatment is likely to be beneficial should occur periodically. Providing palliative care when treatment has been withdrawn is a necessary ethical practice.

A 40-year-old female sees you to discuss a thyroid nodule that was noted on a report from carotid ultrasonography performed by an independent organization. The patient is asymptomatic. Her family history is negative for malignancy. On physical examination you confirm the presence of a thyroid nodule. The remainder of the examination, including the lymphatic system, is negative. A TSH level is normal. Which one of the following would be the most appropriate next step? A) Observation only B) Thyroid ultrasonography C) A radionucleotide thyroid uptake scan D) Fine-needle aspiration only E) Fine-needle aspiration with molecular testing

ANSWER: B The first step in the evaluation of a palpable thyroid nodule is to obtain a TSH level and perform thyroid ultrasonography (SOR C). If the TSH level is low, a radionucleotide thyroid uptake scan is the appropriate next step to assess for a hyperfunctioning nodule. If the TSH level is normal or high, next steps are determined by the size and characteristics of the thyroid nodule on ultrasonography. Fine-needle aspiration (FNA) may be indicated depending on the size and nodule characteristics. Molecular testing of FNA specimens is useful in order to guide management of thyroid nodules with indeterminate cytology. Before molecular testing is performed, patients should be counseled about the potential benefits and limitations of the test (SOR C). Observation only would not be appropriate in this scenario.

An 11-year-old female is brought to your office by her parent who is concerned that the child's spine might be curved. The most appropriate evaluation for scoliosis at this point is A) comparing the length from the pelvic brim to the pelvic floor on the left and the right B) scoliometer measurement with the patient bent over to 90° C) scoliometer measurement with the patient upright and arms to her side D) determination of the Cobb angle with the patient bent over to 90° E) determination of the Cobb angle with the patient upright and arms to her side

ANSWER: B The forward bend test, combined with a scoliometer measurement, is the most appropriate initial test when evaluating for scoliosis. A scoliometer should be used with the patient's spine parallel to the floor (bent over to approximately 90°), with the arms hanging down, palms together, and feet pointing forward. If 5°-7° of trunk rotation is assessed by the scoliometer or by a scoliometer app on a smartphone, radiography can be performed to assess the Cobb angle. This radiography should be performed with the patient upright. A Cobb angle >20° may signify scoliosis, which may benefit from bracing, depending on skeletal maturity. Comparing the length from the pelvic brim to the pelvic floor on the left and the right is not indicated in the evaluation for scoliosis. The U.S. Preventive Services Task Force changed its recommendation for scoliosis screening from grade D to grade I in 2018. Bracing has been found to reduce by over 50% the chance that mild to moderate curvatures will progress to curvatures of greater than 50°

A 34-year-old male presents with a 2-week history of right plantar heel pain that began after he started training for a marathon. The pain is most severe immediately upon standing in the morning and then gradually improves somewhat after ambulation. It worsens again if he stands after sitting for a period of time or after excessive walking or running. Which one of the following is supported by evidence as a first-line intervention to provide pain relief for this condition? A) Night splints B) Plantar fascia stretching exercises C) Acupuncture D) Extracorporeal shock wave therapy E) Platelet-rich plasma injection

ANSWER: B This patient most likely has plantar fasciitis. Stretching exercises are effective in reducing heel pain caused by plantar fasciitis. Clinical trials regarding pain relief with the use of night splints are conflicting and thus inconclusive. The American College of Foot and Ankle Surgeons does not advocate for or against acupuncture to treat plantar fasciitis, as the studies available are of low quality. Extracorporeal shock wave therapy is only recommended after conservative therapies fail and for chronic plantar fasciitis. Platelet-rich plasma injections may be indicated in refractory plantar fasciitis but are not considered first-line therapy for an acute presentation.

A 55-year-old female presents for a telehealth visit because of a 2-month history of right lateral hip pain. Her symptoms began shortly after she started jogging. The pain worsened a week ago after a long car trip. She reports that the pain has been interfering with her sleep quality, particularly if she rests on her right side. You ask her to walk away from her video camera so you can observe her ambulation. You note a Trendelenburg gait with her body shifting to the right side. Which one of the following is the most likely diagnosis? A) Femoroacetabular impingement B) Greater trochanteric pain syndrome C) A hamstring strain D) A labral tear E) Sacroiliac joint dysfunction

ANSWER: B This patient presents with lateral hip pain and symptoms of greater trochanteric pain syndrome, which can include bursitis as well as gluteus medius tendinopathy or tears. She exhibits a Trendelenburg gait, which indicates gluteus muscle weakness. Both femoroacetabular impingement and labral tears generally cause anterior hip pain. Femoroacetabular impingement is one of the most common causes of hip pain in young adults and is usually caused by a cam deformity and/or a pincer deformity of the hip joint. Labral tears are usually associated with a history of trauma or sports-related injury. Hamstring injuries and sacroiliac joint dysfunction generally cause posterior hip pain. Hamstring strains are also associated with a history of trauma, sports-related injury, or overuse.

A 27-year-old male presents to establish care after relocating to the community. He was diagnosed with β-thalassemia major at birth and has been transfusion-dependent since early childhood. His microcytic anemia is stable with blood transfusions every 4 weeks, but his most recent DEXA scan indicates an advancement from osteopenia to osteoporosis. In addition to bisphosphonates, calcium, and vitamin D, which one of the following medications may improve his bone density? A) Hydroxyurea (Hydrea) B) Vitamin C C) Zinc D) Deferoxamine (Desferal) E) Luspatercept (Reblozyl)

ANSWER: C In addition to bisphosphonates, calcium, and vitamin D, zinc supplementation is recommended to improve bone density in patients with thalassemia and osteoporosis (SOR C). Though hydroxyurea is an indicated therapy to minimize the frequency of blood transfusions needed in transfusion-dependent thalassemia, it does not improve bone density (SOR C). Vitamin C supplementation does not improve bone health in patients with thalassemia and osteoporosis. Deferoxamine infusions are indicated when ferritin levels are >1000 ng/mL in patients with transfusion-dependent thalassemia to reduce iron overload (SOR C). Luspatercept reduced transfusion burden by 33% in a phase 3, randomized study but is not indicated to improve bone density.

A 24-year-old female presents to your office in Maine in November because for the past few weeks she has been sleeping much more than usual and craving sweets. She says she has gained 8 lb since her symptoms started. She enjoys her job and is in a stable relationship. She feels somewhat depressed but does not have any suicidal or homicidal thoughts. She has had similar episodes in the fall and winter for the past 4 years, and her symptoms usually resolve in the spring. The symptoms are starting to affect her quality of life and she asks for your advice. Which one of the following would be the most appropriate treatment at this time? A) Vitamin D supplementation B) Trazodone C) Light therapy D) A high-protein diet

ANSWER: C This patient presents with symptoms consistent with seasonal affective disorder (SAD). According to the DSM-5, this condition is not defined as a separate diagnosis, but instead a variant of major depressive or bipolar disorder. SAD is a mood disorder with depressive symptoms occurring at a specific time of year with full remission in between episodes, which usually occur during fall and winter months. A less common form can present during summer or spring. Criteria specify that full remission must occur when the specific season ends and there must be at least two consecutive years of mood episodes. The pathology is unclear but risk factors include family history, living at a more northern latitude, female sex, and age 18-30. First-line therapy for the treatment of SAD includes light therapy (SOR A), with a response rate of about 50%; cognitive therapy (SOR A); and antidepressants such as SSRIs (SOR B). Vitamin D supplementation is not a first-line treatment for SAD, and trazodone would not be used for someone who is already having issues with excessive sleeping. There is no convincing evidence that a high-protein diet is an effective treatment for SAD. For this patient with regular recurrences, long-term preventive intervention each year with light therapy starting in the early fall is indicated.

A 37-year-old male presents for a physical evaluation prior to starting a new job in a hospital. He recently immigrated from Uganda. An interferon-gamma release assay (IGRA, QuantiFERON-TB Gold) is positive. He is otherwise healthy. He has not had any cough, fever, unintended weight loss, or night sweats. Which one of the following is the most appropriate next step? A) Tuberculin skin testing B) Inducing sputum for mycobacterial culture C) Chest radiography D) Proceeding with treatment for latent tuberculosis E) Proceeding with treatment for active tuberculosis

ANSWER: C This patient's tuberculosis (TB) screening test is positive, and the next step in the evaluation involves determining whether he has a latent infection or active disease. Diagnosis of latent TB requires ruling out active disease by assessing the patient clinically with a history, physical examination, and chest radiograph. If this evaluation does not suggest active disease, sputum studies are not needed. Interferon-gamma release assays (IGRA), which are blood tests used to screen for TB infection, are more accurate than tuberculin skin testing, so a tuberculin skin test is not needed. Treatment should not be started until a determination of latent versus active TB is made.

A 42-year-old Asian male presents for follow-up of elevated blood pressure. He has no additional chronic medical problems and is otherwise asymptomatic. An examination is significant for a blood pressure of 162/95 mm Hg but is otherwise unremarkable. Laboratory work shows that his BMP is within normal limits. According to the American College of Cardiology/American Heart Association 2017 guidelines, which one of the following would be the most appropriate medication to initiate at this time? A) Clonidine (Catapres), 0.1 mg twice daily B) Hydralazine, 25 mg three times daily C) Lisinopril/hydrochlorothiazide (Zestoretic), 10/12.5 mg daily D) Metoprolol tartrate (Lopressor), 25 mg twice daily E) Triamterene (Dyrenium), 50 mg daily

ANSWER: C This patient has hypertension and according to both JNC 8 and American College of Cardiology/American Heart Association 2017 guidelines, antihypertensive treatment should be initiated. For the general non-African-American population, monotherapy with an ACE inhibitor, an angiotensin receptor blocker, a calcium channel blocker, or a thiazide diuretic would be appropriate for initial management. It is also appropriate to initiate combination antihypertensive therapy as an initial management strategy, although patients should not take an ACE inhibitor and an angiotensin receptor blocker simultaneously. Studies have shown that blood pressure control is achieved faster with the initiation of combination therapy compared to monotherapy, without an increase in morbidity. Lisinopril/hydrochlorothiazide would be an appropriate choice in this patient. Alpha blockers, vasodilators, beta-blockers, and potassium-sparing diuretics are not recommended as initial choices for the treatment of hypertension.

A 32-year-old female presents to your office 3 months after surviving a serious rollover car accident. Since the accident she has had flashbacks, nightmares, and difficulty sleeping. She has been unable to resume work or care for her young children due to difficulty concentrating and feeling like she is in a daze. She has not been able to drive, and riding in a vehicle triggers anxiety and fear. She tells you that she cannot stop feeling responsible for the accident. She does not take any medications and has no history of substance use. After performing a structured diagnostic interview and review of DSM-5 criteria to confirm your diagnosis, you discuss treatment options. She is not willing to consider psychotherapy at this time. Which one of the following would be the most appropriate pharmacotherapy? A) Clonazepam (Klonopin) B) Divalproex (Depakote) C) Fluoxetine (Prozac) D) Quetiapine (Seroquel) E) Risperidone (Risperdal)

ANSWER: C This patient has posttraumatic stress disorder (PTSD). She was exposed to threatened death and injury (DSM-5 criterion A) and exhibits multiple symptoms from several clusters of the DSM-5 criteria for PTSD (reliving of the traumatic event [criterion B], avoidance of trauma-related stimuli [criterion C], negative thoughts or feelings that began or worsened after the trauma [criterion D], and trauma-related arousal and reactivity that began or worsened after the trauma [criterion E]). She has symptoms that have caused distress and functional impairment for more than 1 month and are not triggered by medication or substance use (criteria F-H). Individual, trauma-focused psychotherapy has strong evidence for benefit in the treatment of PTSD and is recommended as the first-line treatment. If psychotherapy is not available or preferred by the patient, pharmacotherapy is then recommended. Among the options listed, fluoxetine has the strongest evidence of efficacy as monotherapy for PTSD. There is a lack of evidence for the efficacy of benzodiazepines such as clonazepam, antiepileptics such as divalproex, and atypical antipsychotics such as quetiapine and risperidone. Furthermore, risks outweigh any potential benefits from these medications.

Toddler's fractureA 24-month-old female is brought to your office by her mother because the child will not stand on her right leg. Yesterday the patient was playing at the park and her mother did not notice any injury occur. There has been no recent illness or fever. The child was born at full term, has had no medical problems, and is up to date on vaccinations. The patient's vital signs are normal. A physical examination reveals a healthy-appearing child in no apparent distress. She grimaces and pulls away with palpation of the right leg over the lower tibia and she will not bear weight. She has full passive range of motion of her hip, knee, and ankle joints bilaterally without apparent pain. Anteroposterior and lateral radiographs of her right tibia and fibula show no abnormalities. Which one of the following would be the most appropriate next step in management? A) Reassurance only B) A CBC and C-reactive protein level C) Immobilization with a cam boot, and repeat radiographs in 7 days D) Bone scintigraphy E) Referral to an orthopedic surgeon

ANSWER: C A nondisplaced spiral fracture of the distal tibial shaft (toddler's fracture) should be suspected in children from 9 months to 3 years of age who present with pain in the distal third of the tibia after minor or even unnoticed injury. Toddler's fractures can have subtle radiographic findings and may not be visible on initial radiographs, so repeat radiography to look for healing is appropriate. Standard treatment is immobilization of the affected leg. While the fracture may heal without immobilization, reassurance alone is not recommended given the unclear diagnosis. If repeat radiography is negative and symptoms have resolved, reassurance may then be appropriate. For children with possible septic arthritis, laboratory studies should be considered, but in this case there are no signs of infection. Bone scintigraphy is more sensitive than radiography and can be considered if follow-up radiography is negative and symptoms persist. Toddler's fractures routinely heal without complication, so referral to an orthopedic surgeon at this time would be premature.

A 73-year-old female with diabetic neuropathy and osteoarthritis of the knees sees you to request a prescription for an assistive mobility device. The neuropathy has caused poor balance and the knee pain has made walking more painful. As a result her physical endurance has declined over the last several months. Which one of the following assistive devices would be most appropriate for this patient? A) A cane B) Crutches C) A walker D) A wheelchair

ANSWER: C A walker would be the most appropriate assistive device for this patient given her balance limitations and bilateral extremity pain. Canes are most effective for unilateral lower extremity limitations and should be held in the hand opposite the affected leg and advanced simultaneously with the affected leg. Using a cane also requires good balance and dexterity, which are limited in this patient. Crutches require significant upper body strength, balance, and increased energy expenditure, which makes their use impractical in many older adults. Wheelchairs are generally the last option, as patients who can walk should do so to maintain function and avoid deconditioning. Referral to a physical therapist can be helpful to determine the most appropriate assistive device.

A 90-year-old male with metastatic prostate cancer and osteoarthritis of the knees is brought to the hospital with acute kidney injury due to dehydration, along with generalized weakness and ambulatory dysfunction. Prior to his hospitalization he lived independently with part-time caregiver assistance. Over the next 2 days his renal function improves, although physical and occupational therapy evaluations reveal significant deficits in his activities of daily living. The decision is made to transfer him to a skilled nursing facility. In planning for his discharge, you discuss completion of a Physician Orders for Life-Sustaining Treatment (POLST) form with the patient and his daughter, who says that her father already has an advance directive. She asks you what distinguishes a POLST form from other advanced care planning documents. You explain to her that, as opposed to an advance directive, a POLST form A) does not require a signature from a treating physician B) is required for patients with a terminal illness in hospice care C) provides specific orders for emergency medical services D) should be completed for all patients during an annual Medicare wellness visit E) supersedes the expressed wishes of a patient who can communicate

ANSWER: C An organization that is now known as National POLST began as an advisory panel and task force to improve consistency in honoring the wishes of patients with serious illness or frailty regarding end-of-life care. POLST, originally an acronym for Physician Orders for Life-Sustaining Treatment, varies in name at the state level, although at the national level POLST is defined as a portable medical order. POLST forms differ from other legal advanced care planning documents such as advance directives and living wills, although the processes can work together. The primary purpose of a POLST form is to provide specific medical orders in the event that a patient is unable to communicate. Legal advanced care planning documents, in contrast, express general treatment wishes and identify a health care proxy for surrogate decision-making but do not provide specific orders for care. POLST forms are completed by a treating physician and signed by the patient or surrogate and the physician, unlike advance directive documents, which are completed by the individual and are often notarized. POLST forms are intended only for patients with serious illness and frailty, not for healthy persons, whereas advance directives can be completed by any competent adult. POLST completion is voluntary and is neither a requirement for entering hospice nor a routine part of the Medicare annual wellness visit. Neither document type is intended to supersede the expressed wishes of a patient who can communicate.

You diagnose attention-deficit/hyperactivity disorder in a 4-year-old female. Which one of the following would be the most appropriate first-line treatment? A) Dietary modification B) Vitamin supplementation C) Behavioral intervention D) Atomoxetine (Strattera) E) Methylphenidate (Ritalin)

ANSWER: C Attention-deficit/hyperactivity disorder in preschool children aged 4-5 years should be managed initially with behavioral intervention. Starting at age 6, pharmacologic medication such as atomoxetine or methylphenidate could be considered. There is no evidence of any benefit with dietary modification or vitamin supplementation.

Which one of the following regimens is recommended for the treatment of hypertension in a patient with stage 3 chronic kidney disease and proteinuria? A) A loop diuretic and a β-blocker B) An ACE inhibitor and an angiotensin receptor blocker C) An ACE inhibitor and a thiazide diuretic D) A calcium channel blocker and a thiazide diuretic E) A potassium-sparing diuretic and a thiazide diuretic

ANSWER: C Based on a reduction in all-cause mortality, JNC 8 advises more intensive blood pressure control in patients with chronic kidney disease (CKD) and proteinuria. This is most often achieved with combination therapy, with either an ACE inhibitor or an angiotensin receptor blocker (ARB), plus either a thiazide diuretic or a calcium channel blocker. ACE inhibitors and ARBs both slow the progression of CKD to end-stage renal disease and reduce morbidity and mortality in patients with CKD (SOR A). However, combining an ACE inhibitor and an ARB actually increases the risk of end-stage renal disease, so these drugs should not be used simultaneously. The other combinations listed may be effective in improving blood pressure control, but in patients with CKD and proteinuria the combination of an ACE inhibitor or an ARB with a diuretic or calcium channel blocker is most effective for lowering morbidity and mortality.

A recently divorced 47-year-old male comes to your office appearing disheveled, with the smell of alcohol on his breath. His Patient Health Questionnaire-9 (PHQ-9) score today is 20, and his last PHQ-9 score was 7. He has a history of depression and is currently taking citalopram (Celexa). The patient is tearful during the encounter and admits to thinking the world would be better without him in it. He does not have a weapon with him but keeps a gun in an unlocked drawer in his nightstand. Which one of the following would be most appropriate at this point? A) Avoiding direct inquiry about suicide B) Calling 911 C) Crisis planning D) Creating a suicide prevention contract E) Withholding psychogenic medications

ANSWER: C Crisis planning is recommended for patients presenting with suicidal ideation (SOR B). By identifying social support, local resources, and counseling services, suicidal ideation and days spent in the hospital can be reduced. Direct inquiry about suicide is recommended to better evaluate and treat suicidal patients with more favorable outcomes (SOR B). Though calling 911 may be appropriate for transportation for inpatient therapy if involuntary treatment is recommended, further assessment is needed in this case. Suicide prevention contracts do not effectively prevent suicide (SOR B). Psychogenic medications should not be withheld when treating a patient with suicidal ideation. Evidence has shown that the combination of pharmacotherapy and psychotherapy is most effective (SOR C).

In the U.S. legal system, which one of the following is a required finding to determine that medical malpractice has occurred? A) Conflict of interest B) A financial charge rendered to the patient C) A causal relationship between breach of duty and injury to the patient D) Presentation of the case to a grand jury E) Testimony from an expert witness for the plaintiff

ANSWER: C For a physician to be found guilty of medical malpractice in the United States, the plaintiff must show that the physician acted negligently in providing care and that the negligence resulted in injury. Determining this requires proof of the following four legal components: (1) a professional duty owed to the patient; (2) a breach of said duty; (3) an injury caused by the breach; and (4) resulting damages. Although physicians are legally required to disclose conflicts of interest, having a conflict of interest is not an essential element in a malpractice decision. Malpractice may occur even if the patient is not rendered a financial charge, provided that the four essential criteria are met. Medical malpractice cases that are not settled or otherwise dismissed proceed to a jury trial, not to a grand jury. Grand juries are part of the criminal indictment process and are not relevant to medical malpractice cases. While expert witness testimony is often used in a malpractice case, typically to help establish whether there has been a breach of a professional standard of care, such testimony is not a requirement for a medical malpractice decision.

A patient's office spirometry results reveal a normal FEV1/FVC ratio and a decreased FVC. Which one of the following is the most likely explanation for these findings? A) A normal pattern B) A mixed pattern C) A restrictive pattern D) A reversible obstructive pattern E) An irreversible obstructive pattern

ANSWER: C Forced vital capacity (FVC) is the total amount of air that can be expelled from full lungs. A decreased FVC on spirometry indicates a restrictive pattern. FEV1 is the volume of air (in liters) that is exhaled in the first second during forced exhalation after maximal inspiration. A normal FEV1/FVC ratio and normal FVC would indicate a normal pattern. A decreased FEV1/FVC ratio with a decreased FVC is consistent with a mixed pattern. A reduced FEV1/FVC ratio indicates an obstructive pattern. A bronchodilator is then utilized to determine whether the obstructive pattern is reversible or irreversible.

Montelukast (Singulair) has an FDA boxed warning related to an increased risk of: A) delirium B) myocardial infarction C) suicidality D) venous thromboembolism

ANSWER: C In March 2020, the FDA upgraded its warning label for montelukast to a boxed warning (black box warning) based on the trends for all neuropsychiatric adverse events, including suicidality, associated with montelukast use reported in the FDA Adverse Event Reporting System database from the date of FDA approval in February 1998 through May 2019 (SOR B). The boxed warning does not indicate an increased risk of delirium, myocardial infarction, or venous thromboembolism

A 45-year-old female presents to your clinic with a 2-day history of worsening sinus pain and pressure that is worse on the right cheek and radiating into a maxillary molar. She recently recovered from a week-long cold and was getting better when she suddenly relapsed. Her symptoms have progressed over the last 12 days, culminating in a fever of 102°F last night. The patient reports a foul taste in the back of her throat. She has been drinking plenty of fluids, performing sterile saline sinus rinses twice daily, and using acetaminophen and ibuprofen for pain relief. She has not had a cough except when clearing thick postnasal phlegm. She does not have itching of the eyes or nose, or a history of environmental allergies. On examination her lungs are clear to auscultation bilaterally. She has a temperature of 38.9°C (102.0°F), a pulse rate of 100 beats/min, and a respiratory rate of 14/min. She has no known drug allergies. Which one of the following would be the most appropriate treatment for this patient? A) Supportive care for up to 2 additional days B) Continued saline nasal rinses plus inhaled fluticasone (Flonase Allergy Relief) until symptoms resolve, and follow-up if symptoms worsen or the fever increases C) Amoxicillin/clavulanate (Augmentin) for 5-7 days D) Azithromycin (Zithromax) for 5 days, and follow-up if symptoms worsen or the fever increases E) Levofloxacin for 28 days

ANSWER: C Most sinus infections are viral in origin, although there are specific circumstances in which antibiotics are appropriate. This patient exhibits all of the diagnostic clues that are most reliable for determining a bacterial etiology for the sinus infection: worsening symptoms with a new fever of 38.9°C (102°F), duration >10 days, signs of double sickening, unilateral facial and tooth pain, and cacosmia (perception of a foul odor). Since 2012, the Infectious Diseases Society of America, the CDC, and Sanford have all changed their recommendations to amoxicillin/clavulanate as the first-line antibiotic of choice due to emerging antibiotic resistance. However, in practice, azithromycin is commonly prescribed despite significant Streptococcus pneumoniae resistance of approximately 40% or higher. One-third of bacterial sinus infections are caused by S. pneumoniae, another one-third by Haemophilus influenzae, and the remainder by Moraxella catarrhalis, group A Streptococcus, Staphylococcus aureus, and anaerobes. Previous guidelines recommended treatment with antibiotics after 14 days of symptoms. While it is true that most sinus infections, even those that are bacterial in origin, will resolve after 14 days with or without treatment, characteristics such as a long duration, worsening illness, and new fever indicate the need for antibiotics. Nasal rinses may be of some benefit to patients, but evidence is mixed. Fluticasone is recommended as adjunctive treatment in acute sinusitis to facilitate drainage of the sinuses, but this patient's clinical signs and symptoms warrant an appropriate antibiotic. Levofloxacin is an appropriate choice for patients who are allergic to penicillin or who have recurrent bacterial sinus infections. However, treatment for a long duration is appropriate only in the setting of recurrent infection or treatment failure, and after consultation with an otolaryngologist.

You see a 30-year-old male for a routine health maintenance visit. The physical examination is normal, but he mentions that he has an overwhelming urge to keep checking and rechecking doors to make sure they are locked. He senses that something dangerous will happen if he does not do this. These thoughts and behaviors have become very distressing to him, and have started to interfere with his work and home life. Which one of the following would be most appropriate at this time? A) Alprazolam (Xanax) B) Risperidone (Risperdal) C) Sertraline (Zoloft) D) Referral for psychodynamic psychotherapy

ANSWER: C Obsessive-compulsive disorder (OCD) affects 3% of the population and is characterized by intrusive thoughts (obsessions) and repetitive behaviors (compulsions). Obsessions are often religious, sexual, or violent in nature. They may include pathological doubting, preoccupation with contamination, concerns with symmetry, and a sense that something unpleasant or dangerous will happen if a particular ritual is not performed precisely. Typical compulsions include counting, checking, repeating, cleaning, and arranging behaviors. For the diagnosis to be made, these symptoms must be severe enough to cause marked distress or to impair functioning. Most patients develop symptoms prior to age 35, and a large number of them keep these symptoms a secret. There is often a delay of 5-10 years before the illness comes to medical attention. With optimum treatment, 90% have moderate to marked improvement. Treatment of OCD requires the integration of various approaches to maximize the outcome. Most patients experience substantial improvement using a combination of pharmacotherapy, particularly SSRIs, and cognitive behavioral therapy. Benzodiazepines such as alprazolam are capable of relieving generalized anxiety, but do not affect obsessions or compulsions. Antipsychotics such as risperidone may be added to an SSRI as second-line pharmacotherapy. Traditional psychodynamic psychotherapy is not effective for OCD.

A 35-year-old female presents to discuss her recent diagnosis of metastatic breast cancer. She has many questions about potential treatments and outcomes. You discuss palliative care with her. Which one of the following is true regarding patients who receive palliative care? A) Patients must have a life expectancy of 6 months or less to qualify B) Care must be offered in person (outpatient or inpatient) C) Patients may simultaneously undergo aggressive chemotherapy D) Costs for palliative care are higher than for usual chronic disease care E) Medicare and most commercial insurances provide bundled payments for palliative care

ANSWER: C Palliative care can be offered to patients of any age with serious illness. It can be provided at any stage of the illness and there are no life-expectancy criteria. Unlike hospice, patients receiving palliative care may simultaneously undergo aggressive therapy. Palliative care can be offered in any setting, including via telehealth. It has been shown that patients receiving palliative care incur fewer health care costs than patients receiving usual care. The Choosing Wisely campaign recommends early referral to palliative care, as it improves patient care, increases patient satisfaction, and reduces costs. While Medicare covers hospice care through bundled payments, there is no similar coverage for palliative care, which can limit access. Commercial coverage for palliative care is inconsistent.

A 39-year-old female with a BMI of 42 kg/m2 and a history of hypertension, diabetes mellitus, hyperlipidemia, obstructive sleep apnea, and hypothyroidism has been struggling to lose weight. Her medical conditions are controlled. She asks you if weight loss surgery would be better for her than continued dietary efforts. Which one of the following is true regarding weight loss surgery? A) Post-surgical dietary recommendations include consuming carbohydrates first at each meal B) Post-surgical dietary recommendations include drinking fluid with each meal C) Diabetes remission occurs in the majority of patients 2 years after a Roux-en-Y procedure D) Surgical treatment for obesity is equivalent to nonsurgical interventions in overall length of survival E) Surgical treatment for obesity leads to a lifelong need for quarterly laboratory studies to check for nutritional deficiencies

ANSWER: C Surgical treatment for obesity results in remission of diabetes mellitus in 60%-80% of patients at 2 years and sustained remission in 30% at 15 years after a Roux-en-Y procedure. Postsurgical dietary recommendations include consuming protein first at each meal, rather than carbohydrates, to prevent malnutrition, and separating consumption of liquids from solids by 15-30 minutes to avoid food passing through the stomach too quickly, which can lead to a decreased sensation of satiety. Patients who are treated surgically for obesity rather than with nonsurgical interventions have a longer overall length of survival. All-cause mortality is decreased by 30%-50% at 7-15 years after surgery. Evaluation for nutritional deficiencies should be performed quarterly for the first year after surgery; after that, annual checks are recommended.

A 74-year-old male whom you have not seen for several years presents with fatigue, an 8-lb weight loss, and musculoskeletal pain for the last 6 months. He had been in good health before these symptoms started. He states that he has a "deep ache" in his lower back, hips, and shoulders that awakens him at night. Plain films of the lumbar spine ordered by an orthopedic surgeon revealed osteopenia and degenerative disease with osteophytes. A bone scan was within normal limits for his age. The only abnormalities you detect on a thorough physical examination are tenderness to percussion over the scapulae, lumbar vertebrae, and posterior pelvis, and a mildly enlarged but smooth prostate. Laboratory Findings: Hemoglobin 10.3 g/dL (N 14.0-17.0) Mean corpuscular volume 90 μm 3 (N 80-100) PSA 4.6 ng/mL (N 0.0-4.0) Calcium 10.9 mg/dL (N 9.0-10.5) Phosphorus 4.2 mg/dL (N 3.0-4.5) Albumin 3.0 g/dL (N 3.5-5.5) Globulin 6.7 g/dL (N 2.0-3.5) Alkaline phosphatase 86 U/L (N 30-92) BUN 26 mg/dL (N 8-20) Creatinine 2.5 mg/dL (N 0.7-1.5) Of the following, the most likely diagnosis is: A) carcinoma of the prostate metastatic to bone B) hyperparathyroidism C) multiple myeloma D) osteomalacia E) polymyalgia rheumatica

ANSWER: C The combination of a high globulin-to-albumin ratio, anemia, renal insufficiency, and hypercalcemia in a patient with diffuse musculoskeletal pain is highly suggestive of multiple myeloma. Serum and urine immunoelectrophoresis would be the next test to order. Carcinoma of the prostate metastatic to bone should be seen on a bone scan and the PSA level would be much higher. Hyperparathyroidism is part of the differential diagnosis, but a low phosphorus level would be expected. An elevated alkaline phosphatase level would be expected in osteomalacia. Polymyalgia rheumatica is more common in women and would not be associated with elevated calcium and globulins and this degree of anemia.

You see a 50-year-old male in your office with a 2-week history of cough that began as mild and intermittent, but now occurs as paroxysms that frequently cause him to vomit. He feels very tired after these coughing fits. He also reports rhinorrhea throughout the course of his symptoms. He has been feeling overheated but does not report documented fevers. He notes that he does not typically receive routine vaccinations. His vital signs include a temperature of 37.4°C (99.3°F), a pulse rate of 85 beats/min, a respiratory rate of 20/min, and an oxygen saturation of 93% on room air. He has no medication allergies. A test for COVID-19 is negative. Of the following treatments, which one would be the most beneficial for this patient's most likely condition? A) Continued supportive care only B) Tdap vaccination C) Azithromycin (Zithromax) D) Doxycycline E) Oseltamivir (Tamiflu)

ANSWER: C The duration and characterization of this patient's cough are most suggestive of pertussis. Of the options listed, azithromycin is the most appropriate for management of pertussis. Azithromycin is most effective for treatment and minimizing spread of the disease within 21 days of symptom onset. Sulfamethoxazole/trimethoprim and other macrolides, such as erythromycin and clarithromycin, are also acceptable options. Continued supportive care only does not provide the advantages of cure and minimization of community spread that are accomplished by initiating azithromycin. Symptomatic treatment with over-the-counter medication is appropriate but such supportive care does not replace the therapeutic advantages of azithromycin. This patient's objection to routine vaccination should be explored as priorities allow. He should be vaccinated against pertussis with Tdap as soon as feasible, but the vaccination would not provide immediate treatment of his current episode of pertussis. Doxycycline has shown benefit in other bacterial infections but does not provide effective treatment of pertussis. Based upon the duration of symptoms, quality of his cough, and lack of documented fevers, this patient is not likely to have influenza, so oseltamivir would not be appropriate.

Which one of the following is consistent with best practices for prescribing hormonal contraception? A) Delaying initiation until the patient's next menstrual period to avoid incidental use in early pregnancy B) Limiting prescription refills to 3 months at a time C) Obtaining a thorough medical history to screen for contraindications D) Requiring in-person visits rather than telehealth visits for contraceptive counseling due to the need for a gynecologic examination E) Requiring patients to have an up-to-date Papanicolaou test and sexually transmitted infection screening

ANSWER: C There are many patient-related historical factors that may affect the safety and choice of hormonal contraception, but very few physical factors are likely to be found on examination that would not otherwise have been identified. Obtaining a thorough medical history is standard practice, but the Choosing Wisely campaign recommends against requiring a pelvic or other physical examination prior to prescribing oral contraceptives. It is unnecessary to wait to begin hormonal contraception until after the next menses, as inadvertent exposure to oral contraception will not harm an early pregnancy. Prescribing a 1-year supply of hormonal contraceptives improves adherence and lowers cost. There is broad consensus that sexually transmitted infection screening and Papanicolaou testing should not be required to prescribe contraception.

You see a previously healthy 8-year-old female for a well child check. She was born at full term and adopted at birth. She has a history of methamphetamine exposure in utero. She is up to date on vaccinations and is doing well academically. She says she has friends at school and her mother confirms that her teachers report that she interacts well with the other students. Her mother notes, however, that the patient has persistent difficulties with anger and irritability. This behavior has been present since preschool, and while her mother thinks there has been some improvement, she is concerned that it has not resolved. At home, the child has frequent outbursts, often speaks hatefully when upset, refuses to follow instructions, and throws herself on the floor and kicks in frustration at times. Her sisters sometimes worry about upsetting her because they know she will react dramatically, although she has not been physically aggressive. Her mother notes that the patient often blames her sisters or others when she misbehaves. Which one of the following would be the most appropriate next step? A) Reassurance that the behavior should continue to improve with age B) Education on positive reinforcement of desired behaviors C) Obtaining further history to evaluate for additional mental health conditions D) A trial of risperidone (Risperdal) E) Referral for parent management therapy

ANSWER: C This child displays characteristics of oppositional defiant disorder (ODD). The DSM-5 criteria for a diagnosis of ODD include frequently losing one's temper, being easily annoyed, antagonism toward authority figures, deliberately annoying others, placing blame on others, and being spiteful or vindictive. These symptoms must occur for at least 6 months, cause distress or negative impacts, and not occur exclusively with substance use or in the course of a psychotic, depressive, or bipolar disorder. Treatment of common comorbid mental health conditions can be associated with improvement in ODD, so it is important to evaluate for attention-deficit/hyperactivity disorder, depression, and anxiety disorders, as well as ODD. Given the persistence of symptoms and maternal concern in this patient, reassurance alone would not be appropriate. Patients with ODD have a high risk of developing other mental health conditions later, and early therapy is recommended. While positive reinforcement is an important parenting strategy for children with ODD, it would not be expected to be effective in isolation. Medication is rarely indicated for ODD, and not as monotherapy. Parent management therapy is an important part of ODD treatment, but therapy should generally include both child therapy and parent training.

A 2-year-old male is brought to the urgent care clinic by his parents in February with a low-grade fever that started the night before. His mother awoke early in the morning when she heard his barking cough. He recently started attending preschool and the mother does not know of any sick contacts. On initial examination the patient is in mild respiratory distress and appears nontoxic. He does not have any rhinorrhea or congestion. He has a temperature of 38.2°C (100.8°F), a respiratory rate of 40/min, a heart rate of 145 beats/min, and an oxygen saturation of 96% on room air. No rashes or petechiae are present. The most appropriate next step in management would be A) humidified air B) albuterol via nebulizer C) oral dexamethasone D) a viral culture E) a chest radiograph

ANSWER: C This patient has a classic presentation of croup, which peaks in the fall and winter months. There may not be any particular history of sick contacts and it does not present with a prodrome, in contrast to respiratory syncytial virus. The diagnosis of croup is purely clinical and does not require laboratory studies, viral cultures, or imaging (SOR C). The treatment of croup includes corticosteroids such as dexamethasone in mild cases (SOR A) and the addition of epinephrine in moderate to severe cases (SOR A). The inhalation of humidified air does not improve outcomes (SOR B) nor does nebulized albuterol.

A 49-year-old male presents to the urgent care center with a fever, cough, and pleuritic chest pain. His medical history is unremarkable with no cardiac risk factors. The patient's vital signs include a temperature of 39.0°C (102.2°F), a heart rate of 120 beats/min, a respiratory rate of 24/min, a blood pressure of 90/58 mm Hg, and an oxygen saturation of 95% on room air. The patient is awake and able to follow commands. An examination reveals warm skin with a capillary refill time of 2-3 seconds. The patient has normal heart sounds with a rapid peripheral pulse and no evidence of jugular vein distention. A chest radiograph is suggestive of pneumonia. Which one of the following would be the most appropriate next step in treatment? A) Administering a 1-L bolus of half-normal saline B) Administering epinephrine by auto-injector C) Administering a minimum of 30 mL/kg of lactated Ringer's solution within the first 3 hours D) Starting a dobutamine drip at 2 μg/kg/min E) Starting a norepinephrine drip at 0.05 μg/kg/min

ANSWER: C This patient has an acute febrile illness and meets criteria for systemic inflammatory response syndrome (SIRS) and decompensated shock. Shock is a medical emergency requiring urgent treatment to prevent death or other complications. The four types of shock are differentiated based on clinical signs. Correct treatment hinges on accurate determination of the type of shock. This patient demonstrates high-output shock typical of septic shock. Initial treatment of septic shock begins with fluid resuscitation using isotonic crystalloid by an intravenous or intraosseous route. Recent guidelines recommend a minimum of 30 mL/kg of isotonic crystalloid, with a preference for lactated Ringer's solution over normal saline. Hypotonic solutions, such as half-normal saline, should never be administered as a bolus. There is no indication for epinephrine or dobutamine in this patient. Norepinephrine can be indicated for septic shock that has not responded to fluid resuscitation

A 32-year-old female comes to your office because of chronic diarrhea, abdominal cramping, and bloating. She has had these symptoms for many years but has never discussed them in depth with a physician. A thorough history and physical examination are most consistent with irritable bowel syndrome (IBS). You order IgA tissue transglutaminase (tTG) antibody and fecal calprotectin testing to rule out other conditions and both are negative. She has expressed an interest in nonpharmacologic measures as initial management of her IBS. Which one of the following should you recommend initially, given that it has the best evidence of benefit for her condition? A) A gluten-free diet B) A low-FODMAP diet C) Soluble fiber D) Prebiotics E) Probiotics

ANSWER: C This patient has diarrhea-predominant irritable bowel syndrome (IBS-D) and may benefit from validation of her symptoms and a clear diagnosis that has several substantiated treatment options. A 2021 clinical guideline from the American College of Gastroenterology (ACG) is based on a systematic review performed by a committee of experts in this field. Based on this review, soluble fiber (but not insoluble fiber) has good evidence for the alleviation of global IBS symptoms and is recommended strongly as a first-line intervention. In contrast, a gluten-free diet has not been shown to be beneficial for IBS. A diet low in fermentable oligosaccharides, disaccharides, monosaccharides, and polyols (FODMAP) has low-quality evidence supporting its benefit but merits a trial in patients who do not have an adequate response to fiber supplementation. The ACG has not commented on prebiotics as a treatment for IBS. A 2018 systematic review concluded that while there are small individual studies suggesting benefit for prebiotics (and probiotics) there is inadequate long-term consistent evidence to support their routine use. Probiotics have mixed and low-quality evidence for benefit in IBS. Because of the inconsistent data the ACG recommends against their use.

A 4-year-old male is brought to your office by his mother because of fevers, irritability, runny nose, and cough for the past week. On examination he is noted to have bilateral conjunctivitis; dry cracked lips; a maculopapular rash; edema and erythema of his palms and soles bilaterally; and a nontender, enlarged, right anterior cervical lymph node. Which one of the following should be included in the evaluation of this patient's condition? A) Chest radiography B) Neck ultrasonography C) Echocardiography D) Neck CT E) Cardiac MRI

ANSWER: C This patient has the classic clinical manifestations of Kawasaki disease. All patients with Kawasaki disease should undergo echocardiography due to the high risk of coronary artery dilation and aneurysm associated with the disease. Chest radiography, neck ultrasonography, neck CT, and cardiac MRI are not recommended in the evaluation of a patient with Kawasaki disease and would only be recommended if another clinical indication for these studies were present.

A 25-year-old female presents to your office for an annual health maintenance visit. You note a BMI of 17 kg/m2 , a heart rate of 66 beats/min, and a blood pressure of 110/64 mm Hg. The patient reports exercising for 2 hours each day, incorporating cardio and light weights. The patient presents a detailed food diary and asks for advice about how to adjust her nutrition to lose weight. In order to provide the best care for this patient, which one of the following would you order? A) A fecal calprotectin level B) Stool cultures C) A DEXA scan D) Chest radiography E) Thyroid ultrasonography

ANSWER: C This patient likely has an eating disorder. A DEXA scan is recommended to assess for low bone mineral density in patients with suspected or diagnosed eating disorders. Other appropriate screenings include orthostatic vital signs; a basic metabolic panel; a CBC; magnesium, phosphorus, prealbumin, and amylase levels; thyroid testing; and an EKG. Fecal calprotectin levels, stool cultures, chest radiography, and thyroid ultrasonography are not appropriate for this patient.

A previously healthy 44-year-old female presents to the emergency department (ED) with severe, sharp, right upper quadrant abdominal pain and nausea that began shortly after eating dinner. On examination she is noted to have a low-grade fever with a positive Murphy sign. A laboratory analysis is notable for leukocytosis with a left shift and a mildly elevated total bilirubin level. A lipase level and liver transaminases are normal. Ultrasonography reveals several small gallstones, gallbladder wall thickening, and pericholecystic fluid. After receiving intravenous fluids, pain management, and antiemetic treatment in the ED, her symptoms improve. In addition to intravenous antibiotics, the most appropriate next step in management would be to offer: A) expectant management B) endoscopic retrograde cholangiopancreatography (ERCP) C) laparoscopic cholecystectomy D) outpatient general surgery consultation

ANSWER: C This patient presents with signs and symptoms of acute cholecystitis, and ultrasonography confirms the presence of gallstones and gallbladder inflammation. While most patients with acute cholecystitis will have symptoms that improve with supportive care over 2-7 days, the risk of recurrent symptoms and complications increases with delayed surgical intervention. The Choosing Wisely initiative recommends that surgical treatment be offered to the patient during the initial hospitalization. The Society of American Gastrointestinal and Endoscopic Surgeons has found that laparoscopic cholecystectomy is safe and cost-effective in the immediate hospital setting. This stable, uncomplicated patient should be offered laparoscopic cholecystectomy during the current visit. Offering outpatient options such as expectant management and surgical consultation at a later date may increase this patient's risk of recurrent symptoms and complications as well as costs. Since she does not have signs of obstructive cholangitis such as elevated liver enzymes and jaundice, endoscopic retrograde cholangiopancreatography (ERCP) is not indicated.

A 17-year-old male with a history of type 2 diabetes sees you because of fatigue and a 15-lb weight loss in the past month. The patient reports excessive and frequent urination, thirst, and nausea. His only medication has been metformin, but he states that he stopped taking it 6 months ago. His current weight in your office is 93 kg (205 lb), which confirms the reported weight loss. His blood pressure is 130/78 mm Hg, his pulse rate is 90 beats/min, and his temperature is 37.0°C (98.6°F). A physical examination is otherwise unremarkable. A capillary blood glucose level is 348 mg/dL, a hemoglobin A1c is 11.5%, serum ketones are negative, and a urinalysis shows 3+ glucosuria with concentrated urine but is otherwise normal. Which one of the following would be the most appropriate treatment? A) Resuming oral metformin B) Starting oral empagliflozin (Jardiance) C) Starting subcutaneous insulin D) Starting subcutaneous liraglutide (Victoza) E) Hospitalization for continuous intravenous insulin

ANSWER: C This patient presents with symptomatic hyperglycemia in a catabolic state. In such cases insulin therapy is the most reliable way to control hyperglycemia and reverse catabolism. Oral metformin would not be adequate to control this degree of hyperglycemia and might not be tolerated well, given that the current symptoms include nausea and weight loss. Similarly, both empagliflozin, which increases glucosuria and volume contraction, and liraglutide, which decreases gastric emptying and is likely to exacerbate nausea, are likely to be poorly tolerated in this situation. While rapid and effective treatment is essential to prevent further complications, hospitalization is not necessary since the patient has no evidence of diabetic ketoacidosis.

A 75-year-old female with no significant past medical history presents to your office with a recent onset of postural dizziness and lightheadedness. An evaluation reveals a diagnosis of orthostatic hypotension with no underlying etiology identified. After an unsuccessful trial of managing her symptoms with nonpharmacologic measures, she returns to discuss additional treatment options. Which one of the following oral medications would be the preferred initial treatment? A) Atomoxetine (Strattera) B) Clonidine C) Midodrine D) Phenylephrine E) Pyridostigmine (Mestinon)

ANSWER: C Treatment of orthostatic hypotension begins with identifying and addressing the underlying cause(s) when possible. This may include correcting a reversible medical condition or discontinuing an offending medication. Nonpharmacologic measures should be initiated next and typically include increasing fluid and sodium intake, improving physical fitness, wearing compression garments, and avoiding hot and humid environments. When additional treatment is needed, first-line medication options include midodrine or droxidopa, which act by increasing peripheral vascular resistance. Off-label use of atomoxetine or pyridostigmine may be considered as adjunct therapy but these medications are not part of the initial management. The alpha-antagonist clonidine typically causes a decrease in blood pressure through central action on the sympathetic nervous system. In patients with autonomic dysfunction, however, clonidine can increase venous return without a blood pressure-lowering effect and therefore improve orthostatic hypotension, but it should only be considered a supplementary treatment. The alpha-sympathomimetic medication phenylephrine may also be considered as a second-line option, but it is not part of the initial management of orthostatic hypotension

A 55-year-old female with type 2 diabetes, obesity, and hypertension presents for routine follow-up. Despite her best efforts with diet and exercise, she has been unable to achieve a healthy BMI. Her current medications include metformin, lisinopril (Zestril), and atorvastatin (Lipitor). A physical examination is remarkable only for a BMI of 32 kg/m2 . A hemoglobin A1c is 7.5%, which is unchanged from 3 months ago. A basic metabolic panel shows normal electrolytes and renal function. Which one of the following additional medications would be most likely to improve her glucose control and help her achieve weight loss? A) Glipizide (Glucotrol) B) Insulin glargine (Lantus) C) Liraglutide (Victoza) D) Nateglinide (Starlix) E) Sitagliptin (Januvia)

ANSWER: C While each of the listed medications has evidence of benefit for improving glycemic control, only the GLP-1 agonist liraglutide would be expected to cause weight loss. SGLT2 inhibitors are also associated with weight loss. Sulfonylureas such as glipizide, insulins such as glargine, and meglitinides such as nateglinide all increase the risk of weight gain. DPP-4 inhibitors such as sitagliptin are weight neutral.

A 4-year-old female is brought to your office by her father for a well child check. The father reports that the child is having difficulty using her albuterol (Proventil, Ventolin) metered-dose inhaler for asthma exacerbations and he is not sure whether it is improving her symptoms. On demonstration in the office, the child is unable to time her breathing with inhaler actuation. Which one of the following would you recommend? A) Montelukast (Singulair) B) Albuterol via oral liquid C) Albuterol metered-dose inhaler via a spacer device D) Albuterol via nebulizer E) Salmeterol inhaled (Serevent Diskus)

ANSWER: C Young children often have difficulty coordinating inhaler use, which can reduce the effectiveness of asthma medications. The use of spacer devices eliminates the need for coordination and increases medication delivery to the lungs. Oral albuterol is no longer recommended. Montelukast, nebulized albuterol, and inhaled salmeterol are not indicated as first-line treatment for asthma exacerbations.

A 35-year-old female presents with very pruritic, recurrent, grouped papules, vesicles, and erosions on her knees and elbows. She does not have any known connective tissue diseases, gastrointestinal disturbances, sexually transmitted infections, or recurrent exposures. A skin biopsy is consistent with dermatitis herpetiformis. A positive test for which one of the following is most consistent with this diagnosis? A) Anti-thyroid antibodies B) Herpes simplex virus antibody titers C) Intrinsic factor antibodies D) IgA tissue transglutaminase (tTG) antibodies E) Varicella zoster virus antibody titers

ANSWER: D Dermatitis herpetiformis is a very pruritic, papulovesicular reaction that is secondary to cutaneous IgA and immune complex deposition related to gluten sensitivity, as in celiac disease. The majority of patients do not have the gastrointestinal disturbances of celiac disease but do have the changes of gluten enteropathy on small bowel biopsies. The diagnosis is supported by elevated IgA tissue transglutaminase (tTG) antibodies, which is the serology of choice for diagnosing celiac disease. The rash frequently responds well to a gluten-free diet and is classically treated with dapsone. The disease is not related to thyroid disease, herpesviruses, or pernicious anemia.

Despite limited evidence, systemic corticosteroids are frequently prescribed for multiple conditions in primary care. Which one of the following conditions has grade A evidence for treatment with systemic corticosteroids? A) Acute bronchitis B) Acute pharyngitis C) Acute sinusitis D) Bell's palsy E) Lumbar radiculopathy

ANSWER: D Although many providers assume short-term systemic corticosteroids are safe, evidence shows multiple negative effects including elevated blood glucose and blood pressure, mood and sleep disturbance, and an increased risk of sepsis and venous thromboembolism. There are adequate trials to support the use of systemic corticosteroids within 3 days of the onset of Bell's palsy (SOR A). Adequate studies recommend against prescribing systemic corticosteroids for acute bronchitis in the absence of underlying asthma or COPD, or acute sinusitis (SOR B). There is insufficient evidence (SOR B) to support the routine use of systemic corticosteroids for patients with acute pharyngitis or lumbar radiculopathy.

A 43-year-old female with lifelong asthma asks if she would be a candidate for treatment with a biologic agent such as omalizumab (Xolair). A CBC reveals mild eosinophilia, indicating type 2 inflammatory asthma. In which one of the following patient scenarios should biologic treatment for asthma be considered? A) Any patient with poorly controlled, severe asthma B) A patient with severe non-type 2 asthma that is poorly controlled despite adherence to optimal therapy with long-term controller medication C) A patient with type 2 inflammatory asthma that is poorly controlled despite therapy with as-needed inhaled albuterol (Proventil, Ventolin) and low-dose inhaled corticosteroids D) A patient with severe type 2 inflammatory asthma that is poorly controlled despite adherence to optimal therapy with long-term controller medication

ANSWER: D Biologic therapy for asthma targets type 2 inflammation pathways. According to the 2019 Global Initiative for Asthma (GINA) guidelines, diagnosis and management of severe asthma includes determination of the asthma phenotype to assess for type 2 inflammation. Type 2 asthma includes allergic and eosinophilic asthma. Non-type 2 asthma is driven by neutrophils and is associated with smoking and obesity. Type 2 inflammation is diagnosed by elevated eosinophils in the blood or sputum, elevated fractional exhaled nitric oxide, or a need for oral corticosteroid maintenance therapy. Biologic therapy may be considered in patients with severe type 2 inflammatory asthma who continue to have significant symptoms despite adherence to optimal therapy, including high-dose inhaled corticosteroids and a long-acting -agonist.

A 39-year-old female presents to the urgent care clinic on a Saturday evening with fever, cough, diarrhea, and malaise. She is undergoing treatment for breast cancer and her last chemotherapy treatment was 2 weeks ago. On examination her temperature is 38.6°C (101.5°F), her heart rate is 120 beats/min, her blood pressure is 124/68 mm Hg, her respiratory rate is 24/min, and her oxygen saturation is 95% on room air. You order stat laboratory studies, a chest radiograph, a urinalysis, and blood cultures. A CBC reveals a WBC count of 1200/mm3 (N 4800-10,800), 34% neutrophils, 4% bands, and 48% lymphocytes. A COVID-19 rapid antigen test is negative. Which one of the following would be most appropriate at this point? A) No treatment until results are available for the remainder of the laboratory studies, chest radiograph, and urinalysis B) Oral acetaminophen, 1000 mg C) Empiric oral high-dose amoxicillin/clavulanate (Augmentin) D) Empiric intravenous piperacillin/tazobactam (Zosyn)

ANSWER: D Febrile neutropenia is a relatively common complication of chemotherapy. It usually occurs within 6 weeks of a chemotherapy treatment. It is defined as a single oral temperature of 38.5°C (101.3°F) or a sustained temperature of 38°C (100.4°F) for at least 1 hour in patients with an absolute neutrophil count (ANC) <500 cells/mm. This patient's ANC is 456 cells/mm. Such patients should be presumed to have a bacterial infection. For patients who meet the criteria for febrile neutropenia, guidelines recommend administration of empiric intravenous antibiotics within an hour of presentation. Early administration of intravenous antibiotics has been shown to reduce the potential 11% mortality rate of febrile neutropenia (SOR B). Once the remainder of the laboratory results are available, a decision can be made about inpatient versus outpatient treatment in consultation with the patient's oncologist, but initial treatment should not be delayed. The patient may benefit symptomatically from acetaminophen but it is not an urgent consideration. Oral antibiotics have not been shown to be as effective as initial empiric treatment with an intravenous broad-spectrum antibiotic.

A 35-year-old female at 36 weeks gestation presents to your office. She reports that for the past few days she has had itching of her palms and soles that has been quite bothersome. She has tried moisturizer with no improvement. A physical examination is unremarkable with no rashes, erythema, or warmth of the palm and soles. The patient otherwise feels well. Which one of the following would be most appropriate at this time? A) A low-potency corticosteroid cream B) Tacrolimus topical (Protopic) C) Cetirizine (Zyrtec Allergy) D) Bile acid levels E) A BUN/creatinine ratio

ANSWER: D Intrahepatic cholestasis of pregnancy (ICP) presents with pruritus of the palms and soles with or without jaundice along with an elevation in serum bile acid concentrations. ICP, which is most common in the late second and/or third trimester, can cause significant risk to the fetus, including fetal death, and is therefore treated aggressively with ursodeoxycholic acid and often early delivery. Corticosteroid creams, tacrolimus, and cetirizine are not appropriate treatments for ICP. Kidney function tests such as a BUN/creatinine ratio would not be initially appropriate in this case.

A 48-year-old male presents for follow-up of his hypertension. His medications include generic amlodipine, 10 mg; lisinopril (Zestril), 20 mg; and hydrochlorothiazide, 25 mg. His blood pressure today is 156/92 mm Hg. He says that he forgot to take his medications this morning, and reports that he often forgets to take them. He does not have any specific concerns about side effects. Which one of the following would be most likely to increase this patient's medication adherence? A) Taking his medications before bed B) Changing generic amlodipine to branded Norvasc C) Changing generic amlodipine to generic nifedipine D) Replacing lisinopril and hydrochlorothiazide with combination lisinopril/hydrochlorothiazide (Zestoretic) E) Adding metoprolol

ANSWER: D Simplifying medication regimens, including using combination medications to decrease the number of pills a patient must take, has been shown to improve medication adherence in clinical trials (SOR B). Taking antihypertensive medications before bed has not been shown to improve adherence. Prescribing brand name medications increases costs, which may decrease adherence. Unless specific side effects are a concern, changing medications or adding another agent would not be likely to improve adherence.

Which one of the following interventions has been shown to increase retinal screening rates in patients with diabetes mellitus? A) One minute of counseling about the importance of retinal screening at each primary care visit B) Digital reminders sent monthly to patients' cell phones until they complete their retinal screenings C) Conducting an office-wide prize drawing for patients who complete retinal screenings D) Asynchronous teleretinal screening performed at the primary care provider's office E) Sharing office/clinic space with an ophthalmologist

ANSWER: D Telemedicine can be helpful in the management of many chronic conditions, including diabetes mellitus. Medicare and most private insurers pay for telemedicine visits at the same rate as in-person visits. Teleretinal screening performed at the primary care provider's office should be considered in patients with diabetes as a cost-effective option for improving retinopathy screening rates (SOR B). Eyecare specialists can remotely evaluate the retinal photos for timely completion of annual retinopathy screening. Counseling about the importance of retinal screening, digital reminders, office-wide prize drawings, and sharing office space with an ophthalmologist have not been proven to be effective in increasing retinal screening rates in patients with diabetes.

A 53-year-old male undergoes colon cancer screening with multitarget stool DNA testing (Cologuard). The result is negative and you are composing a message to send to his online patient portal to discuss the results. Which one of the following would be the most appropriate screening strategy for this patient? A) Fecal immunochemical testing (FIT) now B) FIT in 1 year C) Colonoscopy now D) Repeat multitarget stool DNA testing in 3 years E) Repeat multitarget stool DNA testing in 5 years

ANSWER: D The recommended interval for colon cancer screening with multitarget stool DNA testing is 3 years at minimum, with the U.S. Preventive Services Task Force recommending an interval of 1-3 years. If the screening is positive, proceeding with a colonoscopy is recommended. Fecal immunochemical testing (FIT) has an annual screening interval if the test is negative, and colonoscopy is recommended if the FIT is positive. Colonoscopy for patients without risk factors should be performed every 10 years, or sooner if indicated by pathology results or risk factors.

A 33-year-old male presents to your office with a 4-week history of a runny and itchy nose, nasal congestion, watery eyes, sneezing, and cough. He reports that he always has similar symptoms this time of year. He has tried taking over-the-counter diphenhydramine (Benadryl Allergy) for the past week but finds it too sedating. He reports that his symptoms are interfering with his quality of life. On examination he has pale, boggy nasal mucosa with clear rhinorrhea. Lung auscultation is normal. Which one of the following is the most appropriate treatment? A) Butterbur herbal supplement B) Oral amoxicillin C) Oral montelukast (Singulair) D) Intranasal fluticasone (Flonase Allergy Relief) E) Intramuscular methylprednisolone

ANSWER: D This patient has signs and symptoms of seasonal allergic rhinitis. He has a history of seasonal symptoms with a predominance of itchy nose, clear rhinorrhea, and watery eyes. The American Academy of Allergy, Asthma & Immunology (AAAA&I) Rhinitis 2020 practice parameter update recommends an intranasal corticosteroid such as fluticasone as initial treatment for seasonal allergic rhinitis (SOR strong, certainty of evidence [COE] high). Due to the lack of available evidence, the AAAA&I cannot make a recommendation for or against the use of herbal treatments such as butterbur or Yu ping feng san. An oral antibiotic such as amoxicillin would be used for suspected bacterial infectious rhinitis, which this patient does not have. The AAAA&I suggests not using the leukotriene receptor antagonist montelukast as initial treatment due to its decreased efficacy when compared to other treatments (SOR conditional, COE very low). Furthermore, the FDA has advised that, due to the risk of serious neuropsychiatric events, montelukast should only be used for treatment of allergic rhinitis when other options are not tolerated or effective. The AAAA&I also advises against the use of depot parenteral corticosteroids such as intramuscular methylprednisolone due to the risks of systemic and local side effects (SOR conditional, COE low).

Which one of the following oral conditions shows the most significant response to oral antivirals? A) Behçet's syndrome B) Hand-foot-and-mouth disease C) Herpangina D) Herpes gingivostomatitis E) Vincent's angina

ANSWER: D Herpes gingivostomatitis is the enanthem associated with a primary herpes simplex virus 1 infection, and it is the only condition listed here that is treated with antivirals such as acyclovir or valacyclovir. Behçet's syndrome is an inflammatory condition presenting with oral and genital aphthous ulcerations. The cause is unknown and it is frequently managed with topical or systemic corticosteroids or colchicine. Hand-foot-and-mouth disease and herpangina are caused by coxsackie or enterovirus and supportive care is most appropriate for both of these. Vincent's angina (also known as trench mouth or necrotizing ulcerative gingivitis) is a bacterial infection of the gingiva associated with poor hygiene. It is treated with systemic antibiotics such as metronidazole or amoxicillin/clavulanate.

An otherwise healthy 29-year-old gravida 2 para 1 at 28 weeks gestation presents to your office with a laceration sustained while doing yard work. After thoroughly cleaning the wound, you decide not to suture it because of the risk of infection. The patient received Tdap during her previous pregnancy 6 years ago and you confirm in her medical records that she completed her primary immunizations as a child. Which one of the following would be most appropriate regarding tetanus prophylaxis? A) No tetanus prophylaxis B) Tetanus immune globulin now C) Td now D) Tdap now E) Tdap at 38 weeks gestation

ANSWER: D This patient needs a tetanus toxoid-containing vaccine for the management of her wound. Since pregnant people should receive a dose of Tdap between 27 and 36 weeks gestation to protect against pertussis, Tdap is the best choice for this patient. Tetanus immune globulin would be appropriate if this patient had not previously completed the primary series or were showing signs of clinical tetanus. Td would be an appropriate option for tetanus prophylaxis in nonpregnant patients who have previously received Tdap. Because this patient requires some form of tetanus prophylaxis at this time, waiting until 38 weeks to administer Tdap is not appropriate.

A 34-year-old gravida 2 para 2 presents for a postpartum examination 6 weeks after an uncomplicated vaginal delivery. Both the mother and infant are doing well. Her only complication during the pregnancy was an abnormal 3-hour glucose tolerance test. She managed her blood glucose with a combination of diet and exercise and delivered at 39 weeks gestation. The patient's vital signs and a physical examination are normal today. Which one of the following should you recommend for this patient based on her history of gestational diabetes? A) No glucose testing today, and initiation of metformin to prevent diabetes B) No glucose testing today, and annual screening with a fasting plasma glucose level starting 1 year after delivery C) No further glucose testing unless she becomes pregnant again D) A 2-hour plasma glucose level using a 75-g oral glucose load

ANSWER: D A 2-hour, 75-g glucose tolerance test should be performed at 4-12 weeks post partum following a pregnancy in which gestational diabetes was diagnosed. This will identify patients who have developed diabetes mellitus, impaired fasting glucose, or impaired glucose tolerance. Women who have a history of gestational diabetes have a sevenfold increased risk of developing type 2 diabetes compared to women without a history of gestational diabetes. This patient should not begin taking metformin because she may not be a candidate for treatment. Testing is required to make the diagnosis of diabetes mellitus, impaired fasting glucose, or impaired glucose tolerance. If a patient who was diagnosed with gestational diabetes tests negative for diabetes mellitus on postpartum screening, fasting glucose levels should still be assessed every 1-3 years regardless of pregnancy status. With the next pregnancy the patient should have early screening with a 1-hour glucose tolerance test at the time the pregnancy is confirmed

Your patient, who is overweight and says she has struggled with bulimia in the past, asks for your advice on strategies to help prevent obesity and eating disorders in her 12-year-old daughter. Which one of the following strategies should be incorporated? A) Implementing a diet of moderate calorie restriction for healthy adolescents and adults 1 week per month B) Having more frequent discussions regarding weight control and healthy eating C) Limiting home-prepared dinners to 1-2 times per week D) Eating meals together as a family at least 7 times per week E) Watching television during mealtimes

ANSWER: D A higher frequency of family meals is associated with improved dietary quality, as evidenced by increased consumption of fruits, vegetables, and grains. Eating meals together as a family on most days or every day is protective against purging and binge eating, as well as frequent dieting, which is a risk factor for both obesity and eating disorders. Parental talk about weight revolving around their children or their own dieting is linked to becoming overweight. Meals that are home-prepared and undistracted are also beneficial to maintaining healthy weight and attitudes toward food.

A 68-year-old patient sees you for treatment of depression. When considering potential adverse effects of antidepressants, which one of the following would be the most appropriate pharmacotherapy for this patient? A) Amitriptyline B) Duloxetine (Cymbalta) C) Nortriptyline (Pamelor) D) Sertraline (Zoloft) E) Venlafaxine (Effexor XR

ANSWER: D A recent review from the Agency for Healthcare Research and Quality (AHRQ) found the frequency of adverse events in older adults taking SSRIs such as sertraline and escitalopram was similar to placebo (SOR B). SSRIs also have lower discontinuation rates than tricyclic antidepressants such as amitriptyline or nortriptyline during treatment of up to 12 weeks (SOR B). Evidence suggests that SNRIs including duloxetine and venlafaxine cause more adverse events and greater discontinuation of therapy during treatment of up to 12 weeks when compared to placebo (SOR B). A randomized, controlled trial involving duloxetine demonstrated an increased risk of treatment withdrawal due to adverse events and an increased risk of falls over 12-24 weeks. Venlafaxine was compared to no antidepressant use in a large cohort study that had a median treatment period of 364 days and was associated with an increased risk of falls, fractures, and mortality

A 47-year-old female sees you for a health maintenance visit. During the course of your discussion she discloses a long history of significant alcohol intake that has impaired her work and personal life, meeting criteria for moderate to severe alcohol use disorder. She began working with a therapist to address this issue several months ago and reports that her last drink was 26 days ago. Her health is otherwise good and laboratory studies reveal normal liver and renal function. Which one of the following medications would help reduce the risk of relapse? A) Buspirone B) Duloxetine (Cymbalta) C) Fluoxetine (Prozac) D) Naltrexone E) Quetiapine (Seroquel)

ANSWER: D Alcohol use disorder (AUD) is common in the United States but remains undertreated, especially with pharmacotherapy. FDA-approved therapies include naltrexone, acamprosate, and disulfiram. Non-FDA-approved therapies with evidence of benefit include baclofen and topiramate. AUD commonly coexists with other psychiatric conditions and in these situations treating those comorbid conditions is critical. For patients with coexisting depression or anxiety, buspirone, duloxetine, or fluoxetine could be considered in addition to one of the other medications listed to address AUD. Quetiapine, which is an antipsychotic, has no role in the treatment of AUD.

A 60-year-old male with moderate COPD presents to your office with shortness of breath and a cough with increased sputum volume. After appropriate evaluation, you diagnose an acute COPD exacerbation. According to Global Initiative for Chronic Obstructive Lung Disease (GOLD) guidelines, which one of the following additional factors would provide the strongest indication for treatment with antibiotics? A) A decline in oxygen saturation from baseline B) Diffuse wheezing on lung auscultation C) A fever D) Increased sputum purulence E) Leukocytosis

ANSWER: D COPD exacerbations, when caused by an infectious agent, may be bacterial or viral. The Global Initiative for Chronic Obstructive Lung Disease (GOLD) guidelines support the use of antibiotics in patients with an acute COPD exacerbation with the three cardinal symptoms of increased dyspnea, increased sputum volume, and increased sputum purulence; in patients with increased sputum purulence and one of the other cardinal symptoms; and in patients who require invasive or noninvasive mechanical ventilation. Hypoxemia and fever, although often seen in the setting of COPD exacerbations, do not provide as strong an indication for treatment with antibiotics. Diffuse wheezing is a hallmark examination finding that is present in most COPD exacerbations, regardless of the underlying cause. Leukocytosis is a relatively nonspecific marker for acute inflammation and may be seen with either viral or bacterial etiologies. Serum biomarkers such as C-reactive protein and procalcitonin have yielded controversial and conflicting evidence in guiding antibiotic therapy

A 48-year-old male with schizophrenia presents for a new patient visit after recently relocating to your area. He has been stable on clozapine (Clozaril) for the past 15 years and asks you to refill his prescription. He has been told the earliest available appointment with a local psychiatric provider is in 3 months. Under the Clozapine Risk Evaluation and Mitigation Strategy (REMS) program, which one of the following is required to prescribe clozapine to this patient? A) A signed patient consent form B) Serum clozapine levels C) Creatinine levels D) Neutrophil counts E) Specialty training in psychiatry

ANSWER: D Clozapine is a highly effective antipsychotic medication, but its use is limited due to its association with severe drug-induced neutropenia, also referred to as agranulocytosis. Patients must be enrolled in the national Clozapine Risk Evaluation and Mitigation Strategy (REMS) program to receive treatment, and all prescribers and pharmacies must be certified by this program in order to dispense clozapine. The patient's absolute neutrophil count must be submitted at least every 30 days, or more frequently as determined by stability in treatment. A signed patient consent form should be obtained but is not a part of the Clozapine REMS monitoring system. Monitoring serum clozapine levels and creatinine levels may be appropriate but is not part of the Clozapine REMS program. Family physicians can prescribe clozapine if registered and certified in the Clozapine REMS program, which includes passing a brief knowledge assessment, but specialty training in psychiatry is not required.

A 62-year-old male sees you for a routine health maintenance examination, and you note that he has not yet received the recombinant zoster vaccine (Shingrix). Which one of the following approaches is most likely to result in your patient accepting this immunization? A) "Are there any immunizations you would like today?" B) "You have been a bit negligent on obtaining your shingles vaccine. May we give that to you?" C) "You have not yet received your shingles vaccine. May we give that to you before you leave?" D) "You have not yet received your shingles vaccine. We are preparing to administer that today. Do you have any questions?"

ANSWER: D Employing a presumptive approach rather than a participatory approach significantly increases the likelihood that a patient, parent, or guardian will accept a recommended vaccine. This strategy implies that accepting the immunization is the usual or normal choice. The correct option in this scenario presumes the patient will accept the immunization, while the remainder of the options ask if they will.

A 4-year-old male is brought to your office by his parents because of a 2-day history of cough and a runny nose, but no fever. The child's symptoms are not progressing. The patient has a history of wheezing when he has mild respiratory infections. The only findings on examination are yellow nasal discharge and mild wheezing. The appropriate management with the LEAST amount of risk would be treatment for 10 days with: A) amoxicillin B) montelukast (Singulair) C) an antihistamine decongestant D) an inhaled corticosteroid E) an oral corticosteroid

ANSWER: D For children up to 4 years of age who only have wheezing with respiratory infections, using an inhaled corticosteroid (IC) daily when a respiratory infection develops reduces exacerbations and the use of systemic corticosteroid therapy. It is uncertain if ICs affect growth, but they would be less likely to do so than systemic corticosteroids. Antibiotic therapy should be reserved for bacterial infections. Montelukast is indicated for the prevention of asthma and allergic rhinitis. The use of antihistamine decongestant preparations in children is not recommended due to potential side effects and minimal benefit.

A 69-year-old male with a history of diabetes mellitus presents to your clinic with concerns of mild vision problems. His brother lost his vision due to glaucoma and encouraged the patient to seek care. Which one of the following is most consistent with the typical vision changes of glaucoma? A) Central vision loss with peripheral sparing B) Halos and decreased night vision C) Intermittent complete blackening of the visual field D) Patchy peripheral vision blurring E) Sudden scattered floaters

ANSWER: D Glaucoma and other common eye conditions cause a range of visual disturbances. Glaucoma is typically associated with blurring of peripheral vision as elevated pressure in the eye pushes on the periphery of the ophthalmic nerve. Central vision loss with peripheral sparing is classically seen with macular degeneration. Halos and decreased night vision are classic problems for patients with cataracts. Intermittent complete blackening of the visual field may be seen with ischemia associated with stroke or temporal arteritis. Sudden scattered floaters should raise concern for retinal detachment.

A 57-year-old male who uses tobacco presents with cough and dyspnea. His symptoms were previously controlled with an albuterol (Proventil, Ventolin) inhaler once or twice a month. After a 3-week trial of a tiotropium (Spiriva) inhaler his symptoms are better, but he is still having frequent episodes of coughing and dyspnea. He has been smoking 1-2 packs of cigarettes a day since age 13 and is not interested in quitting. On examination he is afebrile, his vital signs are stable, and his oxygen saturation is 95% on room air. His lung sounds are diminished, and the remainder of the examination is unremarkable. His in-office peak flow is 300 L/min. You suspect he has moderate COPD and recommend pulmonary function tests but he declines. In addition to continuing tiotropium, which one of the following medications would you recommend adding to his current regimen? A) An oral antibiotic B) An oral corticosteroid C) An inhaled corticosteroid D) An inhaled long-acting β-agonist E) A nebulized short-acting β-agonis

ANSWER: D Guidelines from the Global Initiative for Chronic Obstructive Lung Disease (GOLD), the National Initiative for Health and Care Excellence, and the American College of Chest Physicians all recommend that in addition to smoking cessation, COPD should be treated initially with either a long-acting beta-agonist (LABA) or a long-acting muscarinic antagonist (LAMA). If symptoms persist with either of those inhaled medications then combination therapy should be initiated. An inhaled corticosteroid (ICS) can be added to the LABA/LAMA regimen for triple therapy if symptoms continue. Long-term use of an ICS as monotherapy is not recommended due to a slight increase in the incidence of pneumonia.

A 31-year-old nulligravida presents to your office with an inability to conceive for the past 12 months. She reports irregular menses for the past 2 years. Her medical history is significant for Hashimoto thyroiditis that is currently controlled. A urine pregnancy test is negative. On examination you note vaginal dryness and labial atrophy. You suspect primary ovarian insufficiency. Which one of the following combinations of FSH and LH levels is consistent with this diagnosis? A) Normal FSH and normal LH B) Low FSH and low LH C) Low FSH and elevated LH D) Elevated FSH and elevated LH

ANSWER: D In a female younger than 40 years of age, elevated FSH and LH levels indicate primary ovarian insufficiency. Ovarian insufficiency leads to low estrogen levels, which stimulate increased production of FSH in the pituitary in a feedback loop. In this scenario, LH levels are high but do not rise as much as FSH levels. Normal FSH and LH levels may indicate an outflow tract obstruction. Low FSH and LH levels indicate that the hypothalamic-pituitary axis is suppressed, as in the female athlete triad when there is an excess of energy expenditure compared to intake. Low FSH and elevated LH levels may be detected immediately prior to ovulation as part of a normal cyclical pattern.

A 62-year-old male presents with a 2-day history of a painful abscess in his perianal area. He has a history of well controlled type 2 diabetes and hypertension and he currently takes metformin and lisinopril (Zestril). He also has a history of recurrent skin abscesses, which have responded well to oral sulfamethoxazole/trimethoprim (Bactrim). He has occasional chills but has not had a fever. On examination his vital signs are normal and you note the presence of a 2.5×2.5-cm perianal abscess. A point-of-care glucose level is 172 mg/dL and the results of a CBC are pending. Which one of the following would be the most appropriate next step? A) Continue current management and follow up in 48 hours B) Perform incision and drainage, obtain a culture, and start sulfamethoxazole/trimethoprim only C) Perform incision and drainage, obtain a culture, and start oral linezolid (Zyvox) D) Perform incision and drainage, obtain a culture, and start sulfamethoxazole/trimethoprim plus amoxicillin/clavulanate (Augmentin)

ANSWER: D Incision and drainage of an abscess along with MRSA antibiotic coverage is recommended for all abscesses greater than 2×2 cm. Incision and drainage of an abscess is almost always indicated and is a cornerstone of treatment. A wound culture with antibiotic sensitivity must be obtained in all cases to guide therapy. MRSA is the most common causative pathogen, so an antibiotic that provides coverage against MRSA, such as sulfamethoxazole/trimethoprim, doxycycline, or clindamycin, should be used as an empiric first-line agent for treatment of a skin abscess, pending culture results. Additional anaerobic coverage is recommended when an abscess is located in the perirectal area or when an abscess occurs in an area where tissue ischemia is likely. Continuing current management with follow-up in 48 hours is not recommended due to the potential for expansion of the abscess in patients with diabetes mellitus (SOR A). When an abscess is less than 2×2 cm, incision and drainage is often the only recommended intervention, and treatment with antibiotics is typically not indicated in such cases. Most patients with uncomplicated skin abscesses can be managed in the outpatient setting. The presence of certain host factors such as poorly controlled type 2 diabetes, surrounding cellulitis, rapid progression, signs and symptoms of systemic illness, associated comorbidities or immunosuppression, extremes of age, an abscess in an area difficult to drain, and underlying tissue ischemia/gangrene are indications for hospitalization and parenteral antibiotics. Linezolid is reserved for patients who are allergic or intolerant to commonly used anti-MRSA antibiotics. Because linezolid has limited anaerobic coverage, it is not recommended when anaerobic infection is likely.

According to the Ottawa knee rule, which one of the following factors may warrant radiography in a patient with an acute knee injury? A) Age <30 B) Injury from a fall or blunt trauma C) A twisting mechanism of injury D) Inability to flex the knee to 90° E) The presence of a joint effusion within 24 hours of the injury

ANSWER: D Knee injuries are an extremely common cause for primary care visits and knowing which injuries require radiography can ensure high-value care. The Ottawa knee rule has been repeatedly validated with a sensitivity of 98.5%-100% and can decrease unnecessary imaging. The major criteria of the Ottawa knee rule are age >55, the inability to bear weight for four steps both immediately after the injury and at the time of the examination, the inability to flex the knee to 90°, tenderness over the head of the fibula, and isolated tenderness to the patella without other bony tenderness. If a patient meets any of these criteria, radiography of the knee may be indicated.

Your patient lives in an unincorporated community located about 30 miles from the nearest town and does not have access to municipal water. She reports her family gets their water from a well and that all the members of her family have had intermittent diarrheal illnesses for the past few months. Currently everyone is feeling better. There is no relevant travel history. Which one of the following would you advise? A) Discontinuing use of the well water for any purpose until it is tested B) Routinely testing the well water every 2 years C) Pouring a gallon of bleach into the well D) Drinking, cooking, and bathing with boiled or bottled water until the well is uncontaminated E) A stool culture and testing for ova and parasites for all family members

ANSWER: D Many homes rely on well water, particularly in rural areas. Unlike water from municipal supplies, well water is not tested routinely by government agencies, and it is generally the responsibility of the property owner to ensure the safety of the water. If there is cause for concern in regard to the well, appropriate testing should be performed, in this case for fecal coliforms. In most locations, the county health department or other government agency will test well water on request. The water should be retested following treatment, and not used for drinking, cooking, or bathing until it is known to be safe. Water can still be used for laundry, yard maintenance, or any purpose where ingestion is unlikely. Wells should be tested yearly, generally in the spring, and following flooding or other environmental concerns such as nearby excavation or dumping. While the treatment of bacterial contamination does require bleach in most cases, the dose is calculated by characteristics of the individual well, and most experts advise that treatment be done by a well drilling and maintenance company. Since all of this patient's family members are currently asymptomatic, there is no need to obtain stool samples.

A 68-year-old male with type 2 diabetes with peripheral neuropathy presents for routine follow-up. On examination of his feet, you note that the majority of his toenails are thickened and discolored. The great toenails lift easily from the nailbeds. A clipping from one nail is sent for KOH evaluation with positive results. Of the following, which one is the most effective treatment for this condition? A) Topical efinaconazole 10% (Jublia) daily for 48 weeks B) Topical tea tree oil daily until the nails grow out C) Oral fluconazole (Diflucan) once weekly until the nails grow out D) Oral terbinafine daily for 12 weeks E) Fractional carbon dioxide laser therapy

ANSWER: D More than 10% of U.S. adults have onychomycosis, and age over 60 is an important risk factor. Patients with suspected onychomycosis should undergo testing to confirm the infection, preferably with a KOH preparation. The American Academy of Dermatology's recommendations in the Choosing Wisely initiative support testing before treatment. The most efficacious therapy for onychomycosis of any severity is an oral antifungal. Terbinafine is the most effective oral antifungal and should be first-line therapy for most patients (SOR B). Oral fluconazole and itraconazole are also beneficial but exhibit lower cure rates than terbinafine. Oral antifungals are contraindicated in patients with chronic liver disease; transaminase levels should be checked before starting therapy. Topical antifungals are appropriate for mild onychomycosis but are not as effective as the oral forms. Efinaconazole is the most effective of the topical antifungals but is quite expensive. Tea tree oil and other topical treatments such as Vicks VapoRub, oregano, and vitamin E have shown antifungal activity, but larger studies are needed to validate their effectiveness. Laser therapy for onychomycosis is approved by the FDA but there is a dearth of evidence as to its effectiveness.

Which one of the following classes of diabetes medications is most associated with hypoglycemia? A) Biguanides B) DPP-4 inhibitors C) SGLT2 inhibitors D) Sulfonylureas E) Thiazolidinediones

ANSWER: D Multiple classes of diabetes medications are used to address the pathways that lead to hyperglycemia, and it is important to select medication classes that reduce the risk of hypoglycemia while improving long-term outcomes. Hypoglycemia is associated with cardiovascular disease and all-cause mortality. Sulfonylureas, such as glipizide, glyburide, and glimepiride, commonly cause hypoglycemia as an adverse effect and require glucose monitoring when used. Biguanides most commonly cause diarrhea, vomiting, and other gastrointestinal symptoms. In high-risk patients such as those with heart failure, sepsis, or impaired kidney function, biguanides can also result in lactic acidosis. The only biguanide currently available is metformin. The most common adverse effects of DPP-4 inhibitors, which include saxagliptin, sitagliptin, linagliptin, alogliptin, are headache, nasopharyngitis, infections of the urinary tract or upper respiratory tract, and elevated liver enzymes. SGLT2 inhibitors, such as canagliflozin, dapagliflozin, and empagliflozin, can cause adverse effects such as urinary tract infections, candidiasis, dehydration, and hypovolemia. Only two thiazolidinediones, pioglitazone and rosiglitazone, are available in the United States. Their adverse effects include weight gain, salt retention, edema, and, for some patients, cardiovascular complications. Pioglitazone in particular is contraindicated in patients with heart failure, hemodynamic instability, and hepatic dysfunction. Thiazolidinediones may also increase the risk of bone fractures with long-term use.

Which one of the following is true regarding acute gastroenteritis in children? A) Handwashing and general hygiene alone can prevent rotavirus infection B) Daily probiotics are recommended C) The majority of cases are caused by bacteria D) Oral rehydration is as effective as intravenous rehydration in mild to moderate dehydration E) Promethazine is the antiemetic of choice in patients with moderate dehydration and nausea

ANSWER: D Oral rehydration therapy is as effective as intravenous rehydration in preventing hospitalizations and return emergency department visits in children with mild to moderate dehydration from acute gastroenteritis. Rotavirus infection cannot be prevented by handwashing and hygiene alone, and infants should receive the rotavirus vaccine to reduce the risk of rotavirus infection and associated complications. The benefit of probiotics for treatment of acute gastroenteritis is not yet clear. The majority of acute gastroenteritis infections in children are caused by viruses, not bacteria. Ondansetron is the antiemetic of choice for children, as older antiemetics, including promethazine and metoclopramide, have higher rates of adverse reactions.

A 48-year-old female sees you because she recently felt flutters in her chest while watching television. These were not associated with exertion. She has no significant past medical history and she does not take any medications or use illicit substances. On examination you hear a regular rhythm with occasional premature beats. An EKG reveals multiple unifocal PVCs. You order 48-hour Holter monitoring, which shows a 15% PVC burden that is unifocal with no episodes of ventricular tachycardia. Which one of the following would be most appropriate at this time? A) No further evaluation and reassurance that her palpitations are benign B) Initiation of a β-blocker C) Initiation of flecainide D) Echocardiography E) Left heart catheterization

ANSWER: D Patients found to have a PVC burden >10% are at risk for PVC-induced dilated cardiomyopathy (PVC-CM). In fact, a PVC burden of 16% has a sensitivity of almost 80% for PVC-CM. Echocardiography should be performed in patients with a PVC burden >10%. Treatment with anti-arrhythmic drugs or radiofrequency ablation reverses cardiomyopathy and its associated increase in morbidity, mortality, and health care spending. Further evaluation for ischemic heart disease may be performed if the patient has risk factors for ischemia. Symptomatic palpitations may be treated with β-blockers or calcium channel blockers, even in patients with lower PVC burdens and no cardiomyopathy. Left heart catheterization would not be appropriate.

A 20-year-old female comes to your office for routine follow-up after recently finishing neck irradiation treatment for Hodgkin's lymphoma. Her past medical history is otherwise significant for allergic rhinitis and GERD. She feels generally well after treatment. A physical examination is unremarkable. Which one of the following should be performed to monitor for complications from radiation? A) No routine follow-up surveillance B) Parathyroid hormone levels C) Swallow studies D) Carotid artery ultrasonography E) Neck CT

ANSWER: D Patients who have been treated with neck irradiation for lymphoma require follow-up surveillance with carotid artery ultrasonography every 10 years. There is evidence that asymptomatic carotid artery disease is more common in patients who have been treated with radiation for Hodgkin's lymphoma compared to the general population. The risk of thyroid cancer is increased by neck irradiation. Hypothyroidism is also a common complication from neck irradiation, and a TSH level should be measured annually for up to 5 years. For patients who have completed treatment for lymphoma, additional surveillance laboratory studies include a comprehensive metabolic panel, fasting glucose level, and CBC. Patients with a history of chest or axillary irradiation should receive annual mammography screening starting 8-10 years after treatment or at age 40, whichever comes first. Breast MRI may be appropriate for patients who received chest irradiation treatment between ages 10 and 30. Parathyroid hormone levels, swallow studies, and neck CT would not be appropriate for this patient as surveillance to monitor for complications from radiation.

A 35-year-old female with rheumatoid arthritis currently being treated with adalimumab (Humira) injections sees you for evaluation after developing a red, swollen, warm, and painful right knee. Arthrocentesis is performed, and the synovial fluid analysis is concerning for septic arthritis. Which one of the following organisms is the most likely cause of her infection? A) Candida albicans B) Escherichia coli C) Mycobacterium tuberculosis D) Staphylococcus aureus E) Streptococcus pyogene

ANSWER: D Patients with rheumatoid arthritis being treated with anti-tumor necrosis factor therapy are at increased risk for septic arthritis. The most common cause of septic arthritis in adults is Staphylococcus aureus, followed by Streptococcus species. Escherichia coli causes about a fourth of the cases in the elderly. Fungal and mycobacterial causes such as Candida albicans or Mycobacterium tuberculosis are less common but must be considered in immunocompromised patients.

A patient is brought to the emergency department by his wife due to acute anxiety, jittery movements, confusion, vomiting, and fever, all of which started without warning a few hours ago. The wife reports that he has had a cough and upper respiratory symptoms recently, and he took an over-the-counter medication with dextromethorphan this morning. His usual medications include fluoxetine (Prozac), 30 mg daily for depression, and methylphenidate (Metadate CD), 50 mg daily for attention-deficit/hyperactivity disorder. She checked his medication bottles and does not think he has taken extra doses. His vital signs include a blood pressure of 160/95 mm Hg, a heart rate of 116 beats/min, a respiratory rate of 25/min, a temperature of 38.5°C (101.3°F), and an oxygen saturation of 98% on room air. A physical examination is remarkable for restlessness, anxiety, diaphoresis, and inducible clonus most prominent in the lower extremities. His lungs are clear and his neck is supple. He is alert and oriented to self only. Laboratory studies reveal a WBC count of 14,000/mm3 (N 4500-11,000) and a serum bicarbonate level of 20 mEq/L (N 23-30), and a urine drug screen is positive for amphetamines only. A COVID-19 polymerase chain reaction test is negative. The most likely cause of his symptoms is: A) an overdose of methylphenidate B) an infectious process C) malignant hyperthermia D) serotonin syndrome

ANSWER: D Symptoms of serotonin syndrome range from mild to life-threatening and typically appear minutes to hours after ingestion of serotonergic medications. SSRIs are the most commonly associated class of medication due to their widespread use. The Hunter Serotonin Toxicity Criteria are the most commonly used diagnostic tool. This patient has a history of serotonergic medication use, signs of inducible clonus, agitation, and diaphoresis, as well as hyperthermia. It is likely that the addition of dextromethorphan precipitated this episode. This patient's history does not suggest an overdose of methylphenidate, and there is little evidence in this scenario for a serious infectious process. Malignant hyperthermia generally appears over a longer period of time and does not typically induce clonus. There are few, if any, choices for medication therapy of concomitant attention-deficit/hyperactivity disorder and depression that do not increase the risk of serotonin syndrome, so patients on these regimens should be educated about the symptoms of serotonin syndrome and common causative agents.

A 45-year-old male presents for follow-up of a recent positive HIV test. He has not had any symptoms. An initial laboratory evaluation is significant for the following: HIV viral load: 124,000 copies/mL CD4 lymphocyte count: 289 cells/μL Hepatitis C antibody: negative Anti-HBs: positive Anti-HBc: positive HBsAg: negative Renal function is normal. He has an upcoming appointment with the comprehensive HIV clinic to initiate antiretroviral therapy. Which one of the following would be appropriate to recommend today? A) Prophylactic emtricitabine/tenofovir (Truvada) B) Prophylactic sulfamethoxazole/trimethoprim (Bactrim) C) Hepatitis B vaccine D) Herpes zoster vaccine (Shingrix) E) Meningococcal B (MenB) vaccine

ANSWER: D The CDC's Advisory Committee on Immunization Practices updated its recommendations in 2022 to include a two-dose series of recombinant zoster vaccine for all adults age 19 and older with HIV. Vaccination against meningococcal bacteria A, C, W, and Y (MenACWY) is also recommended, and meningococcal B (MenB) vaccination is only recommended based on the presence of other risk factors, including asplenia, complement deficiency, treatment with complement inhibitors, or risk due to outbreaks. Prophylactic emtricitabine/tenofovir is approved for pre- and postexposure prophylaxis of HIV, but would not be used alone in the care of patients with established HIV. Pneumocystis jirovecii prophylaxis, most commonly with sulfamethoxazole/trimethoprim, is recommended in patients with CD4 lymphocyte counts <200 cells/ L. Hepatitis B vaccine is recommended but would not be necessary for patients such as this one with natural immunity or confirmed immunity from vaccination.

A 39-year-old gravida 3 para 3 comes to your office for a routine health maintenance visit. She reports gradual leaking of urine over the past year. It mostly occurs when she does strength training at the gym and has become so bothersome that she has limited her exercise. She also notices leaking when coughing, sneezing, and picking up her young children. Her BMI is 27 kg/m2 . A pelvic examination is notable for a normal urethral body and thick, pink vulvar tissue. The vaginal vault is without prolapse. You ask her to cough with a full bladder and note leakage of urine. A urinalysis is normal. You counsel her on appropriate fluid intake, timed voiding, the reduction of caffeinated and carbonated beverages, regular moderate physical activity, and weight loss. In addition to these behavioral modifications, which one of the following is the most appropriate intervention at this time? A) An oral antimuscarinic agent B) Intravaginal estrogen C) OnabotulinumtoxinA (Botox) D) Pelvic floor muscle training E) Urethropexy

ANSWER: D The cough stress test confirms that this patient is experiencing chronic stress urinary incontinence. In addition to other behavioral modifications such as appropriate fluid intake, timed voiding, the reduction of caffeinated and carbonated beverages, regular moderate physical activity, and weight loss, pelvic floor muscle training is a first-line treatment for stress urinary incontinence. Surgical intervention with urethropexy or sling procedures can be considered if conservative treatment with behavioral modification and pelvic floor muscle training fails. There are no FDA-approved oral medications for the treatment of stress incontinence. Oral antimuscarinic agents and onabotulinumtoxinA are approved for use in urge incontinence. Intravaginal estrogen can be used to treat underlying vaginal and vulvar atrophy that can contribute to urinary incontinence, but it has not been approved by the FDA for the treatment of urinary incontinence. This patient does not have concurrent atrophic vaginitis so she would not be an appropriate candidate for intravaginal estrogen.

A 70-year-old female who is an established patient at your practice calls you late on a Saturday afternoon. Earlier in the day she misjudged the location of a bench at a neighbor's house and sat down hard on the porch floor. She felt immediate pain in her back. She went home and took naproxen, 440 mg, and sustained-release acetaminophen, 1300 mg, 3 hours ago. She still describes her pain as unbearable, rating it as 10 on a scale of 10. You agree to meet her in the emergency department, where you confirm an acute T12 vertebral compression fracture. Of the following, the most appropriate treatment option for this patient's acute pain is a short course of: A) prescription-strength NSAIDs B) methadone C) transdermal fentanyl D) immediate-release oxycodone (Roxicodone)

ANSWER: D The most appropriate treatment for this patient's acute pain following a T12 vertebral compression fracture is round-the-clock class II narcotics. Subcutaneous calcitonin can also be useful for relieving pain from vertebral fractures. NSAIDs and acetaminophen are usually insufficient during the acute phase of a vertebral compression fracture, and this patient has already tried these. Methadone or transdermal fentanyl can be used, but plasma levels of methadone may take 5-7 days to stabilize and fentanyl takes 24-48 hours to take effect.

A 45-year-old male presents to the urgent care clinic with a 2-hour history of central chest pain that began at rest with associated shortness of breath. In addition, he has had a mild dry cough and rhinorrhea for a few days but no fever. He has not had any nausea, dizziness, or diaphoresis, and the chest pain does not radiate. He has no past medical history, takes no medications, consumes 4-6 alcoholic drinks per night, and does not smoke. On examination the patient has a temperature of 37.0°C (98.6°F), a blood pressure of 150/100 mm Hg, a heart rate of 118 beats/min, a respiratory rate of 14/min, and an oxygen saturation of 98% on room air. The patient appears well, and an HEENT examination reveals no jugular vein distention. A cardiovascular examination is significant for tachycardia without murmur. There is no chest wall tenderness to palpation. The lungs reveal decreased breath sounds on the right compared to the left, and there are no crackles or wheezes. EKG shows sinus tachycardia. A chest radiograph is shown below. What is the most likely etiology of his chest pain?

ANSWER: D This chest radiograph is consistent with a large right pneumothorax and complete lung collapse. In addition, there is a leftward mediastinal shift that raises the concern for a tension pneumothorax. The chest radiograph is not consistent with aspiration pneumonia or community-acquired pneumonia. While a non-ST-elevation myocardial infarction or pulmonary embolus could have a similar presentation, this abnormal chest radiograph points to pneumothorax as the most likely diagnosis

A 26-year-old recreational baseball player presents with recurrent right shoulder pain that tends to gradually worsen during play and is relieved by rest. His other daily activities have not been affected and he has no nighttime pain. Examination of the right shoulder reveals a normal appearance, no tenderness to palpation, normal abduction strength, and a positive painful arc at 90°. The drop-arm rotator cuff test is negative, the Hawkins impingement sign is mildly positive, the empty-can supraspinatus test is moderately positive, and the Gerber liftoff test is negative. Radiographs of the right shoulder are normal. Which one of the following would be appropriate at this time to provide long-term pain relief? A) Complete shoulder rest with temporary use of a shoulder sling B) Recommending that he permanently stop playing baseball C) A subacromial corticosteroid injection D) Physical therapy E) Referral for arthroscopic surgery

ANSWER: D This patient has shoulder impingement syndrome (with a positive Hawkins impingement sign) and evidence of supraspinatus tendinopathy (with a positive empty-can rotator cuff test). However, the negative drop-arm rotator cuff test is evidence against a complete rotator cuff tear with a negative drop-arm rotator cuff test, and the absence of night pain supports this. Physical therapy, along with pain control using NSAIDs, acetaminophen, or short-term opiate medication, would be most appropriate as initial therapy. Complete shoulder rest is inappropriate since his daily activities are not aggravating the problem, and cessation of play is not necessary since other treatment options are available. A subacromial corticosteroid injection, while commonly done and likely to provide short-term pain relief, is unlikely to provide long-term improvement in pain and function. Surgery is a potential option if other treatments fail and a significant tear is proven, but is not preferable as an initial treatment.

A 17-year-old female sees you for a preparticipation evaluation for cross country running. She has not had any falls or other injuries and estimates that she runs 40 miles per week. Menarche occurred at age 12 but over the past year her periods have become irregular, with her last menses occurring about 4 months prior to presentation. She is not sexually active and a urine pregnancy test is negative. She and her parents tell you that she eats a healthy diet, although she does report an inadvertent weight loss of 15 lb as she has increased her running mileage over the past year. She takes a multivitamin and occasional acetaminophen for pain but does not take any chronic medications. A physical examination reveals a thin female who does not show any signs of acute distress. Her vital signs include a weight of 52 kg (115 lb), a height of 173 cm (68 in), a BMI of 18 kg/m2 , a heart rate of 50 beats/min, and a blood pressure of 85/44 mm Hg. Which one of the following is the most likely underlying cause of her condition? A) Anemia B) Anorexia nervosa C) Low circulating estrogen levels D) Low energy availability relative to needs E) Vitamin D deficiency

ANSWER: D This patient has the female athlete triad, a syndrome characterized by low energy availability relative to needs, disordered menses (delayed menarche, oligomenorrhea, or secondary amenorrhea), and decreased bone mineral density. This patient exhibits at least two components of the triad, although only one is required for diagnosis. Low energy availability relative to needs can be related to an eating disorder or to exercising beyond caloric supply. This leads to functional hypothalamic amenorrhea, which results in low circulating estrogen levels and then reduced bone mineral density. Anemia would be secondary to the low energy availability rather than the cause of this spectrum of issues. This patient does not have a history consistent with anorexia nervosa. Vitamin D deficiency would not cause the menstrual irregularities she has noted.

A 63-year-old female presents with a 3-week history of vulvar itching and burning. The patient has not had any vaginal bleeding since her last period at age 51. An examination reveals thin, dry labia minora and sparse pubic hair. The vaginal mucosa is smooth and pale with a frothy yellow discharge. A point-of-care urinalysis shows moderate leukocytes and trace protein. Microscopy of vaginal secretions reveals motile flagellates. Which one of the following medications is most likely to resolve this patient's symptoms? A) Estradiol vaginal cream, 2 g vaginally daily B) Fluconazole (Diflucan), 150 mg as a single dose C) Doxycycline, 100 mg twice daily for 7 days D) Metronidazole, 500 mg twice daily for 7 days E) Sulfamethoxazole/trimethoprim (Bactrim), 800/160 mg twice daily for 7 days

ANSWER: D This patient presents with symptoms suggestive of vaginitis, which can be caused by vaginal infections, atrophy, and irritation from pads, panty liners, soaps, and perfumes. Atrophic vaginitis due to lack of estrogen is common in postmenopausal females. The condition responds to estrogen therapy, but infectious causes must first be ruled out. The presence of motile flagellates confirms the diagnosis of Trichomonas vaginitis, which would be treated with oral metronidazole, either as a one-time 2-g dose or 500 mg twice daily for 7 days. Estradiol, fluconazole, doxycycline, and sulfamethoxazole/trimethoprim are not used for the treatment of Trichomonas.

Which one of the following is the most common cause of prerenal acute kidney injury in the intensive-care setting? A) ACE inhibitor use B) NSAID use C) Membranoproliferative glomerulonephritis D) Polyarteritis nodosa E) Sepsis

ANSWER: E Acute kidney injury (AKI) is defined by a rapid decline in glomerular filtration rate (GFR) and an increase in metabolic waste products. It is associated with an increased risk of cardiovascular events, progression to chronic kidney disease, and mortality. AKI is categorized as prerenal, intrinsic renal, and postrenal. Diagnosing the underlying cause is vital to successful management. Management includes determining volume status, treating acute volume changes with diuretics and fluid resuscitation, adjusting medications according to renal function, and discontinuing nephrotoxic medications. Prerenal AKI is caused by a depletion of intravascular volume, which leads to decreased renal perfusion and GFR. In the intensive-care setting, sepsis is the most common cause of prerenal AKI. Angiotensin receptor blockers, ACE inhibitors, and NSAIDs lower renal perfusion, causing the kidneys to activate compensatory mechanisms to maintain the GFR. For those with chronic kidney disease, this increases the risk for AKI. Membranoproliferative glomerulonephritis and polyarteritis nodosa are intrinsic renal causes for AKI. Postrenal causes include lower and upper urinary tract disorders such as infections, carcinoma, and nephrolithiasis. Systemic postrenal causes include diabetes mellitus, stroke, and multiple sclerosis.

A 77-year-old female presents to your office as a new patient. She recently moved from New York to Florida to live with her daughter, after her partner died 2 years ago. Her past medical history is significant for hypertension, hypothyroidism, osteoarthritis, and cachexia. Her current medications, which were prescribed by her previous primary care physician, include the following: Acetaminophen Hydrochlorothiazide Levothyroxine (Synthroid) Losartan (Cozaar) Megestrol (Megace) A multivitamin Her vital signs are unremarkable except for a 7% weight loss in the past year. A recent TSH level was within the normal range. Which one of the following medications should be discontinued? A) Acetaminophen B) Hydrochlorothiazide C) Levothyroxine D) Losartan E) Megestrol

ANSWER: E Although it is considered an appetite stimulant, megestrol is not recommended in older adults due to potential harmful side effects and a lack of evidence supporting improved outcomes for the treatment of cachexia in the geriatric population, according to the American Geriatrics Society's Choosing Wisely recommendation. Acetaminophen is not considered a contributor to weight loss. Though loop diuretics, spironolactone, ACE inhibitors, calcium channel blockers, and propranolol may contribute to weight loss due to their adverse effects, hydrochlorothiazide and angiotensin receptor blockers such as losartan are less likely to contribute to weight loss and the patient's hypertension is currently controlled on this regimen. The patient's hypothyroidism is currently stable, so changes to the levothyroxine dosage are unnecessary.

A 26-year-old male diagnosed with coccidioidomycosis (valley fever) develops a rash on the extensor surfaces of his lower legs consisting of painful, subcutaneous, nonulcerated, erythematous nodules. This rash is consistent with which one of the following? A) Erythema ab igne B) Erythema infectiosum C) Erythema migrans D) Erythema multiforme E) Erythema nodosum

ANSWER: E Erythema nodosum, a panniculitis that typically affects the subcutaneous fat on the anterior surface of the lower legs, is associated with coccidioidomycosis (valley fever) and can suggest the diagnosis. It is a manifestation of the patient's immune response and often indicates a good prognosis. In addition to coccidioidomycosis, it can also be associated with streptococcal infections as well as tuberculosis. Erythema ab igne is a cutaneous rash caused by prolonged heat exposure (such as a heating pad) presenting as an otherwise asymptomatic, red, reticulated pattern on the skin. Erythema infectiosum is associated with parvovirus B19 infection and is usually seen in young children. It manifests as an erythematous rash of the face (slapped cheek appearance), arms, and legs. Erythema migrans is an expanding, erythematous, annular rash with or without central clearing and is often associated with tick exposure (Lyme disease). Erythema multiforme consists of raised, annular, target-like lesions with central erythema and is usually associated with herpes simplex virus type 1.

You are utilizing shared decision-making with patients while determining whether to recommend starting a statin for the primary prevention of atherosclerotic cardiovascular disease. In which one of the following patients would a screening coronary artery calcium score be most appropriate for guiding this recommendation? A) A 35-year-old at low (<5%) 10-year risk B) A 55-year-old at high (>20%) 10-year risk C) A 55-year-old with diabetes mellitus at high (>20%) 10-year risk D) A 60-year-old at low (<5%) 10-year risk E) A 60-year-old at intermediate (7.5% to <20%) 10-year risk

ANSWER: E For primary preventive interventions for the management of lipids, the 10-year atherosclerotic cardiovascular disease risk estimate is useful as a starting point for shared decision-making with patients. Specifically, it is a helpful tool when deciding on the use and intensity of statin therapy. The coronary artery calcium score can refine the risk assessment even further for those at intermediate predicted risk (>7.5% to <20%) or borderline predicted risk (5% to <7.5%). For those at intermediate or borderline risk with a coronary artery calcium score of 0, it would not be reasonable to start a statin. If the coronary artery calcium score is 100 or greater, starting a statin is acceptable in patients >55 years of age.

While reviewing laboratory studies for a patient who was recently started on antihypertensive medication, you note new hyperkalemia. Which one of the following medications is most likely to cause this finding? A) Amlodipine (Norvasc) B) Chlorthalidone C) Hydrochlorothiazide D) Metoprolol E) Olmesartan (Benicar)

ANSWER: E Hyperkalemia is a known side effect of ACE inhibitors and angiotensin receptor blockers such as olmesartan. The risk of hyperkalemia is increased with chronic kidney disease, diabetes mellitus, moderately severe to severe heart failure, NSAID use, and older adults. Chlorthalidone and hydrochlorothiazide can cause hypokalemia, while amlodipine and metoprolol have no significant effect on potassium levels.

Which one of the following interventions has the best evidence for effectiveness in the treatment of frailty syndrome in geriatric patients? A) Protein supplements B) Vitamin D supplements C) Hormonal treatment with anabolic steroids D) Aerobic conditioning training E) Progressive resistance training

ANSWER: E In geriatric patients with frailty syndrome, the intervention with the best evidence for effectiveness is progressive resistance training as a part of a physical activity program. The routine use of protein supplements, vitamin D supplements, or hormonal treatments, including anabolic steroids, is not recommended. Aerobic conditioning training may be helpful but the strongest evidence for efficacy is for activities that include progressive resistance training.

A 50-year-old male presents with chronic abdominal pain. A workup leads you to suspect peptic ulcer disease, and you refer him for endoscopy, which shows a small duodenal ulcer. The endoscopist also notes some small esophageal varices without red wale signs. Further evaluation confirms that the patient has compensated cirrhosis in the setting of alcohol use disorder. He readily accepts this diagnosis and enters an Alcoholics Anonymous program. His ulcer symptoms resolve with antibiotic therapy for Helicobacter pylori. He says he has abstained from alcohol for 6 weeks, and he would like to further reduce his risks from cirrhosis. The most appropriate next step in the management of his esophageal varices would be: A) octreotide (Sandostatin) B) omeprazole (Prilosec) C) propranolol D) endoscopic variceal ligation E) repeat endoscopy in 1-2 years

ANSWER: E Primary prevention of variceal hemorrhage is an important consideration in the management of patients with cirrhosis. Although this patient's varices were diagnosed incidentally, patients with cirrhosis and clinically significant portal hypertension should be screened for varices every 2-3 years with esophagogastroduodenoscopy (EGD). EGD can be deferred in patients with platelet counts <150,000/mm3 and transient elastography with liver stiffness <20 kPa. Once esophageal varices are identified, the criteria for initiating prophylaxis to prevent variceal hemorrhage is based on the risk of bleeding. Findings associated with a high risk of bleeding include small varices in patients with decompensated cirrhosis, small varices with red wale signs (thinning of the variceal wall), and medium to large varices. Patients with small varices not meeting these criteria have a low risk of hemorrhage and do not require prophylaxis. They should be rescreened with EGD every 1-2 years. For patients requiring treatment due to high-risk features, options for primary prophylaxis of hemorrhage include nonselective -blockers such as propranolol or endoscopic variceal ligation. Treatment decisions are based on patient preference, other potential contraindications, and local resources. The need for repeat endoscopy in these cases will depend on the clinical circumstances. If nonselective -blockers are used, they should be continued indefinitely. Octreotide is only given intravenously for acute hemorrhage. There is no evidence that omeprazole slows the progression of esophageal varices.

An 18-year-old football player collapses on the field at the beginning of summer conditioning workouts. There was no obvious contact or injury. Upon assessment, he is awake but somnolent and diaphoretic. He reports a headache and is unable to identify where he is or the day of the week. His core temperature is 40.2°C (104.4°F). Which one of the following is the most likely diagnosis? A) Exercise-associated collapse B) Heat edema C) Heat exhaustion D) Heat injury E) Heat stroke

ANSWER: E The prompt recognition of heatstroke is critical to effective treatment. Heatstroke is characterized by a core temperature >40°C (104°F) in association with neurologic abnormalities such as headache, confusion, altered mental status, irritability, and seizure. Exercise-associated collapse, previously called heat syncope, generally occurs immediately after strenuous exercise and is more associated with hydration status. Heat edema is a benign condition manifested by mild swelling in the extremities and facial flushing in a patient with a normal temperature. Heat exhaustion may involve neurologic symptoms but is associated with a lower temperature (38.3°C-40.0°C [101°F-104°F]) and thus a better outcome. Like heatstroke, heat injury can be associated with a temperature >40°C, but does not involve neurologic symptoms. Instead, kidney, muscle, or liver injury may be present.

A 70-year-old female presents to your office to discuss osteoporosis that was noted on a recent bone density test. Initial laboratory studies reveal an abnormal TSH level of 0.27 μU/mL (N 0.36-3.74). Additional studies reveal the following: Repeat TSH -- 0.04μU/mL FreeT3 -- 3.4pg/mL(N1.7-5.2) FreeT4 -- 1.4ng/dL(N0.7-1.6) A radioactive iodine uptake scan is notable for multiple areas of increased and suppressed uptake Which one of the following is the most likely explanation for these findings? A) Exogenous thyroid hormone use B) Graves disease C) Painless thyroiditis D) Recent excess iodine intake E) Toxic multinodular goiter

ANSWER: E This patient has a low serum TSH level in the presence of normal free T4 and total or free T3 levels, which is consistent with subclinical hyperthyroidism. The etiology of overt and subclinical hyperthyroidism should be determined by clinical symptoms, biochemical markers, and, if indicated, diagnostic studies such as a radioactive iodine uptake (RAIU) scan. A scan that shows multiple areas of increased and suppressed uptake is consistent with toxic multinodular goiter. There is no RAIU with exogenous ingestion of thyroid hormone, painless thyroiditis, and recent excess iodine intake. Graves disease causes diffuse RAIU. It is important to determine the etiology of subclinical hyperthyroidism in order to treat it appropriately. In patients who have TSH levels that are persistently <0.1 U/mL, the American Thyroid Association has a strong recommendation with moderate-quality evidence for treating patients 65 years of age and older; persons with cardiac risk factors, heart disease, or osteoporosis; postmenopausal women not on estrogens or bisphosphonates; and those with hyperthyroid symptoms.

Which one of the following is an individual risk factor for committing intimate partner violence? A) A belief in flexible gender roles B) Having many friends C) High income D) Planned pregnancy E) Young age

ANSWER: E Understanding individual, relational, societal, and community risk and protective factors associated with intimate partner violence (IPV) perpetration can help prevent it. Among the options listed, young age is an individual risk factor for committing IPV. IPV is most prevalent in adolescence and young adulthood and declines with age. A belief in strict gender roles, having few friends, low income, and unplanned pregnancy are also risk factors.

A 22-year-old female sees you to establish care. She describes firing her previous primary care physician because he was not adequately treating her. She states that she was diagnosed with borderline personality disorder 1 year ago and is seeking better treatment. Which one of the following would be the most appropriate first-line therapy? A) Lamotrigine (Lamictal), 25 mg daily B) Lithium, 300 mg twice daily C) Omega-3 fish oil (Lovaza), 4 g daily D) Quetiapine (Seroquel), 50 mg at bedtime E) Cognitive behavioral therapy

ANSWER: E Borderline personality disorder is characterized by emotional dysregulation, unstable self-image, and instability in interpersonal relationships. Patients with borderline personality disorder frequently overvalue and then rapidly devalue relationships, depending on perceived rejection. Cognitive behavioral therapy, specifically dialectical behavioral therapy and mentalization-based therapy, has shown the best efficacy in treating borderline personality disorder (SOR B). Pharmacologic treatments, including mood stabilizers, fatty acids, antipsychotics, and antidepressants, have been utilized in the treatment of borderline personality disorder despite limited or low-quality evidence, and would not be considered first-line therapy.

Chronic cough in an adult is defined as a cough that has been present for longer than 8 weeks. Which one of the following is the most common cause of chronic cough in an adult? A) Asthma B) Laryngopharyngeal reflux disease C) Nonasthmatic eosinophilic bronchitis D) Protracted bacterial bronchitis E) Upper airway cough syndrome

ANSWER: E Chronic cough in adults is a common presenting symptom for primary care visits. The four most common causes of chronic cough in adults include upper airway cough syndrome (UACS), asthma, nonasthmatic eosinophilic bronchitis, and reflux-related disorders. UACS, previously referred to as postnasal drip syndrome, is the most common cause of chronic cough in adults. This syndrome can have multiple etiologies, including chronic rhinosinusitis, allergic rhinitis, and nonallergic rhinitis. The diagnosis may be suggested by symptoms of rhinorrhea such as nasal stuffiness, sneezing, and postnasal drainage, but the absence of these symptoms does not rule out the diagnosis. The most common causes of chronic cough in children 6-14 years of age are asthma, protracted bacterial bronchitis, and UACS.

A 30-year-old male presents to your office after sustaining a scratch to the eye while playing with his 2-year-old nephew. A penlight examination reveals sensitivity to light and mild conjunctival irritation with no foreign body. Pupillary response, extraocular movements, and visual acuity are all normal. Fluorescein staining reveals a 3-mm corneal abrasion. Which one of the following would be the most appropriate management? A) Patching the affected eye B) Patching the unaffected eye C) Prednisolone ophthalmic drops D) Tetracaine ophthalmic drops E) Oral naproxen

ANSWER: E Corneal abrasions are a common cause of acute eye pain and are often evaluated in primary care settings. Small (<4 mm), uncomplicated abrasions typically heal within 1-2 days and usually respond to oral analgesics such as acetaminophen or NSAIDs. A 2013 review reported effective pain relief and earlier return to work with use of topical NSAIDs, although a 2017 Cochrane review subsequently found that evidence may be lacking to support their use, especially considering the higher cost when compared to oral options. Patching has been proven ineffective for pain relief and can delay healing, although ophthalmologists sometimes use patching to treat large abrasions or to provide a protective barrier for patients who may have difficulty avoiding rubbing their eyes, such as children or people with cognitive impairment. Patching of the unaffected eye is done to treat amblyopia but is not appropriate for managing corneal abrasions. Topical corticosteroids such as prednisolone are not appropriate for treatment of corneal abrasions due to increased susceptibility to infection and the risk of delayed healing and should only be used under the guidance of an ophthalmologist. While point-of-care use of topical anesthetics such as tetracaine may be considered, repeated administration is not recommended as continued use may cause damage to the corneal epithelium, delay healing, and mask symptoms. While topical antibiotics are often prescribed in the setting of corneal abrasion to prevent bacterial superinfection, evidence to support this practice in general is lacking. However, contact lens wearers are at increased risk of infection due to Pseudomonas aeruginosa and should be prescribed an antibiotic with antipseudomonal activity.

A 53-year-old female sees you for a routine health maintenance visit. The patient reports that she is newly menopausal and asks you about osteoporosis screening. Her past medical history includes morbid obesity, and her family history includes type 2 diabetes in her mother and hypertension in her father. The patient is a nonsmoker and rarely consumes alcohol. Her only medication is loratadine (Claritin), 10 mg daily. Which one of the following would you recommend regarding osteoporosis screening for this patient? A) No screening now or in the future, and calcium supplementation only B) No screening now or in the future, and calcium and vitamin D supplementation C) Radiography of her hip and lumbar spine now D) A DEXA scan now E) A DEXA scan at age 65

ANSWER: E For women with no risk factors, the U.S. Preventive Services Task Force (USPSTF) recommends screening for osteoporosis in women 65 years and older with bone measurement testing such as DEXA to prevent osteoporotic fractures (grade B recommendation). The USPSTF recommends screening for osteoporosis with DEXA in postmenopausal women younger than age 65 who are at increased risk of osteoporosis, as determined by a formal clinical risk assessment tool (B recommendation). Factors associated with an increased risk of osteoporosis include smoking, excessive alcohol consumption, low body weight, and a parental history of hip fracture. This patient is not at increased risk for osteoporosis, so a DEXA scan at age 65 would be most appropriate. Calcium and vitamin D supplementation to prevent osteoporosis are no longer routinely recommended. Plain radiography would not be recommended as screening for osteoporosis.

Which one of the following medical conditions is most likely the result of severely elevated triglycerides? A) Asthma B) Chronic kidney disease C) Gallstones D) Hypothyroidism E) Pancreatitis

ANSWER: E Hypertriglyceridemia, defined as triglyceride levels >500 mg/dL, increases the risk of pancreatitis. It does not increase the risk of asthma, chronic kidney disease, gallstones, or hypothyroidism. Patients with hypertriglyceridemia should initiate therapeutic lifestyle modifications and should be treated with fibrates or niacin to help reduce the risk of pancreatitis.

A 50-year-old male presents with a 3-month history of persistent burning and numbness in his anterolateral left thigh. He has not had any injury, back pain, radiation of pain, or weakness. He has not noticed any exacerbating or remitting factors. He has type 2 diabetes and a BMI of 37 kg/m2 , and his job is sedentary. An examination reveals normal deep tendon reflexes in the patella and ankle, and the straight leg raising test is negative bilaterally. His strength is preserved throughout his lower extremities. His pinprick sensation is slightly reduced along the anterolateral thigh. Burning discomfort is reproduced with tapping over the lateral aspect of the inguinal ligament. Which one of the following is the most likely diagnosis? A) Cauda equina syndrome B) Diabetic neuropathy C) Femoral neuropathy D) Left S1 radiculopathy E) Meralgia paresthetica

ANSWER: E Meralgia paresthetica is a common cause of anterolateral hip pain and dysesthesia. It is caused by compression of the lateral femoral cutaneous nerve as it courses under the inguinal ligament into the subcutaneous tissue of the thigh. Tapping over this area during the examination can reproduce symptoms. Obesity is a common cause due to compression of the nerve from overlying pannus. Diabetes mellitus is associated with a sevenfold higher incidence over the general population. Cauda equina syndrome presents with saddle anesthesia and generally marked neurologic disability. Diabetic neuropathy is a peripheral neuropathy initially affecting distal structures such as the toes and feet. Femoral neuropathy would affect sensation in the anteromedial thigh and medial lower leg with weakness in the quadriceps muscle group. The anterolateral thigh would represent the L3-L4 dermatome rather than S1, and the normal straight leg raising test and absence of back pain are evidence against an S1 issue.

A 55-year-old female with a BMI of 50 kg/m2 and recently diagnosed severe obstructive sleep apnea (OSA) presents for follow-up after a sleep study. She was unable to tolerate positive pressure therapy. Her OSA could be most effectively addressed by which one of the following interventions? A) Use of a nasal dilator device B) A positional sleep alarm to avoid the supine position C) Clonidine, 0.1 mg orally before bedtime D) Uvulopalatopharyngoplasty E) Bariatric surgery

ANSWER: E Obstructive sleep apnea (OSA) is a common disorder that, if left untreated, can be associated with other serious health conditions such as atrial fibrillation, depression, heart failure, and stroke. Positive pressure therapy is effective and considered the first-line treatment for OSA, although some patients are unable to tolerate this therapy. In obese patients with OSA, bariatric surgery has been shown to reliably result in improvement in >75% of patients and result in remission in 40% of patients after 2 years. Nasal dilator devices and pharmacologic interventions such as clonidine have not been shown to improve symptoms or to be effective for treatment. Positional therapy is not recommended as a long-term solution for severe OSA due to poor long-term compliance. Currently there is insufficient evidence to support oral procedures such as uvulopalatopharyngoplasty as primary interventions for OSA.

A 56-year-old male was recently diagnosed with hypertension and started on lisinopril (Zestril). At a follow-up visit his blood pressure remains elevated and his serum creatinine level has increased from 0.9 mg/dL to 1.8 mg/dL (N 0.7-1.3). He has no other known medical issues and has a normal BMI. Which one of the following should be ordered to confirm the most likely cause of his hypertension? A) Renin and aldosterone levels B) A TSH level C) 24-hour urinary free cortisol D) 24-hour urinary fractionated metanephrines and normetanephrines E) CT angiography of the abdomen and pelvis

ANSWER: E Resistant hypertension occurs in 5%-10% of adults with hypertension. In this patient, renal artery stenosis is suggested by the increase in creatinine of more than 50% after starting an ACE inhibitor. CT angiography, renal artery duplex ultrasonography, and MR angiography are appropriate diagnostic tests for renal artery stenosis. Other causes of resistant hypertension include hyperaldosteronism (diagnosed with renin and aldosterone levels), thyroid disorders (diagnosed with TSH levels), Cushing syndrome (diagnosed with 24-hour urinary free cortisol), and pheochromocytoma (diagnosed with 24-hour urinary fractionated metanephrines and normetanephrines).

55-year-old female comes to your clinic for follow-up of her poorly controlled hypertension. Her medical history also includes type 2 diabetes and worsening obstructive sleep apnea (OSA). Her BMI is 52 kg/m2 . Her heart rate is 62 beats/min and regular. Her blood pressure in the clinic today is 160/96 mm Hg, and she reports similar average readings at home. She is asymptomatic and says she has been following lifestyle modifications including a low-salt diet. She also reports that she has been adherent with her current antihypertensive regimen, which includes the following: Amlodipine (Norvasc), 10 mg daily Carvedilol (Coreg), 25 mg twice daily Chlorthalidone, 25 mg daily Losartan (Cozaar), 100 mg daily Which one of the following antihypertensive medication changes would benefit both her blood pressure and her OSA? A) Switching chlorthalidone to hydrochlorothiazide, 25 mg daily B) Switching carvedilol to an equivalent dosage of metoprolol tartrate (Lopressor) C) Switching losartan to an equivalent dosage of an ACE inhibitor D) Increasing the current dosage of losartan to 100 mg twice daily E) Adding a low dosage of spironolactone (Aldactone) to her current regimen

ANSWER: E Secondary forms of hypertension are common in patients with resistant hypertension, and sleep-disordered breathing is an important cause of resistant hypertension. Multiple studies have shown that excess aldosterone plays a key role in the association between the two. As rates of obesity and obstructive sleep apnea (OSA) have increased, the prevalence of resistant hypertension has also increased. It is estimated that almost 17%-22% of patients with resistant hypertension may have undiagnosed primary hyperaldosteronism. The increased expression of mineralocorticoid receptors in patients with a high BMI contributes to hyperaldosteronism, and blockage of these receptors with medications such as spironolactone has been shown to provide benefit in reducing the severity of OSA as well as hypertension in these patients. Substituting one thiazide for another does not have a beneficial effect in patients with resistant hypertension. Switching beta-blockers in this case is unlikely to have a significant impact on blood pressure and might have an adverse impact, as carvedilol has been shown to have more favorable effects on glycemic control and other components of metabolic syndrome relative to metoprolol tartrate in patients with diabetes. Increasing the angiotensin receptor blocker dosage or substituting an ACE inhibitor will not be as beneficial in controlling this patient's blood pressure.

A 78-year-old male presents to the emergency department (ED) after experiencing a syncopal event witnessed by his spouse. The patient reports that he has been easily fatigued and has not been able to walk as long as usual during the past few months. He recently saw his cardiologist and completed Holter monitoring but has not received the results yet. Laboratory studies during the ED visit were unremarkable. However, an EKG showed sinus bradycardia with 44 beats/min as well as pauses of more than 3 seconds, during which the patient reported dizziness. He says he wants to recover quickly so he can resume his regular activities. Which one of the following is the first-line treatment for the presumed diagnosis? A) Atropine B) Cilostazol C) Dopamine D) Glucagon E) Permanent pacemaker placement

ANSWER: E Sinus node dysfunction, also known as sick sinus syndrome, is defined by an abnormal initiation and propagation of electrical impulses from the sinoatrial node, causing sinus pauses of more than 3 seconds, sinus arrest, and bradycardia (heart rate <50 beats/min). Due to the resulting hypoperfusion, patients may experience decreased tolerance of physical activity, palpitations, dizziness, easy fatigability, and syncope. The diagnosis is made using heart rate monitoring. If symptoms are associated with exertion, an exercise stress test should be performed. Patients not reaching a heart rate of at least 80% of their predicted maximum (220 beats/min minus age) may have chronotropic incompetence. If the diagnosis of sinus node dysfunction is confirmed, the first-line treatment is placement of a permanent pacemaker. Medication control of sinus node dysfunction, such as with cilostazol, is an option for patients unwilling to receive a permanent pacemaker, which is not the case in this patient. Atropine, dopamine, and glucagon are used in advanced cardiac life support protocols for patients who are acutely unstable. These medications are not appropriate for long-term management.

A healthy 40-year-old male is concerned about his risk for myocardial infarction (MI) because his father had an MI at age 45. The patient is a nonsmoker and does not take any medications. He states that he plans to start a regular exercise program, and asks for your advice regarding the best dietary approach for him. His vital signs are normal, including his BMI. Which one of the following would be most likely to reduce this patient's cardiovascular risk? A) Intermittent fasting (fasting for up to 16 hours each day, or eating only one meal on certain days) B) A low-fat, low-cholesterol diet C) A low-carbohydrate diet (Atkins diet) D) A very-low-carbohydrate, high-fat diet (ketogenic diet) E) A Mediterranean diet

ANSWER: E The Mediterranean diet has moderate to strong evidence for reducing the incidence of cardiovascular disease and associated mortality, preventing type 2 diabetes, decreasing overall mortality, and treating obesity. Intermittent fasting has been shown to be effective in weight loss, although not clearly more effective than overall calorie restriction, but a decrease in cardiovascular risk has not been shown. Low-fat, low-cholesterol diets may lead to substituting foods with increased sugar and overall calories. A low-carbohydrate diet has been shown to have more beneficial effects on lipid profiles than a low-fat diet. Additionally, mono- and polyunsaturated fats are actually beneficial in cardiovascular health, so focusing on a low-fat diet may be counterproductive. Low-carbohydrate diets can be useful to promote weight loss and decrease the incidence of type 2 diabetes, but their impact on cardiovascular disease is less clear. It is recommended that less than 5%-10% of total calories should come from added sugars, but a diet very low in carbohydrates may excessively limit healthy carbohydrates such as those found in whole grains, fruits, and vegetables.

The results of a meta-analysis of lung cancer screening using low-dose CT were as follows: the pooled lung cancer-specific mortality rate in the control group was 2.12%, the estimated lung cancer-specific mortality rate in the screened population was 1.72%, and the absolute risk reduction for lung cancer mortality was 0.4% (2.12% minus 1.72%). What is the number needed to screen to prevent one death due to lung cancer?

ANSWER: E The absolute risk reduction (ARR) in this meta-analysis was 0.4%. The number needed to screen is the reciprocal of the ARR. The number needed to screen would equal 1 divided by the ARR (1/ARR), or 1/0.004, which equals 250. Based upon this meta-analysis, 250 individuals would need to be screened to prevent one lung cancer death.

A 23-year-old female with a history of systemic lupus erythematosus presents to discuss contraceptive management. You note that she is no longer taking hydroxychloroquine (Plaquenil) and has not seen her rheumatologist in over a year. She reports generalized fatigue and intermittent joint pain. She is agreeable to obtaining laboratory studies at today's visit. In addition to checking anti-double-stranded DNA, complement levels, an erythrocyte sedimentation rate, a C-reactive protein level, a CBC, and a comprehensive metabolic panel, which one of the following laboratory tests would be most helpful in monitoring her disease activity? A) An anti-cyclic citrullinated peptide level B) An antinuclear antibody titer C) A creatine kinase level D) A fasting lipid panel E) A urinalysis with microscopic examination

ANSWER: E The majority of patients with systemic lupus erythematosus (SLE) follow a relapsing-remitting course, and management requires monitoring clinical symptoms and laboratory studies. Prolonged remission and persistently active disease are both infrequent. In addition to monitoring anti-double-stranded DNA, complement levels, erythrocyte sedimentation rates, C-reactive protein levels, WBC counts, platelet counts, serum creatinine levels, and glomerular filtration rates, monitoring urine studies for proteinuria is important to assess disease activity over time. Lupus nephritis is a significant cause of morbidity and mortality associated with SLE. Anti-cyclic citrullinated peptide and creatine kinase levels are associated with rheumatoid arthritis and polymyositis, respectively. These tests may assist in ruling out other autoimmune diseases but are not helpful once a diagnosis of SLE is made. While an antinuclear antibody (ANA) titer is helpful in making a diagnosis of SLE, it does not assist with assessing disease activity over time and should not be checked once it is known to be positive. Monitoring lipid levels and atherosclerotic cardiovascular risk is important in managing SLE, but lipid levels do not correlate with flares of SLE.

A 69-year-old male is found to have an infrarenal abdominal aortic aneurysm (AAA) on screening ultrasonography. Which one of the following is most important when determining the risk of his AAA rupturing? A) His age B) His sex C) His history of hypertension D) His history of smoking E) The diameter of his aneurysm

ANSWER: E The single most important determinant of the risk that an abdominal aortic aneurysm (AAA) will rupture is the diameter of the aneurysm. In men, aneurysm repair is recommended when the aneurysm reaches 5.5 cm in diameter. In women, whose aortas tend to be smaller, the recommended maximum diameter is 5.0 cm. Age, sex, a history of hypertension, and a history of smoking all increase the risk of developing an AAA, but do not increase the risk of rupture.

Which one of the following vaccines is CONTRAINDICATED during pregnancy? A) Hepatitis A B) Hepatitis B C) Rabies D) Tdap E) Varicella

ANSWER: E The varicella vaccine is a live virus vaccine that is contraindicated during pregnancy because of the potential for fetal infection. The indications for hepatitis A, hepatitis B, rabies, and Tdap vaccines are the same for pregnant patients and nonpregnant patients.

A 41-year-old male who is a construction worker sees you because he experienced palpitations followed by an episode of syncope after drinking four beers at a hotel bar while on vacation. He says that he was hospitalized briefly, and an electric shock was administered in the emergency department after medications were given. On discharge he was told that he had a brief episode of atrial fibrillation but that an echocardiogram was normal. He was advised to follow up promptly with his personal physician. An EKG performed in your office is shown below. Which one of the following would you recommend for this patient? A) Aspirin B) Oral flecainide to be taken if a similar episode occurs in the future C) Oral metoprolol therapy D) Warfarin therapy E) Catheter ablation therapy

ANSWER: E This EKG shows a short P-R interval with a slurred upstroke in the QRS complex in the precordial leads. These findings are classic for Wolff-Parkinson-White (WPW) syndrome. In symptomatic patients with sustained ventricular tachycardias, especially with the more dangerous paroxysmal atrial fibrillation, catheter ablation is the preferred therapy. Up to 95% success has been reported. Aspirin alone would be ineffective for anticoagulation in atrial fibrillation, and would not be indicated for this young patient with no other risk factors for stroke. "Pill-in-the-pocket" therapy with agents such as flecainide or propafenone can be used for paroxysmal atrial fibrillation in structurally normal hearts. However, they are not the treatment of choice with WPW syndrome. Beta-Blockers, adenosine, digitalis, diltiazem, and verapamil are contraindicated because they can facilitate conduction on the accessory pathway. This can lead to more rapid atrial fibrillation or even degeneration into ventricular fibrillation. A young patient with a structurally normal heart does not require anticoagulation with warfarin.

A 28-year-old female presents with a 2.5-cm pruritic, erythematous, oval macule on her left thigh. She was seen at an urgent care facility 2 days ago for a urinary tract infection (UTI) and was treated with sulfamethoxazole/trimethoprim (Bactrim). Her UTI symptoms have improved. She reports that she was called earlier this morning and told that her infection was caused by Escherichia coli. The patient reports a similar lesion in the same area about a year ago at the time of her last UTI. You explain this is most likely secondary to: A) an immunologic reaction to E. coli B) erythema multiforme C) nummular eczema D) the Shiga toxin sometimes produced by E. coli E) the sulfamethoxazole/trimethoprim used to treat the infection

ANSWER: E This is a typical history for a fixed drug eruption (FDE), which is an immunologic reaction that recurs upon re-exposure to the offending drug. It is most likely related to T-lymphocytes at the dermal-epidermal junction. Sulfonamides and anticonvulsants are the most frequently cited medications, but tetracycline and penicillins have also been reported to cause FDE. FDE is not caused by bacteria. Erythema multiforme does not present as an isolated, recurrent macule and generally has central clearing. Nummular eczema is a coin-shaped, very pruritic patch but does not fit this clinical scenario. Shiga toxin-producing Escherichia coli are rarely found in extra-intestinal sites.

A 29-year-old male presents to your office because of pain and paresthesia in his right fourth and fifth fingers for the last several weeks. He has had some generalized weakness in his hands, noting that it is more difficult for him to grasp and pick up small objects with his thumb and forefinger. There is no history of trauma. He is very physically active and lifts weights 5-6 days per week. On physical examination you note weakness of the pincer mechanism and decreased sensation over the hypothenar eminence and fourth and fifth fingers. These findings are associated with peripheral entrapment of which one of the following nerves? A) Axillary B) Median C) Radial D) Suprascapular E) Ulnar

ANSWER: E This patient has entrapment of the ulnar nerve at the wrist level. This is more common in activities that place pressure on the volar aspect of the wrist, including weightlifting and cycling. Classic symptoms include paresthesia of the fourth and fifth fingers and hypothenar eminence, weakness in finger adduction and abduction, and weakness of the pincer mechanism. Axillary nerve entrapment can result from shoulder dislocations, humeral neck fracture, and pressure from crutch use, and can cause decreased sensation or pain over the lateral shoulder as well as weakness with shoulder external rotation, abduction, and extension. Suprascapular nerve entrapment can present similarly to axillary nerve entrapment, with shoulder pain and abduction and forward flexion deficits. Median nerve entrapment results in paresthesia of the first three fingers and can result in thenar muscle atrophy. The radial nerve can be entrapped or compressed at many different locations, most commonly due to sustained pressure over the radial groove. This will result in paresthesia and pain in the posterior forearm and dorsal hand as well as weakness in wrist and finger extensors, which can result in wrist and finger drop.

A 17-year-old cross country runner sees you to discuss the results of pulmonary function tests to evaluate his episodic shortness of breath and chest tightness. He had previously been diagnosed with exercise-induced asthma and prescribed albuterol (Proventil, Ventolin), which provided minimal relief. You tell him that the pulmonary function tests revealed normal expiratory findings including normal FEV1 and FVC and a flattened inspiratory flow loop. The test most likely to confirm a diagnosis for this patient's shortness of breath is: A) a sleep study B) chest radiography C) chest CT D) esophagogastroduodenoscopy E) nasolaryngoscopy

ANSWER: E This patient has vocal cord dysfunction, sometimes called paradoxical vocal fold motion, a condition in which the vocal cords close during inspiration when they should be open. It is not entirely understood why this occurs but it is associated with other conditions including asthma, GERD, and anxiety disorders. It typically causes sudden, severe shortness of breath and often has a trigger such as exercise, gastroesophageal reflux, inhalation of an irritant, or stress. Symptoms may include chest or throat tightness, inspiratory stridor, and wheezing predominantly over the upper airway. In less severe situations the voice may be impacted, and patients sometimes also describe a chronic cough that occurs separately from more acute symptoms. Vocal cord dysfunction is confirmed by direct visualization of the vocal cords during inspiration via nasolaryngoscopy. Pulmonary function tests are often performed as part of the assessment for shortness of breath and, if performed while the patient is experiencing symptoms, will show a flattened inspiratory flow loop. Treatment is primarily focused on therapeutic breathing maneuvers and vocal cord relaxation techniques. A speech therapist may assist in instructing patients in these techniques. Associated conditions should also be treated to help prevent vocal cord dysfunction. A sleep study, chest radiography, chest CT, and esophagogastroduodenoscopy would not confirm a diagnosis of vocal cord dysfunction.

A 42-year-old male presents for a routine health maintenance visit and asks about hepatitis C virus (HCV) testing. He reports that he experimented with intravenous drugs in college but never developed a habit. He consumes 1-2 alcoholic drinks a week and is a nonsmoker. He is not married but is in a monogamous, heterosexual relationship. Which one of the following would confirm active hepatitis C infection? A) Anti-HCV antibody testing B) AST and ALT levels C) An AST/platelet ratio index D) A CBC with differential and peripheral smear E) HCV RNA polymerase chain reaction testing

ANSWER: E This patient is at risk for hepatitis C virus (HCV) infection due to his history of intravenous drug use. The initial screening test for HCV is anti-HCV antibody testing, and a positive result could indicate either a prior infection that cleared or an active infection. If the antibody test is positive, HCV RNA polymerase chain reaction testing is the next step to confirm an active infection. AST and ALT levels and a CBC should be obtained if the patient is diagnosed with an active HCV infection. The AST/platelet ratio index is a screening test for hepatic fibrosis and cirrhosis.

A 56-year-old male comes to your office for a routine health maintenance examination. He has not seen a medical provider in the past 10 years. He has one sex partner, exercises regularly, and follows a Mediterranean diet. He has a 10-pack-year smoking history and quit 20 years ago. He has a family history of type 2 diabetes. Based on U.S. Preventive Services Task Force guidelines, which one of the following screening recommendations requires shared decision-making with the patient? A) Office blood pressure measurement B) A depression assessment with the Patient Health Questionnaire-9 C) A whole-body skin examination D) A fasting lipoprotein profile E) Prostate-specific antigen testing

ANSWER: E This patient presents for a health maintenance examination in which evidence-based guidelines are used to promote health, screen for chronic disease, prevent complications of chronic disease, and recommend age-appropriate cancer screenings. Because both screening and not screening for prostate cancer with a prostate-specific antigen test are reasonable options, shared decision-making with the patient and consideration of his particular situation are necessary. For a male in his fifties, the U.S. Preventive Services Task Force (USPSTF) recommends hypertension screening with office blood pressure measurement, depression screening with the Patient Health Questionnaire-9, and cardiovascular disease screening with a lipoprotein profile. A whole-body skin examination for skin cancer screening is not recommended by the USPSTF due to insufficient evidence regarding the benefits versus the harms.

A 35-year-old female presents with a 3-week history of radial-sided wrist pain without acute trauma. Her symptoms are most prominent when lifting her 4-month-old infant into his car seat. Which one of the following physical examination findings is most consistent with the likely diagnosis? A) Focal tenderness in the anatomic snuffbox B) Focal tenderness over the triangular fibrocartilage complex C) Pain reproduced by repeated percussion over the volar wrist D) Pain reproduced by thumb passive circumduction with axial load to the first carpometacarpal joint E) Pain reproduced by ulnar wrist deviation with the thumb placed inside a closed fist

ANSWER: E This patient presents with a history of an atraumatic onset of radial-sided wrist pain that is typical of de Quervain's tenosynovitis, a common overuse injury involving the tendons of the first dorsal compartment, specifically the abductor pollicis longus and extensor pollicis brevis. This injury is most common in women 30-50 years of age and often occurs in new mothers who frequently pick up a child. Pain localizes to the radial styloid and is reproduced by Finkelstein's test, which involves placing the thumb inside a closed fist followed by ulnar deviation of the wrist. Focal tenderness in the anatomic snuffbox would suggest injury to the scaphoid, which is in the differential diagnosis for radial-sided wrist pain, although it usually occurs through direct trauma such as a fall onto an outstretched hand. Injury to the triangular fibrocartilage complex is a common cause of ulnar-sided, rather than radial-sided, wrist pain. Repeated percussion over the volar wrist with resultant pain and/or paresthesia in a median nerve distribution describes Tinel's sign for carpal tunnel syndrome. Thumb passive circumduction with an axial load to the first carpometacarpal (CMC) joint describes the CMC grind test, which elicits pain from CMC joint arthritis.

A 67-year-old male presents for follow-up of ongoing chest pain that he experiences when walking up hills. His medical history is significant for hypertension and coronary artery disease. Four months ago he had a positive exercise stress test and underwent coronary angiography, which showed diffuse atherosclerotic disease but no lesions suitable for percutaneous intervention. His current medications include aspirin, 81 mg; atorvastatin (Lipitor), 80 mg; and metoprolol succinate (Toprol-XL), 100 mg. His vital signs include a blood pressure of 120/66 mm Hg and a pulse rate of 68 beats/min. Recent laboratory studies are significant for an LDL-cholesterol level of 58 mg/dL, a triglyceride level of 120 mg/dL, and a troponin level of 0.05 ng/mL (N <0.04). The addition of which one of the following agents would decrease this patient's all-cause mortality risk? A) Clopidogrel (Plavix), 75 mg daily B) Colchicine (Colcrys), 0.6 mg daily C) Icosapent ethyl (Vascepa), 2 g twice daily D) Isosorbide mononitrate, 30 mg daily E) Rivaroxaban (Xarelto), 2.5 mg twice daily

ANSWER: E This patient presents with stable angina and documented coronary atherosclerosis. His slight troponin elevation is a marker of elevated risk. The addition of low-dose rivaroxaban to aspirin has been shown to decrease cardiac and all-cause mortality in patients with coronary artery disease (CAD) and may be offered to this patient (SOR A). Dual antiplatelet therapy with clopidogrel and aspirin is recommended for 1 year after stenting but is not recommended in patients with stable angina who do not have stents. Colchicine has been associated with decreased cardiac events in patients with CAD but may increase all-cause mortality. Icosapent ethyl has been shown to decrease cardiac events but not mortality in patients with hypertriglyceridemia >150 mg/dL. Isosorbide mononitrate may be indicated to improve angina symptoms but does not improve mortality risk.

A 64-year-old male is hospitalized with anorexia, intractable abdominal pain, and dehydration due to locally advanced pancreatic cancer. He is started on intravenous fluids and morphine, along with a prophylactic dose of subcutaneous heparin. Shortly after admission he develops right-sided chest pain and shortness of breath. His vital signs are normal, except for a respiratory rate of 24/min. An abdominal examination reveals tenderness in the epigastric area. An examination of the heart and lungs is normal. There is no calf tenderness or leg edema. An EKG shows new right bundle branch block. Which one of the following tests should you order next? A) A D-dimer level B) A troponin level C) Doppler ultrasonography of the lower extremities D) A ventilation-perfusion (V/Q) scan E) Computed tomography pulmonary angiography (CTPA)

ANSWER: E This patient's pretest probability for pulmonary embolism is high given his multiple risk factors, signs, and symptoms. The presence of a new onset of right bundle branch block in a patient presenting with a sudden onset of shortness of breath and chest pain, especially in the setting of active cancer, should raise suspicion of pulmonary embolism. Other EKG abnormalities include tachycardia or bradycardia, an S1Q3T3 pattern, atrial fibrillation, and T-wave inversions in the anterior leads. Patients with cancers of the pancreas and stomach have the highest risk of developing venous thromboembolism (VTE) and should receive pharmacologic VTE prophylaxis during hospitalizations. In the absence of renal failure, a computed tomography pulmonary angiogram (CTPA) is the most appropriate diagnostic study and would be preferred over a ventilation-perfusion (V/Q) scan. A D-dimer level has a high negative predictive value in the diagnosis of pulmonary embolism; however, it has low specificity, and therefore a high rate of false positives, in patients with active cancer. An elevated troponin level can occur in the setting of pulmonary embolism and is nondiagnostic. Their principal value is in the diagnosis of acute myocardial infarction (SOR A). Doppler ultrasonography of the lower extremities helps identify and locate peripheral deep vein thrombosis and helps support but not confirm the diagnosis of pulmonary embolism. A V/Q scan is a reasonable option when CTPA is contraindicated.

The most effective therapy to improve quality of life in patients with tinnitus is A) a benzodiazepine B) an SSRI C) transcutaneous electrical nerve stimulation (TENS) D) acupuncture E) cognitive behavioral therapy

ANSWER: E Tinnitus is the sensation of hearing an abnormal sound, such as a ringing, buzzing, or clicking, that is perceived in the ear or head in the absence of an internal or external source. Cognitive behavioral therapy is the only treatment that has been shown to improve quality of life in patients with tinnitus. Treatments that should be avoided include benzodiazepines, transcutaneous electrical nerve stimulation (TENS), and acupuncture. An SSRI could be considered for the management of tinnitus-associated anxiety, but is not considered the most effective therapy for tinnitus.

A 43-year-old male sees you because of popping and clicking at the base of his index finger. On examination you note a nodule on the palmar aspect of the metacarpophalangeal joint with the finger flexed. Which one of the following is the most likely diagnosis? A) Calcific peritendinitis B) Dupuytren contracture C) Flexor tenosynovitis D) Rheumatoid arthritis E) Trigger finger

ANSWER: E Trigger finger, which can be associated with diabetes mellitus, presents with locking, clicking, or popping at the base of the finger or thumb. The finger may lock when flexed. Treatment consists of corticosteroid injection or splinting, and surgery may be necessary. Calcific peritendinitis causes pain, tenderness, and edema. Dupuytren contracture is manifested as a palpable cord in the palm and is not associated with locking. Flexor tenosynovitis causes fusiform digit swelling and is associated with rheumatoid arthritis. Rheumatoid arthritis involves multiple joints.

A 4-month-old female is brought to your clinic for a routine well child visit. She has not been seen by a physician since 2 weeks of age due to parental concerns about seeking care during the COVID-19 pandemic. Her growth and development appear to be normal. Which one of the following vaccines is CONTRAINDICATED today? A) Haemophilus influenzae type b B) Hepatitis B C) Inactivated poliovirus D) 13-valent pneumococcal conjugate (PCV13, Prevnar 13) E) Rotavirus

ANSWER: E Vaccine delay and vaccine hesitancy are on the rise in the United States, so family physicians should be familiar with the nuances of vaccine catch-up schedules as well as contraindications. Rotavirus vaccine has age restrictions and should not be initiated after 14 weeks and 6 days of age. In addition, the rotavirus series must be complete by 8 months of age. These age restrictions are intended to ensure the vaccine is administered when it will be of maximal benefit to children given the slightly increased risk of intussusception after vaccine administration. Hepatitis B vaccine should be administered at routine intervals. Haemophilus influenzae type b, inactivated poliovirus, and pneumococcal conjugate can all be administered to this patient today. However, these vaccines have complex follow-up intervals based on the age at prior doses and age at catch-up. Therefore, the clinician should consult the CDC catch-up vaccine schedule to verify dosing intervals.

A 60-year-old male with symptomatic low testosterone is started on a testosterone patch. You should order specific periodic monitoring of his PSA level and A) no other laboratory studies B) ALT and AST levels C) BUN and creatinine levels D) C-reactive protein level E) hematocrit

ANSWER: E When testosterone therapy is started, baseline and periodic measurements of PSA and hematocrit should be performed. If the hematocrit increases to >54% the testosterone dosage should be stopped or decreased to avoid hemoconcentration. Liver and renal function tests should be monitored routinely with many drugs but not specifically with testosterone. The C-reactive protein level is not monitored during testosterone therapy.

The American Academy of Pediatrics recommends obtaining a blood pressure reading at well child checks beginning at what age?

The American Academy of Pediatrics recommends annual blood pressure screening at well child checks beginning at 3 years of age. This recommendation does not differ for children who have a strong family history of hypertension.

The diagnosis of type 2 diabetes can be confirmed by two hemoglobin A1c values at or above a threshold of ____ %

The diagnostic cutoff point for type 2 diabetes is a fasting plasma glucose level >126 mg/dL or a hemoglobin A1c >6.5%. The diagnosis requires confirmation by repeat testing or by obtaining both a fasting glucose level and hemoglobin A1c.


Set pelajaran terkait

Chp 03, 04, 05. Federal Goverment

View Set

05.04 Confidence Intervals for Proportions

View Set

Chapter 9 Test Questions AP Biology

View Set